OB Exam 1 wks 1-2

अब Quizwiz के साथ अपने होमवर्क और परीक्षाओं को एस करें!

During pregnancy, the cardinal rule concerning medications and herbal remedies is that all drugs cross the placenta and have a potential impact on the fetus. What is one disease where treatment must continue during pregnancy? a. asthma b. gout c. cholelithiasis d. gastritis

a. Treatment (including medications) for certain diseases and conditions must continue during pregnancy. Examples include epilepsy, asthma, diabetes, and depression.

The nurse explains to a pregnant woman that the germ layers that develop in the embryo and become different organs and tissues consist of which layers? Select all that apply. a. ectoderm b. epithelium c. mesoderm d. endoderm e. subcutaneous

a, c, d. The three germ layers in the embryo are the ectoderm, mesoderm, and endoderm. The others are layers of the skin.

Nurses are expected to know how to use the first genetic test. What is it? a. the developmental assessment b. the family history c. the physical assessment d. the psychosocial assessment

b. The family history is considered the first genetic test. It is expected that all nurses will know how to use this genetic tool.

A nurse at the health care facility assesses a client at 20 weeks' gestation. The client is healthy and progressing well, without any sign of complications. Where should the nurse expect to measure the fundal height in this client? a. at the top of the symphysis pubis b. halfway between the symphysis pubis and the umbilicus c. at the level of the umbilicus d. at the xiphoid process

c. In the 20th week of gestation, the nurse should expect to find the fundus at the level of the umbilicus. The nurse should palpate at the top of the symphysis pubis between 10 to 12 weeks' gestation. At 16 weeks' gestation, the fundus should reach halfway between the symphysis pubis and the umbilicus. With a full-term pregnancy, the fundus should reach the xiphoid process.

A client in her third trimester of pregnancy wishes to formula feed her baby. What instruction should the nurse provide? a. Mix one scoop of powder with an ounce of water. b. Feed the infant every 8 hours. c. Serve the formula at room temperature. d. Refrigerate any leftover formula.

c. The nurse should instruct the client to serve the formula to her infant at room temperature. The nurse should instruct the client to follow the directions on the package when mixing the powder because different formulas may have different instructions. The infant should be fed every 3 to 4 hours, not every 8 hours. The nurse should specifically instruct the client to avoid refrigerating the formula for subsequent feedings. Any leftover formula should be discarded.

Why is a Papanicolaou test done at the first prenatal visit? a. It predicts whether cervical cancer will occur. b. It helps to date the pregnancy. c. It detects if uterine cancer is present. d. It identifies abnormal cervical cells.

d. A Pap test is a test for cervical cancer. Should abnormal cells be present, the woman may need to make a decision about her priorities of therapy for cervical disease or continuing the pregnancy.

When preparing a teaching plan for a group of women during their first pregnancy, the nurse reviews how maternity care has changed over the years. Which information would the nurse include when discussing events occurring in the 20th century? a. epidemics of puerperal fever b. performance of the first cesarean birth c. x-ray developed to assess pelvic size d. development of birthing centers

d. In the 20th century (1900s), birth centers were developed. Puerperal fever epidemics, the first cesarean birth, and X-ray to assess pelvic size were events occurring during the 19th century (1800s).

A nurse is reading a journal article about premenstrual syndrome (PMS) and the millions of women affected by it during their reproductive years. The nurse would expect to find approximately what percentage of women meeting the American College of Obstetricians and Gynecologists criteria for PMS? a. 45% b. 55% c. 65% d. 75%

d. It is estimated that up to 75% of reproductive-age women experience premenstrual symptoms that meet the ACOG criteria for PMS and up to 8% meet the diagnostic criteria for premenstrual dysphoric disorder.

A nurse is working as part of a research team evaluating children for participation in a research study. The nurse demonstrates respect when the nurse obtains assent from children over which age? a. 18 b. 16 c. 12 d. 7

d. Respect for persons requires that the subject is able to independently and autonomously volunteer to participate in a study. Because infants and children do not have this ability, parents must consent on behalf of their children. Cognitively intact children over the age of 7 years should always be asked to give their assent.

A group of nursing students are comparing male and female reproductive organs. The instructor determines the class is successful when the students correctly deduce the male glans penis is complementary to which female structure? a. vulva b. vagina c. labia majora d. clitoris

d. The female clitoris is complementary (equitable) to the glans; both of these are composed of erectile tissue and are the primary organs responsible for orgasm. Other comparable structures include the testes and ovaries, and scrotum and labia majora. The vagina does not have a comparable male structure.

The nurse is explaining about the function of the mons pubis to a client. Which explanation describes the function of the mons pubis? a. To enhance sexual pleasure b. To serve as the receptacle for sperm c. To allow for stretching during birth d. To protect the pelvic bones during sexual intercourse

d. The mons pubis is a fatty pad that lies over the pubic symphysis and serves to protect the pelvic bones during intercourse. The vagina is the receptacle for sperm, the clitoris is the body that enhances sexual pleasure in the female, and the perineum and pelvic floor are the structures that stretch during birth.

When using the contraceptive patch, a client should understand that it: a. should be applied to the breasts, hips, or back. b. should be covered when swimming in a pool because of chlorine's effect on the adhesive. c. is immediately effective after application. d. should be applied to the abdomen, buttocks, or back.

d. The patch should be applied only to the buttocks, back, abdomen, or torso (never the breasts). The patch is safe for wearing during swimming and bathing. The patch requires application for 1 week before becoming effective.

A client in her 29th week of gestation reports dizziness and clamminess when assuming a supine position. During the assessment, the nurse observes there is a marked decrease in the client's blood pressure. Which intervention should the nurse implement to help alleviate this client's condition? a. Keep the client's legs slightly elevated. b. Place the client in an orthopneic position. c. Keep the head of the client's bed slightly elevated. d. Place the client in the left lateral position.

d. The symptoms experienced by the client indicate supine hypotension syndrome. When the pregnant woman assumes a supine position, the expanding uterus exerts pressure on the inferior vena. The nurse should place the client in the left lateral position to correct this syndrome and optimize cardiac output and uterine perfusion. Elevating the client's legs, placing the client in an orthopneic position, or keeping the head of the bed elevated will not help alleviate the client's condition.

During a prenatal visit, the nurse inspects the skin of the client's abdomen. Which would the nurse identify as an abnormal finding? a. bruising b. linea nigra c. striae d. darkening of the umbilicus

a. Bruising would not be a normal finding. Evidence of bruising might suggest domestic violence. Linea nigra, striae, and darkening of the umbilicus are normal findings.

A nurse is instructing a client on birth control methods. The client asks about the cervical mucus method. When should the nurse tell the client she is fertile in relation to her mucus? a. when it is thin, watery, and copious b. when it is thick c. Cervical mucus is not a reliable indicator. d. when it does not stretch

a. The cervical mucus method relies on the changes that occur naturally with ovulation. Before ovulation, cervical mucus is thick and does not stretch when pulled. With ovulation, the mucus becomes thin, copious, watery, transparent, and stretchy.

The nurse is assessing a client who believes she is pregnant. The nurse points out a more definitive assessment is necessary due to which sign being considered a probable sign of pregnancy? a. Fatigue b. Amenorrhea c. Positive home pregnancy test d. Nausea and vomiting

c. A urine pregnancy test is considered a probable sign of pregnancy as the hCG may be from another source other than pregnancy. Fatigue, amenorrhea, and vomiting are presumptive or possible signs of pregnancy and can also have other causes.

Which question would be most important for a nurse to ask a G2P1 client who has a child with sickle cell anemia? a. "When was your last exacerbation?" b. "Have you done well to control your stress?" c. "Do you know the sex of your baby?" d. "Did you and your partner get genetic testing?"

d. The nurse should assess whether the client has had genetic testing completed. If not, the client should be referred for testing.

A client in the first trimester reports having nausea and vomiting, especially in the morning. Which instruction would be most appropriate to help prevent or reduce the client's compliant? a. Drink plenty of fluids at bedtime. b. Avoid foods such as cheese. c. Avoid eating spicy food. d. Eat dry crackers or toast before rising.

d. The nurse should recommend the client eat dry crackers or toast before rising to prevent nausea and vomiting in the morning. Drinking plenty of fluids at bedtime could cause nocturia. Foods such as cheese should be avoided to prevent constipation. Spicy foods could cause heartburn.

A client is attempting to become pregnant and is using varied methods to identify ovulation. What assessment finding would most clearly suggest that the client is ovulating? a. The client has clear, thin cervical mucus b. Cervical mucus is absent c. The client menstruated eight days ago d. The client's progesterone levels are peaking

a. Cervical mucus is clear and thin near ovulation. Ovulation typically occurs near day 14 of the menstrual cycle. Progesterone levels are low near the time of ovulation.

A pregnant client is scheduled to undergo chorionic villus sampling (CVS) to rule out any birth defects. Ideally, when should this testing be completed? a. 10 to 12 weeks' gestation b. 7 to 9 weeks' gestation c. 5 to 6 weeks' gestation d. 4 to 5 weeks' gestation

a. Chorionic villus sampling (CVS) is typically performed between 10 to 12 weeks' gestation. Sometimes it may be offered up to 14 weeks. The test is not conducted before 10 weeks' gestation.

A nurse is providing education to a client who is 8 weeks' pregnant. The client stated she does not like milk. What is a source of calcium that the nurse can recommend to the client? a. dark, leafy green vegetables b. deep red or orange vegetables c. white bread and rice d. meat, poultry, and fish

a. Dark leafy green vegetables are a source of calcium. Red and orange vegetables contain a variety of vitamins, bread and rice contain carbohydrates, and meat and fish contain protein, but none of these foods are a good source of calcium.

The nurse in the sexual health clinic is obtaining a health history of a client who suffers form heroin use disorder. The client reports chronic flulike symptoms accompanied by pruritis, fatigue, anorexia, and constant upper right quadrant pain. Which sexually transmitted infection would the nurse suspect? a. hepatitis A b. herpes simplex c. trichomoniasis d. syphilis

a. Hepatitis A produces flulike symptoms with malaise, skin rashes, fatigue, anorexia, nausea, pruritus, fever, and upper right quadrant pain. Symptoms of hepatitis B are similar to those of hepatitis A, but with less fever and skin rash involvement. Syphilis, herpes simplex, and trichomoniasis do not present with these types of symptoms.

Which statement best describes the pattern of maternal mortality since World War II? a. It has steadily decreased. b. It has remained constant. c. It has steadily increased. d. After decreasing until the 1960s, it has increased steadily.

a. Improved prenatal care and early ambulation after birth are factors that have decreased maternal death rates following birth over the past 70 years.

In 2007 the World Factbook published statistics that showed the United States still lagged behind other industrialized nations in the incidence of infant mortality. What is one reason that the United States has a higher infant mortality rate than other countries? a. Low birth weight b. Early prenatal care c. Low adolescent delivery rates d. Home deliveries

a. Many factors may be associated with high infant mortality rates and poor health. Low birth weight and late or nonexistent prenatal care are the main factors in the poor rankings in infant mortality.

A nurse is working in an ambulatory health care clinic located in a poor neighborhood. Which nursing intervention would provide the greatest benefit for the women and their children seen at this clinic? a. educating the parents about preventive care b. cautioning the parents about home safety issues c. screening for diabetes and malnutrition d. promoting healthy sleep and rest habits

a. Preventative care is consistent with a health promotion approach that aims to prevent illnesses and health disorders before they develop. Home safety is one example of health promotion and preventative care but there are many others. Screening for disease and promoting sleep are likely to be beneficial, but a general focus on preventative care is likely to have the greatest benefit.

The nurse is teaching about an iron supplement that the client is going to take every day. The nurse teaches the client to take the iron supplement with which type of fluid? a. citrus juice b. ice water c. low-fat milk d. hot tea

a. The citric acid in juice enhances absorption of iron in the GI tract. Ice water and tea do not enhance iron absorption, and milk can inhibit iron absorption.

A 23-year-old female has come to the clinic for her first prenatal visit. After the examination reveals no concerns and potential low-risk pregnancy, the nurse discusses nutritional needs for her and her growing baby. As per the Institute of Medicine, the nurse suggests the client take which amount of ferrous iron daily? a. 27 mg b. 20 mg c. 10 mg d. 40 mg

a. The dietary reference intakes as per the Institute of Medicine are for 27 mg of ferrous iron and 400 to 800 mcg of folic acid per day. Women with a previous history of a fetus with a neural tube defect are often prescribed a higher dose.

A nurse is asked to teach a woman to take her basal body temperature daily to assess the time of ovulation. She can detect her day of ovulation, following ovulation, because her temperature will: a. increase a degree. b. decrease a degree. c. fluctuate a degree daily. d. no longer reflect basal body temperature.

a. The effect of progesterone, released with ovulation, is to increase body temperature.

In teaching a fertility class, what is the most appropriate method for the nurse to explain self-ovulation monitoring that promotes conception? a. 14 days before the next menses is expected b. At the midpoint between menses cycles c. Immediately following the menses d. 14 days after the beginning of the last menses

a. The nurse should explain the ovulation occurs approximately 14 days before the next menses and that this is the best time for intercourse to promote conception.

A client learns she is 6 weeks' pregnant and asks the nurse for the name of the local abortion clinic. What is the most appropriate response by the nurse? a. Provide the client with the requested information. b. Tell the client that abortion (elective termination of pregnancy) results in death of the fetus. c. Call the hospital chaplain to talk to the client. d. Ask the health care provider to discuss the issue with the client.

a. The nurse should provide information to the client regardless of his or her own personal views.

A nurse is teaching a group of women at the local health clinic about reproduction. After describing the various internal structures involved, the nurse determines that the teaching was successful when the group identifies which structure as connecting the external structures with the cervix? a. vagina b. uterus c. rugae d. fundus

a. The vagina is a canal that connects the external genitals to the cervix. The uterus is an inverted, pear-shaped muscular organ at the top of the vagina and is subdivided into the convex portion above the uterine tubes, called the fundus, the central portion or corpus between the fundus and the cervix, and the cervix, or neck which opens into the vagina. Rugae are transverse folds in the vagina that allow it to dilate during labor and birth.

A male client appears in the walk-in clinic and requests treatment for trichomoniasis as his girlfriend was recently diagnosed with it. What medication would the health care provider most likely prescribe? a. metronidazole b. acyclovir c. clotrimazole d. penicillin G

a. Trichomoniasis is a common vaginal infection with the therapeutic management of metronidazole or tinidazole for both partners. Trichomoniasis is a common, curable sexually transmitted infection (STI) caused by a parasitic protozoa called Trichomonas.

The nurse understands that the maternal uterus should be at what location at 20 weeks' gestation? a. at the level of the symphysis pubis b. at the level of the umbilicus c. at the level near the bottom of the sternum d. three finger-breadths above the umbilicus

b. By 20 weeks' gestation, the uterus is at about the level of the umbilicus; by 36 weeks, it nears the bottom of the sternum.

A client is being prepared for intrauterine (artificial) insemination. Which finding is the most suggestive to determine if the client is ovulating? a. slight weight gain b. change in the cervical mucus c. abdominal cramps d. fall in body temperature

b. Change in the appearance and consistency of cervical mucus is an indication of ovulation. Slight weight gain and abdominal cramps are not signs used to determine ovulation. At the time of ovulation, body temperature is slightly elevated.

Which hormone is secreted from the hypothalamus in a pulsatile manner throughout the reproductive cycle? a. follicle-stimulating hormone (FSH) b. gonadotropin-releasing hormone (GnRH) c. luteinizing hormone (LH) d. estrogen

b. GnRH is secreted from the hypothalamus in a pulsatile manner throughout the reproductive cycle. It induces the release of FSH and LH to assist with ovulation, both of which are secreted by the anterior pituitary gland. Estrogen is secreted by the ovaries and is crucial for the development and maturation of the follicle.

A nursing student correctly identifies that a person's outward appearance or expression of genes is referred to as the: a. allele. b. phenotype. c. genotype. d. genome.

b. Phenotype refers to a person's outward appearance or the expression of genes. Alleles are two like genes. Genotype refers to his or her actual gene composition. Genome is the complete set of genes present in a person.

A group of nursing students are preparing a presentation for a health fair illustrating what happens to the body during pregnancy. Which structure will the students point out is influenced the most by hormones to prepare for conception? a. myometrium b. endometrium c. perimetrium d. fundus

b. The endometrium is the inner layer of the uterus; it builds with hormones during the month in potential preparation for pregnancy. The myometrium is the muscular layer. The perimetrium is a protective layer, and the fundus is the upper area of the uterus.

A client who is scheduled for hysteroscopy says to a nurse, "I thought I would be able to have children, but now I know that will never be possible." Which response would be most appropriate for the nurse to make? a. "Adoption is always an acceptable option for anyone." b. "I will call the health care provider to come visit with you." c. "The use of surrogates is more acceptable today." d. "Being childless is not as bad as you think it is."

b. The nurse needs to notify the health care provider because the client does not have a clear understanding of the outcomes of the procedure in which she is about to undergo.

A couple comes to the clinic asking questions regarding getting pregnant. The nurse explains that fertilization normally occurs in what structure of the woman's body? a. Uterine isthmus b. Fallopian tube c. Uterine corpus d. Ovarian surface

b. The ova mature and are released from the ovary. It then is attracted into the fallopian tube by the fimbriae where the sperm fertilizes it, usually in the ampulla portion of the tube.

The nurse is reviewing the functions of the ovaries, uterus, clitoris, and vagina with a group of nursing students. What would be the best response by a nursing student about the function of the uterus? a. It produces female gametes or ova and secretes female sex hormones. b. It receives the fertilized ovum and provides housing and nourishment for a fetus. c. It receives sperm, provides an exit for menstrual flow, and serves as the birth canal. d. It is a small erectile structure that responds to sexual stimulation.

b. The uterus's functions are to receive the fertilized ovum and provide housing and nourishment for a fetus. The ovaries' functions are to produce female gametes or ova and secrete female sex hormones. The vagina's functions are to receive sperm, provide an exit for menstrual flow, and serve as the birth canal. The clitoris is a small erectile structure that responds to sexual stimulation.

A pregnant woman comes to the clinic for a prenatal visit for her third pregnancy. She reveals she had a previous miscarriage at 12 weeks and her 3-year-old son was born at 32 weeks. How should the nurse document this woman's obstetric history? a. G3, T1, P0, A2, L1 b. G3, T0, P1, A1, L1 c. G2, T1, P2, A1, L2 d. G2, T0, P1, A1, L1

b. The woman's obstetric history would be documented as G3, T0, P1, A1, L1. G (gravida) = 3 (past and current pregnancy), T (term pregnancies) = 0, P (number of preterm pregnancies) = 1, A (number of pregnancies ending before 20 weeks viability to include miscarriage) = 1, and L (number of living children) = 1.

The nurse is caring for a child with Down syndrome (trisomy 21). This is an example of which type of inheritance? a. Mendelian recessive b. Mendelian dominant c. chromosome nondisjunction d. phase 2 atrophy

c. Down syndrome occurs when an ovum or sperm cell does not divide evenly, permitting an extra 21st chromosome to cross to a new cell.

The nurse has been working with a subfertile couple through the process of determining their best plan of action to conceive a child. Which action should the nurse prioritize to provide emotional support to this couple? a. Speak only with both partners present so they will feel more comfortable. b. Speak separately with each partner so both will feel more free to talk. c. Speak both separately and together with both partners. d. Only speak with the woman, who is the client.

c. Talking with both partners together may be advantageous, because this may make them feel more comfortable to discuss their problem. On the other hand it is important to spend time alone with each partner, in case there is something one person wants to discuss privately. Both partners are the client in this case.

Which hormone stimulates the release of the follicle-stimulating hormone (FSH) from both male and female glands? a. growth hormone-releasing hormone (GHRH) b. thyrotropin-releasing hormone (TRH) c. gonadotropin-releasing hormone (GnRH) d. prolactin-releasing hormone (PRH)

c. The male and female glands respond to LH and FSH, which are released from the anterior pituitary in response to stimulation from gonadotropin-releasing hormone that is released from the hypothalamus. GHRH stimulates the release of the growth hormone, which targets cell growth. THR stimulates the thyroid stimulating hormone, which targets the thyroid gland. PRH stimulates the release of prolactin, which is necessary for milk production.

A pregnant client in her second trimester informs the nurse that she needs to travel by air the following week. Which precaution should the nurse instruct the client to take during the flight? a. Wear a padded bra. b. Wear low-heeled shoes. c. Wear support hose. d. Wear cotton clothes.

c. The nurse should instruct the client to wear support hose while traveling by air. The nurse should also instruct the client to periodically exercise the legs and ankles, and walk in the aisles if possible. Wearing low-heeled shoes, cotton clothes, or a padded bra will have no effect on the client during the flight.

The nurse obtains a human chorionic gonadotropin (hCG) level from a woman who thinks that she is pregnant. Which result would the nurse identify as a positive pregnancy result? a. 8 mIU/mL (8 IU/L) b. 16 mIU/mL (16 IU/L) c. 24 mIU/mL (24 (IU/L) d. 32 mIU/mL (32 IU/L)

d. An hCG level lower than 5 mIU/mL (5 IU/l) is considered negative for pregnancy, and anything higher than 25 mIU/mL (25 IU/l) is considered positive for pregnancy.

Which description best explains the hysterosalpingogram procedure? a. insertion of an endoscope through the posterior fornix to visualize the reproductive organs b. instillation of carbon dioxide through the cervix into the uterus and fallopian tubes c. passage of an endoscope through a small abdominal incision to inspect the reproductive organs d. radiograph of the uterus and fallopian tubes following introduction of a radiopaque medium through the cervix

d. Hysterosalpingogram is a procedure to document the patency of the fallopian tubes through the use of a radiopaque medium.

The nurse educates a client who is confused about her ovarian cycle. Which client statement would best validate her understanding of the education? a. "I will ovulate every month on Day 21 of my cycle." b. "My menstrual cycles are controlled by progesterone production." c. "When I ovulate, there is a follicle on my uterus that forms showing that an ova was released." d. "Two hormones control my ovulation, follicle-stimulating hormone (FSH) and luteinizing hormone (LH)."

d. Ovulation is controlled by FSH and LH, with the follicle-stimulating hormone encompassing days 1 to 14 of a 28-day cycle and the luteinizing hormone controlling the luteal phase, which is days 15 to 28. The follicle forms only in the ovary, not the uterus. Ovulation should occur on Day 14 of a normal 28-day cycle. Both estrogen and progesterone are necessary to the menstrual cycle, not just progesterone.

A nurse is caring for a client who has just been told that her unborn baby carries a genetic disorder associated with significant health related issues. The client asks, "Should I kill the baby?" What is the most appropriate nursing diagnosis for this client? a. Fear related to birth of a baby b. Situational low self-esteem c. Deficient knowledge related to inheritance pattern d. Decisional conflict related to continuation of pregnancy

d. The client is unsure as to whether to continue the pregnancy or terminate it as evidenced by the question she asked the nurse.

At the first prenatal visit of all clients who come to the clinic appropriate blood screenings are obtained. The nurse realizes that a hemoglobin A1C above which level is concerning for diabetes and warrants further testing? a. 6.5% b. 6.0% c. 5.5% d. 5.0%

a. A hemoglobin A1C level of at least 6.5% is concerning for overt diabetes, and further testing should be conducted to ensure the client does not have diabetes. If glucose testing is not diagnostic of overt diabetes, the woman should be tested for gestational diabetes from 24 to 28 weeks' gestation with a 75-gm oral glucose tolerance test.

While interviewing a Chinese man about his son's health, the nurse notices that the man keeps his eyes focused on the floor. What is the best action by the nurse? a. Continue with the interview. b. Stop and ask the man to elaborate about his anxiety regarding his son's care. c. Ask the man if he would like to continue the interview at a different time. d. Ask the man if anything is wrong.

a. For some Chinese Americans, it is not culturally acceptable to make eye contact while conversing; this can be seen as a cultural norm for this man. Continuing with the interview is the best response so as not to make him feel uncomfortable.

What is the most common viral infection? a. human papillomavirus (HPV) b. gonorrhea c. chlamydia d. trichomoniasis

a. HPV infection is the most common viral infection. Millions of Americans are infected with HPV, many unaware that they carry the virus.

A clinic nurse explains to a client who is undergoing an infertility workup that the patency of her fallopian tubes will be checked. Which test is currently used to do this? a. hysterosalpingography b. computed tomography scan c. magnetic resonance imaging d. uterine biopsy

a. Hysterosalpingography is radiologic examination with radiopaque contrast of the fallopian tubes. It is widely used to assess the patency of the fallopian tubes in women who are subfertile.

The nurse is assisting a primigravida on calculating the due date of her baby using Naegele rule. The most important information provided by the mother is: a. the first day of the last menstrual period. b. the ovulation date between her periods. c. the date that intercourse occurred. d. the last day of her menstrual period.

a. Naegele rule is calculated using the first day of the last menstrual period. From there, 7 days are added and then 3 months are subtracted. The ovulation date, intercourse date, or last day of the menstrual period are not needed.

A nurse is providing a client with information on hormonal contraception. Which could the nurse use as an example of hormonal contraception? a. medroxyprogesterone b. mifepristone c. sildenafil d. tadalafil

a. The nurse should inform the client that medroxyprogesterone is a hormonal contraceptive. Sildenafil and tadalafil are medications designed to induce erection. Mifepristone causes abortion (elective termination of pregnancy). They do not act as hormonal contraceptives.

When teaching the client how to use a contraceptive sponge, the nurse must tell the client that leaving the sponge in place longer than 30 hours may lead to: a. toxic shock syndrome. b. pelvic inflammatory disorder. c. sexually transmitted infections. d. cervical inflammation.

a. The sponge provides protection for up to 12 hours but should not be left in place for more than 30 hours after insertion to avoid the risk of toxic shock syndrome.

A client prescribed oral contraceptive pills (OCPs) has presented for a routine visit. Which finding, if reported by the client upon assessment, should the nurse prioritize? a. abdominal pain b. small amount of breakthrough bleeding c. light menstrual flow d. cramping during menses

a. The warning signs to report for a client on OCPs are severe abdominal or chest pain, dyspnea, headache, weakness, numbness, blurred or double vision, speech disturbances, or severe leg pain and edema. Light bleeding, light flow, and cramping are all normal.

The nurse is assessing a 15-year-old female who reports extreme itching in the genital area, dysuria, and foul-smelling, yellow, foamy, vaginal discharge. What would most likely be responsible for these symptoms? a. trichomoniasis b. herpes simplex 2 c. human papillomavirus d. syphilis

a. Trichomoniasis symptoms are vulvar itching and a malodorous foamy yellow vaginal discharge.

The nurse, while teaching a group of pregnant women about the production of breast milk, describes colostrum and compares it with mature breast milk. Which characteristic would the nurse identify as associated with colostrum? a. high sugar content b. high mineral content c. low maternal antibody content d. low protein content

b. Colostrum is rich in maternal antibodies, especially IgA, which protects the newborn against enteric pathogens. Colostrum contains more minerals and protein but less sugar and fat than mature breast milk.

The nurse cares for a client recently diagnosed with syphilis who has no known drug allergies. Which medication will the nurse expect the health care provider to prescribe? a. miconazole b. benzathine penicillin G c. metronidazole d. doxycycline

b. Current guidelines recommend benzathine penicillin G injection (long-acting penicillin) as the preferred treatment for syphilis. If the client is allergic to penicillin, other medications (e.g., doxycycline, tetracycline, or ceftriaxone) may be used; however, data to support the use of these medications to treat syphilis are limited. Metronidazole is the treatment for trichomoniasis, and miconazole is the treatment for candidiasis.

The partner of a pregnant client in her first trimester asks the nurse about the client's behavior recently, stating that she is very moody, seems happy one moment and is crying the next and all she wants to talk about is herself. What response would correctly address these concerns? a. Her body is changing and she may be angry about it. b. Pregnant women often experience mood swings and self-centeredness but this is normal. c. Moodiness and irritability are not usual responses to pregnancy. d. What you are describing may be normal but we need to talk to her more in depth.

b. During the first trimester of pregnancy, the woman often has mood swings, bouts of irritability and is hypersensitive. The partner needs to know that these are all normal behaviors for a pregnant woman.

The nurse is teaching a pregnant client some nonpharmacologic ways to handle common situations encountered during pregnancy. The nurse determines the session is successful when the client correctly chooses which condition that can be minimized if she avoids drinking fluids with her meals? a. nosebleeds b. heartburn c. blood clots d. constipation

b. Filling the stomach with heavy food and fluid can cause overfill and place pressure on the stomach, increasing gastric reflux. Avoid excess fluids with meals and eat small frequent meals to avoid heartburn. Nosebleeds result from increased estrogen. Blood clots can result from sitting still for too long. Constipation can result from increased progesterone.

While in utero, a fetus swallows many substances that are deposited in the fetal intestinal system as meconium. What problem can arise from this occurrence? a. The fetus can become constipated following birth. b. If the fetus becomes stressed, the meconium is released into the amniotic fluid, placing the fetus at risk for pneumonia. c. Abdominal distension occurs and infection can set in. d. Meconium-stained fluids cause an increased risk of jaundice.

b. Infants develop meconium in their intestines; if they are stressed or hypoxic, the anal sphincter relaxes and meconium is passed into the amniotic fluid. This poses a danger to the fetus since they breathe in this fluid and swallow it. The meconium lines the lungs and respiratory passages, making it difficult for the infant to breathe once it is born.

A new mother voices concerns about breastfeeding her infant. The nurse would explain to the mother the two hormones that control lactation and letdown are: a. follicle-stimulating hormone and thyroid hormone. b. prolactin and oxytocin. c. estrogen and progesterone d. luteinizing hormone and hCG

b. Prolactin and oxytocin are both important hormones in regulation of breastfeeding. Prolactin helps in producing the breast milk and oxytocin stimulates letdown during breastfeeding. The other hormones do not play a role in breastfeeding or milk production.

A client's menstrual period is two weeks late. She has been feeling tired and has had episodes of nausea in the morning. What classification of pregnancy symptoms is this client experiencing? a. positive b. presumptive c. probable d. no classification

b. The most common presumptive sign of pregnancy is a missed menstrual period, or amenorrhea. Other presumptive signs include nausea, fatigue, swollen, tender breasts, and frequent urination.

When preparing a class for a group of pregnant women about nicotine use during pregnancy, the nurse describes the major risks associated with nicotine use, including: a. increased risk of spontaneous abortion (miscarriage). b. decreased birth weight in neonates. c. increased risk of stillbirth. d. increased risk of placental abruption (abruptio placentae).

b. The nurse should inform the client that children born of mothers who use nicotine will have a decreased birth weight. Spontaneous abortion (miscarriage) is associated with caffeine use. Increased risks of stillbirth and placental abruption (abruptio placentae) are associated with mothers addicted to cocaine.

A client is trying to have a baby and wants to know the best time to have intercourse to increase the chances of pregnancy. Which time for intercourse is ideal to help her chances of conceiving? a. a week after ovulation b. one or two days before ovulation c. any time after ovulation d. any time during the week before ovulation

b. To increase the chances of conceiving, the best time for intercourse is 1 or 2 days before ovulation. This ensures that the sperm meets the ovum at the right time. The average life of a sperm cell is 2 to 3 days, and the sperm cells will not be able to survive until ovulation if intercourse occurs a week before ovulation. The chances of conception are minimal for intercourse after ovulation.

A newborn is diagnosed with ophthalmia neonatorum. The nurse understands that this newborn was exposed to which infection? a. syphilis b. Candida albicans c. gonorrhea d. human immunodeficiency virus

c. Gonorrhea can be transmitted to the newborn in the form of ophthalmia neonatorum during birth by direct contact with gonococcal organisms in the cervix. The newborn would develop congenital syphilis if exposed in utero. Exposure to Candida would cause thrush in the newborn. Exposure to HIV during gestation could lead to the birth of an HIV-positive newborn.

A client is receiving ceftriaxone as treatment for gonorrhea. What would be most important for the nurse to emphasize to the client? a. Take the drug on an empty stomach. b. Use a sunscreen when outside. c. Avoid alcohol consumption. d. Report signs of an oral yeast infection.

c. If alcohol is ingested when taking ceftriaxone, the client can experience a disulfiram-like reaction. Therefore the nurse would need to emphasize avoiding alcohol consumption. Taking the drug on an empty stomach may be appropriate but not the most important consideration. Using a sunscreen would be appropriate if the client was receiving doxycycline or tetracycline. Reporting the appearance of an oral yeast infection would be appropriate for a client receiving tetracycline.

A client diagnosed with genital herpes asks the nurse about future sexual experiences. Which response by the nurse is appropriate? a. "If your partner has been exposed to the herpes virus in the past, then there is no significant risk of reinfecting your partner." b. "Future sexual intercourse should be avoided due to the high risk of transmitting the virus to others and causing yourself additional outbreaks." c. "If you plan to have future sexual relationships, you need to let your partner know of your history prior to intercourse." d. "As long as you wear a condom and have intercourse while no lesions are present, you will not spread the virus to your partner."

c. The client should be instructed to notify future partners before having intercourse with them. Herpes simplex virus transmission can occur both during and between recurrences. Transmission is more likely to occur during an outbreak but can also happen during times of no outbreak. The client does not have to avoid any future sexual experiences, because condom use decreases transmission. The client needs to be properly educated on how to prevent transmission to others and avoid contracting another sexually transmitted infection in the future. Sexual intercourse does not increase the client's chance of future outbreaks. Prior contact with the virus does not confer immunity.

The nursing instructor is preparing a class presentation covering the various hormones and their functions during pregnancy. The instructor determines the class is successful when the class correctly matches which function with hCG? a. provides rich blood supply to decidua b. maintains nutrient-rich decidua c. continues progesterone production by corpus luteum d. sustains life of placenta

c. The corpus luteum is responsible for producing progesterone until this function is assumed by the placenta. hCG is a fail-safe mechanism to prolong the life of the corpus luteum and ensure progesterone production. Estrogen is responsible for providing a rich blood supply to the decidua. Progesterone helps maintain a nutrient-rich decidua.

As part of caring for childbearing families, the nurse must be able to educate families regarding which topic? a. Proper selection of a physician b. The best hospital for delivery of their babies c. Knowledge of reproductive anatomy and physiology d. The cost of obstetrical and neonatal care

c. The obstetrical/gynecological nurse must be well versed in pregnancy and childbirth, as well as an understanding of clients of childbearing age. As a gynecological nurse, he or she must also have a good grasp of reproductive anatomy and physiology and the menstrual cycle. Helping select a physician, a hospital, or providing input on cost factors is not the nurse's job.

The client is 32 weeks' pregnant and has been referred for a biophysical profile (BPP) after a nonreassuring nonstress test (NST). Which statement made by the client indicates that the nurse's explanation of the procedure was effective? a. The BPP is a diagnostic procedure whereby a needle is inserted into the amniotic sac to obtain fluid. b. The BPP is a blood test to detect placental problems. c. The BPP is a screening for neural tube defects. d. The BPP is an ultrasound that measures breathing, body movement, tone, and amniotic fluid volume.

d. A biophysical profile uses a combination of factors to determine fetal well-being based upon five fetal biophysical variables. An NST is done to measure FHR acceleration. Then an ultrasound is done to measure breathing, body movements, tone, and amniotic fluid volume. Each variable receives a score from 0 to 2 for a maximum score of 10. A score of 6 or less indicates altered fetal well-being and indicates a need for further assessment. A needle is not involved with the BPP. The BPP does not detect placental problems, and the BPP is not a screening for neural tube defects.

A nursing student is explaining to a newly pregnant woman what happens during each stage of fetal development. At which stage does the nurse inform the woman that the lungs are fully shaped? a. end of 4 weeks b. end of 8 weeks c. end of 12 weeks d. end of 16 weeks

d. At the end of 16 weeks, the lungs are fully shaped, fetus swallows amniotic fluid, skeletal structure is identifiable, downy lanugo hair is present on the body, and sex can be determined using ultrasound.

Copious amounts of frothy, greenish vaginal discharge would be a symptom of which infection? a. candidiasis b. Gardnerella vaginalis vaginitis c. gonorrhea d. trichomoniasis

d. The discharge associated with infection caused by Trichomonas organisms is homogenous, greenish gray, watery, and frothy or purulent. The discharge associated with candidiasis is thick, white, and resembles cottage cheese in appearance. The discharge associated with infection due to G. vaginalis is thin and grayish white, with a marked fishy odor. With gonorrhea, vaginal discharge is purulent when present but, in many women, gonorrhea produces no symptoms.

Fetal circulation differs from the circulatory path of the newborn infant. In utero the fetus has a hole connecting the right and left atria of the heart. This allows oxygenated blood to quickly pass to the major organs of the body. What is this hole called? a. Foramen venosus b. Foramen magnum c. Foramen arteriosus d. Foramen ovale

d. The foramen ovale is a hole that connects the right and left atria so the majority of oxygenated blood can quickly pass into the left side of the fetal heart, go to the brain, and move to the rest of the fetal body.

A couple is considering vasectomy as a contraception option. However, the husband is nervous about how such a procedure would affect his sexual functioning. Which information should the nurse mention to the man? a. The man will still have full erection capacity. b. The man will no longer produce testosterone. c. The man will continue to ejaculate sperm. d. The man will no longer ejaculate seminal fluid.

a. Some men resist the concept of vasectomy because they are not sufficiently aware of their anatomy to know exactly what the procedure will involve. They can be assured vasectomy does not interfere with the production of sperm; the testes continue to produce sperm as always, but the sperm simply do not pass beyond the plugged vas deferens and are absorbed at that point. The man will still have full erection capacity and continue to produce testosterone. Because he also continues to form seminal fluid, he will ejaculate seminal fluid; it will just not contain sperm.

There are two major functions of the reproductive system, continuation of the species and sexual pleasure. In the male, the reproductive system has a third function. What is this third function? a. urinary elimination b. elimination of solid wastes c. production of sex hormones d. stimulation of sexual maturation

a. Some of the structures in the reproductive tract serve dual purposes. Most often these alternate functions have to do with urinary elimination because the urinary and reproductive systems are closely connected.

A 22-year-old client comes to the walk-in clinic complaining of fatigue, breast heaviness and extreme tenderness, and a clear vaginal discharge. What question would the nurse ask this client? a. Have you been sexually active in the past 2 months? b. Do you have a family history of breast cancer? c. Do you have vaginal itching? d. Are you taking oral contraceptives?

a. The client is presenting with presumptive or subjective symptoms of pregnancy. Given her symptoms and age, asking about sexual activity is the most appropriate question. Whether she is taking an oral contraceptive will not assist in identifying the cause of her symptoms. If she has vaginal itching, the underlying cause of her symptoms needs to be identified before treatment can be prescribed. Asking about family history is part of a comprehensive health history, but is not the priority based on the client's presentation.

A client who has one child with a genetic disorder tells the nurse, "I told my husband I was ready to have another baby and now he does not want to be intimate with me." What is the most appropriate nursing diagnosis for this client's husband? a. Altered sexuality pattern related to fear b. Ineffective coping related to personal issues c. Hopelessness related to increased tension d. Risk for loneliness related to lack of communication

a. The client's husband may be not engaging in intercourse because he is afraid of conceiving a second child with a genetic disorder. The nurse should identify resources for the client that allow for increased communication and education for the couple.

The diagonal conjugate of a pregnant woman's pelvis is measured. Which measurement would the nurse interpret as presenting a potential problem? a. 12.0 cm b. 12.5 cm c. 13.0 cm d. 13.5 cm

a. The diagonal conjugate, usually 12.5 cm or greater, indicates the anteroposterior diameter of the pelvic inlet. The diagonal conjugate is the most useful measurement for estimating pelvic size because a misfit with the fetal head occurs if it is too small.

A nurse is conducting an in-service education program about the female reproductive cycle for a group of nurses working at a women's health clinic. When describing the phases of the endometrial cycle, the nurse would explain that the proliferative phase of the endometrial cycle corresponds to which phase in the ovarian cycle? a. follicular b. secretory c. ischemic d. menstrual

a. The endometrial cycle consists of four phases: proliferative, secretory, ischemic, and menstrual. The proliferative phase of the endometrial cycle corresponds to the follicular phase of the ovarian cycle.

A young woman says she needs a temporary contraceptive but has a latex allergy. She mentions that she has had a papillomavirus infection. Also, she says she is terrible about remembering to take pills. Which method should the nurse recommend? a. transdermal contraception b. sterilization c. cervical cap d. diaphragm

a. The fact that this woman has a latex allergy rules out the cervical cap and diaphragm. Moreover, the diaphragm is contraindicated in her case due to her papillomavirus infection. The best choice for her is transdermal contraception, which involves wearing a patch for a week at a time and does not require taking pills daily.

The process of oogenesis begins before birth but is not complete until the childbearing years. At what point does the second meiotic division occur in this process? a. at the time of fertilization by the sperm b. at the time of division of the secondary oocyte and the first polar body c. when the ovum attaches to the uterine wall d. as the ovum is expelled from the corpus luteum

a. The first meiotic division occurs before ovulation, forming a secondary oocyte and a polar body. The polar body disintegrates due to lack of cytoplasm. This leads to the second meiotic division as the sperm fertilizes the oocyte. The second division occurs long before attachment to the uterine wall and after the division, producing the polar body as the ovum leaves the ovary.

The nurse is teaching a group of women about the hormone that makes the ovum within the ovaries mature each month. Which hormone is responsible for stimulating the ovum? a. Follicle-stimulating hormone b. Luteinizing hormone c. Estrogen d. Progesterone

a. The follicle-stimulating hormone (FSH) is the messenger from the pituitary gland to stimulate the ovum to mature. Luteinizing hormone is the hormone that is released from the pituitary gland once the follicle containing the ovum is mature, but it has no role in maturing the ovum. Estrogen and progesterone are integral parts of the ovulatory cycle but do not mature ovum.

Which medical pair is the highest concern if reported during a pregnant client's medical history? a. Heart disease and diabetes b. Asthma and environmental allergies c. Irritable bowel syndrome and nausea d. Sinus congestion and bronchitis

a. The highest concern is heart disease and diabetes. Due to the increase in circulating blood volume, the heart has significantly more workload. Diabetes must be closely regulated, as a high glucose can have an impact on the status of the fetus. All of the other options are important to discuss with the health care provider but not of highest concern.

When developing a program for STI prevention, which action would need to be done first? a. educating on how to promote sexual health b. getting individuals to change their behaviors c. increasing the availability of resources d. interfering with the mode of transmission

a. The key to successful treatment and prevention of STIs is education to promote sexual health. Behavior changes, increasing the availability of resources, and interfering with modes of transmission are important, but all of these require education.

A nurse supervisor observes a nurse massage a client's injection site after giving a dose of depot medroxyprogesterone acetate (Depo-Provera). What is the priority response by the nurse supervisor? a. Remind the nurse that this injection should absorb slowly. b. Thank the nurse for excellent client care. c. Write up an incident report for the medication error. d. Call the health care provider immediately.

a. The medication should absorb slowly from the muscle, so the nurse supervisor should remind the nurse not to rub the injection site following administration.

Which medication is the most effective treatment for trichomoniasis? a. metronidazole b. penicillin G benzathine c. doxycycline d. azithromycin

a. The most effective treatment for trichomoniasis is metronidazole and tinidazole. Penicillin G benzathine is used for syphilis. Doxycycline and azithromycin are used in the treatment of chlamydia.

The nurse is conducting a well-visit physical assessment on a 29-year-old female. The nurse determines the client is probably ovulating based on which condition of the cervical mucus? a. Thin, slippery, and stretchy b. Cloudy, thick, and watery c. Clear and of large quantity d. Thin and red-tinged

a. The mucus at the time of ovulation is thin, slippery, and stretchy to allow for the passage of sperm into the uterus. This is called spinnbarkeit fiber. If the mucus is too thick, it will inhibit fertilization. Before ovulation, the mucus is thick and does not stretch easily.

It is important that couples who wish to practice natural forms of birth control understand that the mucus-secreting glands of the cervix can store live sperm for up to: a. 48 hours. b. 36 hours. c. 24 hours. d. 12 hours.

a. The mucus-secreting glands of the cervix are capable of storing live sperm for 48 to 72 hours (2 to 3 days), enabling pregnancy to occur after intercourse has occurred 1 or 2 days before ovulation.

The nurse is teaching a group of pregnant females about changes that occur in the uterus during pregnancy. The nurse identifies which area as experiencing marked hypertrophy? a. upper myometrium b. lower perimetrium c. inner endometrium d. cervix

a. The myometrium makes up the major portion of the uterus and is composed of smooth muscle linked by connective tissue with numerous elastic fibers. During pregnancy, the upper myometrium undergoes marked hypertrophy, but there is limited change in the cervical muscle content. Neither the endometrium (the innermost layer) nor the perimetrium (the outer serosal layer that covers the body of the uterus) undergo such a change.

The school nurse is preparing a health education session on the topic of birth control for a college group. Which method should the nurse emphasize for this group? a. barrier b. hormonal c. rhythm d. transdermal

a. The nurse should emphasize a barrier method of birth control such as a condom. The condom will help not only with birth control but with the spread of STIs, which is common in this age group. The other methods, such as hormonal, rhythm, and transdermal, will only prevent pregnancy and are more commonly utilized in long-term monogamous relationships.

A client who delivered her baby 3 months ago is seen in the clinic and tells the nurse that she and her husband have yet to resume a sexual relationship. The nurse notes that no contraception is currently being used. What is the most appropriate nursing diagnosis for this client? a. Altered sexual pattern related to fear of pregnancy b. Spiritual distress related to partner's preference for contraception c. Decisional conflict related to unintended pregnancy d. Powerlessness related to failure of chosen contraceptive

a. The nurse should recognize that this exceeds the recommended postpartum abstinence period and might contribute to a fear of a repeat pregnancy.

A nurse is conducting a sexual health education class about the structures and events of semen production. Which component will the students identify as responsible for semen production if the teaching has been successful? a. accessory gland alkaline fluids b. sperm c. testes d. vas deferens e. seminiferous tubules

a, b. The alkaline fluids from the accessory glands and sperm combine to form a thick, whitish secretion termed semen or seminal fluid. The testes and seminiferous tubules within the testes are where sperm are produced. The vas deferens is the muscular tube in which sperm begin their journey out of the man's body. It connects the epididymis with the ejaculatory duct.

A nurse is reading a journal article about the use of real-time ultrasonography, which allows the health care provider to obtain information about the fetus. The nurse would expect the article to describe which type of information? a. biophysical profile b. chromosomal abnormalities c. the effectiveness of neural tube defect treatment d. the size and shape of placenta

a. A biophysical profile uses real-time ultrasound to allow assessment of various parameters of fetal well-being. This may include fetal movements, fetal tone, and fetal breathing, as well as assessment of amniotic fluid volume with or without assessment of fetal heart rate. Chromosomal abnormalities are detected via amniocentesis. Neural tube defect treatment is not evaluated via biophysical profile, and although the placenta may be observed, it is not the focus of this procedure.

A nurse is presenting a program for a local women's group about STIs. When describing the information, the nurse would identify which infection as the most common cause of vaginal discharge? a. candidiasis b. syphilis c. gonorrhea d. chlamydia

a. Although vaginal discharge can occur with any STI, genital/vulvovaginal candidiasis is one of the most common causes of vaginal discharge. It is also referred to as yeast, monilial, and a fungal infection. It is not considered an STI because candida is a normal constituent in the vagina and becomes pathogenic only when the vaginal environment becomes altered.

After teaching a group of students about sexually transmitted infections (STIs), the instructor determines that additional teaching is necessary when the students identify which STI as curable with treatment? a. genital herpes b. syphilis c. gonorrhea d. chlamydia

a. Besides AIDS, the five most common STIs are chlamydia, gonorrhea, syphilis, genital herpes, and genital warts. Of these, chlamydia, gonorrhea, and syphilis are easily cured with early and adequate treatment. Genital herpes recurs.

What is the major concern for a lactose intolerant woman who is pregnant? a. calcium deficiency b. dangerous symptom of abdominal cramping c. nausea and vomiting d. vitamin D deficiency

a. Calcium deficiency is a major concern for the pregnant woman who is lactose intolerant. There are several ways to address this concern. Some lactose-intolerant individuals are able to tolerate cooked forms of milk, such as pudding or custard. Cultured or fermented dairy products, such as buttermilk, yogurt, and some cheeses may also be tolerated. A chewable lactase tablet may be taken with milk. Lactase-treated milk is available in most supermarkets and may be helpful. Other options are to drink calcium-enriched orange juice or soy milk or to take a calcium supplement. If the woman is infrequently exposed to sunlight, she will need a vitamin D supplement.

A pregnant client is concerned she may develop preeclampsia, so she has stopped adding any salt to her food and is now questioning the nurse about avoiding prepared foods. The nurse should point out some salt is very beneficial and can help prevent which negative outcome for her baby? a. Congenital hypothyroidism b. Low birth weight c. Neural tube defects d. Night blindness

a. Iodized sodium is needed by the body for normal thyroid function. Women with severe iodine deficiencies deliver infants with congenital hypothyroidism. Low birth weight is related to smoking and alcohol. Neural tube defects are caused by low folic acid levels. When vitamin A levels are too low, night blindness may occur.

A pregnant woman tells the nurse she often has allergic responses to drugs. She is concerned that she will be allergic to her fetus or her body will reject the pregnancy. The nurse's reply would be based on which statement? a. Immunologic activity is decreased during pregnancy. b. The level of aldosterone during pregnancy reduces production of IgG antibodies. c. The decreased corticosteroid activity during pregnancy ensures this will not happen. d. The kidneys release a hormone during pregnancy to prevent this from happening.

a. It is unproven why women do not reject fetal (foreign) tissue, but a substance secreted by the placenta is thought to decrease the usual immunologic response and prevent this from happening.

The client is interested in using an injectable contraceptive that works by suppressing pituitary secretions. The nurse provides the client with literature and discusses which contraceptive with her? a. medroxyprogesterone acetate b. etonogestrel c. norelgestromin/ethinyl estradiol d. levonorgestrel/ethinyl estradiol

a. Medroxyprogesterone acetate is an injectable form of a progesterone-only contraceptive that is given every 12 weeks. It works by suppressing ovulation and the production of FSH and LH by the pituitary gland. Etonogestrel is a single-rod progestin implantable form contraceptive. Norelgestromin/ethinyl estradiol is a patch-type contraceptive, and levonorgestrel/ethinyl estradiol is an oral contraceptive that uses a 365-day combination dosing.

A nurse is providing prenatal care to a pregnant woman. Understanding a major component of this care, the nurse would conduct a risk assessment for: a. genetic conditions and disorders. b. infant nutritional needs. c. family dynamics. d. cultural differences.

a. Nurses at all levels should be participating in risk assessment for genetic conditions and disorders, explaining genetic risk and genetic testing, and supporting informed health decisions and opportunities for early intervention.

A 41-year-old pregnant woman and her husband are anxiously awaiting the results of various blood tests to evaluate the fetus for potential Down syndrome, neural tube defects, and spina bifida. Client education should include which information? a. Further testing will be required to confirm any diagnosis. b. The blood tests are definitive. c. Treatment can be started once the test results are back. d. A second set of screening tests can be obtained to confirm results.

a. Nursing management related to marker screening tests consists primarily of providing education about the tests. Remind the couple that a definitive diagnosis is not made without further tests such as an amniocentesis. The blood tests are not definitive but only strongly suggest the possibility of a defect. For some conditions there are no treatments. The couple may request a second set, but the health care provider will probably suggest proceeding with the more definitive methods to confirm the diagnosis.

A woman at 15 weeks' gestation is about to undergo amniocentesis. Which nursing intervention should be made first? a. Obtain a signed consent form. b. Have the client void. c. Place the client in supine position. d. Observe the fetal heart rate monitor.

a. Nursing responsibilities for assessment procedures include seeing a signed consent form has been obtained as needed (necessary if the procedure poses any risk to the mother or fetus that would not otherwise be present, as is the case with amniocentesis). All of the answers are nursing interventions that should be made before or during amniocentesis, but having the client sign a consent form should be completed before the others.

The nurse is teaching a pregnant woman about breastfeeding. The nurse determines that the teaching was successful when the woman identifies which hormone as being released when the newborn sucks at the breast? a. oxytocin b. follicle-stimulating hormone c. antidiuretic hormone d. cortisol

a. Oxytocin is responsible for milk ejection during breastfeeding. Its secretion is stimulated by stimulation of the breasts via sucking or touching. Secretion of follicle-stimulating hormone is inhibited during pregnancy. The secretion of antidiuretic hormone has no effect on breastfeeding. Cortisol secretion regulates carbohydrate and protein metabolism and is helpful in times of stress.

Pelvic infection is most commonly caused by: a. sexual transmission. b. induced abortion (medical abortion). c. hysteroscopy. d. insertion of intrauterine device.

a. Pelvic infection is most commonly caused by sexual transmission but can also occur with invasive procedures such as endometrial biopsy, induced abortion (medical abortion), hysteroscopy, or insertion of an intrauterine device.

A client at 38 weeks' gestation is diagnosed with placental insufficiency. Which prescription from the health care provider will the nurse anticipate? a. Prepare the client for an induction of labor. b. Discharge the client home with daily nonstress testing. c. Assess the client's blood pressure every 2 hours. d. Administer one dose of betamethasone.

a. Placental insufficiency is a serious complication where the placenta no longer works properly to provide nutrition and oxygen to the fetus, nor remove waste products from the fetus. Because this client's fetus is at full term, the nurse would anticipate an induction of labor or a cesarean birth. The client is not stable enough to be sent home for monitoring. Hypertension can be a cause of placental insufficiency; however, at this point in the pregnancy, birth is the best option. Betamethasone is a steroid given to clients to hasten preterm fetal lung development. This client is at term and does not need betamethasone.

A woman is taking vaginal progesterone suppositories during her first trimester because her body does not produce enough of it naturally. She asks the nurse what function this hormone has in her pregnancy. What should the nurse explain is the primary function of progesterone? a. maintains the endometrial lining of the uterus during pregnancy b. ensures the corpus luteum of the ovary continues to produce estrogen c. contributes to mammary gland development d. regulates maternal glucose, protein, and fat levels

a. Progesterone is necessary to maintain the endometrial lining of the uterus during pregnancy. It is human chorionic gonadotropin (hCG) that acts to ensure the corpus luteum of the ovary continues to produce estrogen and progesterone. Estrogen contributes to mammary gland development, and human placental lactogen regulates maternal glucose, protein, and fat levels.

The nurse in a primary care clinic and a client have come to see the primary care provider because the client is getting married and wants to have a prenuptial gynecologic examination. After the exam, the nurse asks about the woman's plan to have a family. The woman responds, "Why do I have to plan for a family?" What would be the nurse's best response? a. Families are always best when pregnancies are not planned. b. The couple can control the time between births when they plan their family. c. An unwanted pregnancy can always be aborted. d. The couple can decide on a method of birth control.

b. Women and couples can avoid unwanted pregnancies, bring about wanted births, and control the intervals between births with family planning.

During a health education session, a teenage client asks the nurse when she should have her first "Pap test." How should the nurse reply? a. "It all depends on your family history of cancer." b. "When you first start having sex." c. "As soon as possible and every year thereafter." d. "When you turn 21 years old."

d. Current guidelines recommend that the woman obtain the initial Papanicolaou test at 21 years of age, regardless of when she first has sexual intercourse.

Which finding from a woman's initial prenatal assessment would be considered a possible complication of pregnancy that requires reporting to a primary care provider for management? a. episodes of double vision b. increased lumbar curvature c. nasal congestion and swollen nasal membranes d. palpitations when lying on her back

a. Difficulty with vision can occur from cerebral edema or is a symptom of hypertension of pregnancy.

A nurse is assessing a pregnant client. The nurse understands that hormonal changes occur during pregnancy. Which hormones would the nurse most likely identify as being inhibited during the pregnancy? a. FSH and LH b. FSH and T4 c. T4 and GH d. LH and MSH

a. During pregnancy, FSH and LH are both inhibited as there is no need to develop a follicle and release an ovum. There is an increase in the secretion of T4 and MSH. There is a decrease in the production of GH and MSH but not an inhibition.

A nurse is counseling a couple who have a 5-year-old daughter with Down syndrome. The nurse recognizes that their daughter's genome is represented by which chromosome combination? a. 47XX21+ b. 47XY21+ c. 46XX d. 46XX5p-

a. In Down syndrome, the person has an extra chromosome 21, so this is abbreviated as 47XX21+ (for a female) or 47XY21+ (for a male). 46XX is a normal genome for a female. The abbreviation 46XX5p- is the abbreviation for a female with 46 total chromosomes but with the short arm of chromosome 5 missing (Cri-du-chat syndrome).

Which statement best explains the action of the lactational amenorrhea method (LAM) of fertility control? a. LAM causes suppression of the ovulation process. b. LAM prevents introduction of sperm into the vagina. c. LAM prevents fertilization between the egg and sperm. d. LAM thickens the cervical mucus and blocks sperm.

a. LAM prevents pregnancy by suppressing the ovulation process. Withdrawal and barrier methods of contraception prevent the introduction of sperm into the vagina. The copper IUD, combined hormonal methods, and oral contraceptive pills (OCPs) prevent fertilization between the egg and the sperm. Levonorgestrel system thickens the cervical mucus and blocks sperm.

The nurse is meeting with a 36-year-old client who wishes to begin using contraceptives. The client reports being in a long-term, monogamous relationship, runs 2 miles per day, and smokes a pack of cigarettes each day. Which method will the nurse be least likely to suggest to the client? a. oral contraceptive pills (OCPs) b. condoms c. coitus interruptus d. spermicides

a. Oral contraceptive pills (OCPs) are contraindicated for women who smoke; these women would be at a higher risk for blood clots. Condoms, coitus interruptus, and spermicides can be used at no risk.

A nurse who has been caring for a pregnant client understands that the client has pica and has been regularly consuming soil. For which condition should the nurse monitor the client? a. iron-deficiency anemia b. constipation c. tooth fracture d. inefficient protein metabolism

a. Pica is characterized by a craving for substances that have no nutritional value. Consumption of these substances can be dangerous to the client and her developing fetus. The nurse should monitor the client for iron-deficiency anemia as a manifestation of the client's compulsion to consume soil. Consumption of ice due to pica is likely to lead to tooth fractures. The nurse should monitor for inefficient protein metabolism if the client has been consuming laundry starch as a result of pica. The nurse should monitor for constipation in the client if she has been consuming clay.

The nurse is preparing to teach a drug education class at a local elementary school. The nurse is focused on providing which type of care to the community? a. Primary prevention b. Secondary prevention c. Tertiary prevention d. Preventive care

a. Primary prevention is promotion of healthy activities and includes education concerning safety, diet, rest, exercise, and disease prevention. Secondary prevention focuses on health screening activities that aid in early diagnosis and encourage prompt treatment. Tertiary prevention focuses on rehabilitation and instruction on ways to prevent further injury or illness. "Preventive care" is not considered a specific category but is a general function that encompasses all three levels.

A nurse is conducting a presentation at a community health center about congenital malformations. The nurse describes that some common congenital malformations can occur and are recognized to be caused by multiple genetic and environmental factors. Which example would the nurse most likely cite? a. spina bifida b. cystic fibrosis c. color blindness d. hemophilia

a. Spina bifida is a multifactorial inherited disorder thought to be due to multiple genetic and environmental factors. Cystic fibrosis is considered an autosomal recessive inherited disorder, while color blindness and hemophilia are considered X-linked inheritance disorders.

A client has presented reporting symptoms that suggest a gonorrheal infection. After laboratory testing confirms this diagnosis, the nurse anticipates that the client will also be treated for which infection? a. chlamydia b. HPV c. syphilis d. candidiasis

a. The CDC recommends that any client being treated for gonorrhea should also be treated for chlamydia due to the common occurrence of co-infection of these two organisms.

The fluid-filled, inner membrane sac surrounding the fetus is which structure? a. amnion b. chorion c. endometrium d. decidua

a. The fluid-filled, inner membrane sac surrounding the fetus is the amnion. The chorion is the outer membrane surrounding the fetus. The endometrium is the inner lining of the uterus. The decidua is the name used for the endometrium during pregnancy.

A couple had decided not to circumcise their newborn for nonreligious reasons. What is the appropriate response from the nurse? a. "OK, great. I will let the physician know." b. "What arrangements have you made outside the hospital?" c. "That's very unusual; tell me what has led you to this decision." d. "It's much better for the health of your newborn if you circumcise him."

a. The nurse should remain as nonjudgmental as possible when speaking with a client. There are many reasons why a couple may decide to leave their newborn uncircumcised. Both health benefits and risks are associated with circumcision. The nurse should respect the couple's decision.

A nurse is preparing a teaching plan for a female client diagnosed with genital ulcers. Which instructions would the nurse include in this teaching plan? Select all that apply. a. "Apply ice packs to the area for 20 minutes every few hours." b. "Use a condom when having sexual intercourse with any noninfected partner." c. "Avoid having sex when any ulcers are present." d. "Try using cool sitz baths to help relieve the discomfort." e. "Air dry any lesions with a hair dryer on the low setting."

b, c, e. Teaching a client with genital ulcers includes avoiding extremes of temperature such as ice packs or hot packs, using a condom with all new or uninfected sexual partners, avoiding sex during the prodromal period and when lesions are present, using lukewarm sitz baths for discomfort, and air-drying lesions with a hair dryer on the low setting.

Which finding would the nurse most likely find in a male diagnosed with a chlamydia trachomatis infection? a. erectile dysfunction b. dysuria c. painful ejaculation d. hematuria

b. A male with the diagnosis of chlamydia will first experience dysuria as the primary symptom. The other symptoms listed do not correlate to men with the diagnosis of chlamydia.

The nurse is reviewing client data following a regular monthly appointment at 6 months' gestation. Which fundal height requires no further intervention? a. 18 cm b. 24 cm c. 30 cm d. 32 cm

b. An anticipated fundal height for 24 weeks' gestation (6 months) is 24 cm. Between 18 and 32 weeks' gestation, the fundal height in centimeters should match the gestational age. All of the other measurements would require further intervention.

A 47-year-old woman is being seen at the local clinic reporting irregular menstrual cycles. The nurse would explain to the client that the irregularity is most likely due to: a. pregnancy. b. menopause. c. hormonal fluctuations. d. an underlying endocrine problem.

b. As a woman's reproductive capabilities end, her menstrual cycles will become irregular and finally cease. The average age for menopause is 47 to 55 years. This client meets these criteria. Pregnancy would cause absence of menstrual cycles. Hormonal fluctuations and underlying endocrine problems would not be the anticipated causality.

Gynecologic health is an important part of a woman's health history. Which statement best illustrates the way to begin a menstrual history? a. "Discussing menstrual periods can be embarrassing. . ." b. "I'd like to ask you some questions about your menstrual periods." c. "I know you're probably uncomfortable talking about your health. . ." d. "I bet you have pain with menstrual periods."

b. Beginning any history with an open-ended question allows the woman the optimal opportunity to elaborate on her health concerns.

A young woman comes to the free clinic asking for oral contraceptive pills. Which factor best indicates that another type of contraception would be better for this client? a. cost of the pills b. history of noncompliance with medications c. age of her partner d. frequency of sexual encounters

b. If a woman has a history of poor compliance with medication, it might not be realistic for her to plan on taking an oral contraceptive pill every day.

Which definition best explains the term "subfertility/infertility"? a. failure to achieve pregnancy after 6 months of unprotected intercourse b. failure to achieve pregnancy after 1 year of unprotected intercourse c. inability to achieve pregnancy because of a known factor that prevents conception d. inability to achieve pregnancy following a previous viable pregnancy

b. Infertility is the failure to achieve conception after 1 year of unprotected intercourse. Because most couples have the potential to conceive but are just less able to conceive without additional help, the term subfertility is more often used today.

The nurse is teaching the pregnant woman about nutrition for herself and her baby. Which statement by the woman indicates that the teaching was effective? a. "I can eat any seafood that I like because it contains phosphorus, which is a nutrient that pregnant women need." b. "I will need to take iron supplementation throughout my pregnancy even if I am not anemic." c. "Milk production requires higher levels of calcium; therefore, if I am going to breastfeed, I must take a calcium supplement during pregnancy." d. "Because I am pregnant, I can eat anything I want and not worry about weight gain."

b. Iron is recommended for all pregnant women because it is almost impossible for the pregnant woman to get what is required from diet alone, especially after 20 weeks' gestation when the requirements of the fetus increase. Pregnant women can get many nutrients from seafood including phosphorus, but there are specific recommendations about types of fish to avoid because of the risk of mercury poisoning. Milk production actually requires higher levels of zinc, which can be obtained from a healthy diet. Calcium requirements do not increase above prepregnancy levels during pregnancy because calcium absorption is enhanced during pregnancy. It can be unsafe for the pregnant woman to eat anything she wants and gain too much weight. A woman who gains too much weight during pregnancy is at risk for delivering a macrosomic baby.

A couple has just been notified that their unborn child carries a genetic disorder. The couple expresses concern that the insurance company will not cover the costs associated with the medical bills for the child. What is the most appropriate response by the nurse? a. "The insurance company may consider it a preexisting condition since you know." b. "There are laws in place that prohibit that from happening." c. "The insurance company may ask you to change policies once the baby is born." d. "They will charge you a higher premium every month."

b. The Genetic Information Nondiscrimination Act of 2008 prohibits insurance companies from denying coverage or charging higher premiums based solely on genetic predisposition.

After teaching a class on the female reproductive system, the nurse determines that the teaching was successful when the class identifies which action as the primary function of the ovaries? a. carrying the ovum to the endometrium b. secreting estrogen and progesterone c. secreting mucus that supplies lubrication for intercourse d. implanting a fertilized ovum

b. The ovaries have two primary functions: development and release of the ovum and secretion of estrogen and progesterone. The fallopian tubes are responsible for conveying the ovum from the ovary to the uterus and sperm from the uterus toward the ovary. The Bartholin glands when stimulated secrete mucus that supplies lubrication for intercourse. The uterus is the site of implantation of a fertilized ovum.

A woman's prepregnant weight is within the normal range. During her second trimester, the nurse would determine that the woman is gaining the appropriate amount of weight when her weight increases by which amount per week? a. 2/3 lb (0.30 kg) b. 1 lb (0.45 kg) c. 1.5 lb (0.68 kg) d. 2 lb (0.90 kg)

b. The recommended weight gain pattern for a woman whose prepregnant weight is within the normal range would be 1 lb (0.45 kg) per week during the second and third trimesters. Underweight women should gain slightly more than 1 lb (0.45 kg) per week. Overweight women should gain about 2/3 lb (0.30 kg) per week.

The health care provider has prescribed an over-the-counter antacid for a pregnant client in her first trimester who is having ongoing nausea, vomiting, and heartburn. Which instruction concerning the antacid should the nurse prioritize after noting the client is also prescribed a multivitamin supplement? a. Avoid caffeinated beverages. b. Take only at bedtime. c. Take antacid 1 hour after the multivitamin. d. Take with dairy products.

c. Antacids interfere with the uptake of the vitamin contents so the client should take the antacid 1 hour after taking the multivitamin. Caffeine should be avoided due to increases in blood pressure and diuretic effects. Antacids can be taken more often than solely at bedtime, and some clients need them after each meal. Antacids do not have to be taken with dairy products. The priority is to avoid allowing the antacid to cancel out the multivitamin.

The nurse is preparing to teach a community class to a group of first-time parents. Which information should the nurse include concerning what the pregnant woman's partner may experience as a normal response? a. feeling distanced from the mother b. no changes, only the mother has changes during pregnancy c. physical symptoms similar to the mother d. desire to be the woman and give birth

c. Couvade syndrome is the occurrence of physical symptoms by the partner, similar to the physical symptoms of the mother. Other emotional symptoms may occur, but they are typically on a person-to-person basis.

A pregnant woman of Jewish descent comes to the clinic for counseling and tells the nurse that she is worried her baby may be born with a genetic disorder. Which disease does the nurse identify to be a risk for this client's baby based on the family's ancestry? a. sickle cell anemia b. beta-thalassemia c. Tay-Sachs disease d. Down syndrome

c. Sickle cell anemia occurs most often in African Americans. Tay-Sachs disease occurs most often in people of Jewish ancestry. Beta-thalassemia is a blood dyscrasia that occurs frequently in families of Greek or Italian heritage. Down syndrome is not attributed to Jewish ancestry.

During pregnancy a woman has many psychological adaptations that must be made. The nurse must remember that the baby's father is also experiencing the pregnancy and has adaptations that must be made. Some fathers actually have symptoms of the pregnancy along with the mothers. What is this called? a. pseudo pregnancy b. pregnancy syndrome c. couvade syndrome d. cretinism

c. Some fathers actually experience some of the physical symptoms of pregnancy, such as nausea and vomiting, along with their partner. This phenomenon is called couvade syndrome.

The nurse is educating the client at 12 weeks' gestation regarding the best types of exercise throughout pregnancy. Which activities should the nurse encourage? a. All activities that the client does in a prepregnant state b. Relaxing activities such as hot baths and jacuzzis c. High-impact movements enabling less time in the activity d. Stretching and breathing exercises such as yoga

d. It is important to exercise during pregnancy. One excellent type of exercise includes yoga, which reduces stress and increases relaxation. Yoga also gently stretches muscles and can increase muscle tone. Contact and high-impact sports are not appropriate for the pregnant mother. Hot areas such as a jacuzzi, hot tub, and sauna are also inappropriate.

Clients who have had PID are prone to which complication? a. ectopic pregnancy b. multiple gestation c. ovarian cancer d. inguinal lymphadenopathy

a. All clients who have had PID need to be informed of the signs and symptoms of ectopic pregnancy because they are prone to this complication.

A client who has been treated for recurring gonorrhea visits the sexual health clinic reporting dysuria, urinary frequency, and whitish-yellow vaginal discharge. A pelvic examination reveals cervicitis. How would the nurse interpret these findings? a. the possibility of a chlamydia infection b. an acute stage of the gonorrheal infection c. a reaction to the gonorrheal medications d. a positive pregnancy test

a. Any woman suspected of having gonorrhea should be tested for chlamydia also because coinfection (45%) is extremely common. The nursing management of chlamydia and gonorrhea includes treatment of both. The prevalence of chlamydia and gonorrhea is increasing dramatically.

Which would be a normal finding by the nurse during a physical exam of a woman in her third trimester? a. Dyspnea b. Kyphosis c. Ptyalism d. Increased hematocrit

a. In the third trimester, a women experiences dyspnea from the uterus pushing up into the diaphragm. A pregnant woman will experience lordosis, not kyphosis. Ptyalism is excessive saliva production and is often seen in the first trimester of pregnancy. The hematocrit of a pregnant woman will decrease in the third trimester, not increase.

The client at 32 weeks' gestation expresses concern regarding lower extremity edema and bulging leg veins. Which suggestion(s) by the nurse is helpful? Select all that apply. a. Limit fluid intake to 1 liter daily. b. Complete moderate exercise daily. c. Wear compression stockings. d. Keep legs below the level of the heart. e. Avoid sudden position changes.

b, c. Suggestions of exercising and wearing compression stockings are most helpful to the client with lower extremity edema and varicose veins. It is also helpful to have frequent rest periods with the legs elevated above the heart. Fluid intake is not to be limited. Avoiding sudden position changes helps prevent round ligament pain.

A nurse educator is describing prenatal care for clients at 15 weeks' gestation. What information would the nurse not include? a. The fetus makes active movements. b. Fetal weight will have tripled. c. Quickening is detected. d. The fetal head is dominant.

b. At weeks' 13 to 16 gestation, the fetus makes active movements. Quickening is detected by the mother during this time period and is an indicator of fetal wellness. Ultrasound will indicate a quadruple weight gain. The fetal head still remains the dominant structure.

The nurse is educating a woman about the importance of folic acid before conception and during pregnancy, to prevent neural tube defects in the fetus. The client plans to take prenatal vitamins and minerals. What food source would the nurse recommend to add to the woman's diet? a. Yogurt and low-fat milk b. Green leafy vegetables c. Oily fish such as salmon d. Green and iced tea

b. Green leafy vegetables are a good source of folic acid. In the past, green tea was thought to interfere with the absorption of folic acid; however, studies do not support this. The women would be advised to avoid green and iced tea due to the caffeine content. Yogurt, low-fat milk and oily fish are not known to be high in folic acid.

A group of student nurses are comparing the various events which take place during the sexual arousal of both males and females. The students are able to correctly point out which event is one of the underlying physiologic responses after the session? a. ovulation b. vasocongestion c. climacteric d. myospasm

b. There two underlying physiologic responses to sexual stimulation in both men and women: vasocongestion and myotonia (muscular tension). Myospasm is the spasmodic muscular contraction that is most often a sign of a muscular disorder. Ovulation occurs during the follicular phase of the menstrual cycle. Climacteric phase occurs during menopause when the various changes gradually take place.

A client is diagnosed with pediculosis infection. When teaching the client about measures to treat and minimize the transmission, which measures would the nurse include? Select all that apply. a. thoroughly shampooing all rugs and upholstery with hot soapy water b. washing all clothes and bed linens in warm water c. following the manufacturer's instructions for the medication d. not sharing any personal items with others e. removing any nits with a fine toothed comb

c, d, e. For a pediculosis infection, the nurse would teach the client to use medication according to the manufacturer's instructions; remove nits with a fine-toothed nit comb; not share any personal items with others or accept items from others; wash objects, clothing, and bedding in hot water; and meticulously vacuum carpets to prevent a recurrence of infestation.

What effect does progesterone have on normal gallbladder function? a. It has no effect on the gallbladder. b. The gallbladder will hypertrophy. c. Progesterone interferes with gallbladder contraction, leading to stasis of bile. d. Bile will be produced at a more rapid rate due to the progesterone.

c. Progesterone interferes with normal gallbladder contractions, which leads to stasis of bile. This stasis results in cholestasis, either seen in the gallbladder or the liver.

Which recommendation would the nurse give to a pregnant client with a sexually transmitted infection who is at risk for transmitting the infection? a. Pat or blot the skin dry. b. Have regular cancer screening examinations. c. Participate in early prenatal care. d. Take tub baths regularly.

c. The nurse should recommend that a pregnant client with a sexually transmitted infection who is at risk for infection transmission receive early prenatal care because some STIs can be transmitted during birth. Patting or blotting the skin dry is helpful in reducing friction and the itch-scratch-itch cycle. The nurse should recommend clients infected with carcinogenic viruses to have regular cancer screening examinations to facilitate early diagnosis and optimistic prognosis. Taking tub baths regularly would have no effect on infection transmission.

A nurse is assessing a client diagnosed with bacterial vaginosis. What is a symptom of bacterial vaginosis? a. cottage cheese-like discharge b. intense itching of the vulva c. warts in the vulva and labia d. vaginal odor smelling of fish

d. Bacterial vaginosis causes a "stale fish" vaginal odor. Cottage cheese-like discharge is seen in candidiasis and not in bacterial vaginosis, in which the discharge is thin and gray-white. Intense itching of the vulva is associated with candidiasis, not vaginosis. Warts in the vulva and labia are seen in human papillomavirus infection, not in vaginosis.

Utilize the GTPAL system to classify a woman who is currently 18 weeks pregnant. This is her 4th pregnancy. She gave birth to one baby vaginally at 26 weeks who died, experienced a miscarriage, and has one living child who was delivered at 38 weeks gestation. a. 3, 2, 1, 2, 1 b. 4, 2, 2, 1, 1 c. 3, 2, 1, 1, 1 d. 4, 1, 1, 1, 1

d. The GTPAL system is used to classifying pregnancy status. G = gravida, T= term, P = preterm, A = number of abortions, L= number of living children.

A female client is prescribed metronidazole for the treatment of trichomoniasis. Which instruction should the nurse give the client undergoing treatment? a. Avoid extremes of temperature to the genital area. b. Use condoms during sex. c. Increase fluid intake. d. Avoid alcohol.

d. The nurse should counsel the client taking metronidazole to avoid alcohol during the treatment because mixing the two causes severe nausea and vomiting. Avoiding extremes of temperature to the genital area is a requirement for clients with genital ulcers, not trichomoniasis. The nurse should instruct the client to avoid sex, regardless of using condoms, until she and her sex partners are cured (when therapy has been completed and both partners are symptom-free). It is not required to increase fluid intake during treatment.

Which physical signs would a parent note in a child who is entering puberty? Select all that apply. a. Pubic hair in both genders b. Increased height c. Weight gain d. Breast buds e. Deepening of voice in males f. Penile growth

a, d, e, f. The changes noted in individuals entering puberty include growth of external genitalia, growth of body and facial hair, female breast development, possible male breast development (called gynecomastia), and deepening of the male voice. Puberty does not include increased height and weight of the individuals. Increases in both of these are individualized with some adolescents growing faster than others, so these are not good gauges of puberty.

A nurse educator is conducting a class on intimate partner violence for a group of new graduate nurses during orientation. Which statement by the educator best reflects current practice regarding these problems in women's health? a. "Asking every client about intimate partner and family violence is the best way to elicit accurate information." b. "The nurse should screen for these problems at every client encounter." c. "The nurse is not legally responsible for reporting suspected intimate partner violence." d. "Since families are more stable than in the past, nurses are not as concerned about these problems as they used to be."

b. Both child and intimate partner violence is increasing in incidence. Families are more mobile than in the past. Screening for child abuse (child maltreatment) or intimate partner violence should be included in all family contacts. Nurses must be aware of the legal responsibilities for reporting violence.

A urinalysis is done on a client in her third trimester. Which result would be considered abnormal? a. Trace of glucose b. 2+ Protein in urine c. Specific gravity of 1.010 d. Straw-like color

b. During pregnancy, there may be a slight amount of glucose found in the urine due to the fact that the kidney tubules are not able to absorb as much glucose as there were before pregnancy. However, there should be minimal protein in the urine. A specific gravity of 1.010 and a straw- like color are both normal findings.

A group of nursing students are analyzing the fetal circulation. After the session, the students correctly point out which fetal structure contains the highest concentration of oxygen? a. umbilical artery b. umbilical vein c. ductus arteriosus d. pulmonary vein

b. The umbilical vein carries oxygenated blood from the placenta to the fetus; the umbilical artery carries deoxygenated blood from the fetus to the placenta. The ductus arteriosus shunts blood from the right atrium to the left atrium in order to bypass the deflated lungs. The pulmonary vein drains the deflated lungs.

A woman who is 4 months pregnant notices frequent heart palpitations and leg cramps. She is anxious to learn how to alleviate these. Which nursing diagnosis would best apply to her? a. Impaired urinary elimination related to inability to excrete creatine from her muscles b. Risk for ineffective breathing pattern related to pressure of the growing uterus c. Pain related to severe complications of pregnancy d. Health-seeking behaviors related to ways to relieve discomforts of pregnancy

d. Health-seeking behaviors is a diagnosis used to describe clients who are actively interested in learning ways to improve their health.

Which possible complication associated with back pain can lead to premature contractions? a. increased intracranial pressure b. leak of spinal fluid into the epidural space c. herniated disc d. bladder or kidney infection

d. Obtaining a detailed account of a woman's back symptoms is crucial because back pain can be an initial sign of a bladder or kidney infection. Increased ICP, spinal fluid leak, and a herniated disc are usually not associated with back pain during a normal pregnancy.

A 17-year-old client arrives for an annual examination and reports no changes since the last exam; however, the nurse assesses a positive Chadwick sign, slightly enlarged uterus, and subsequent positive urine pregnancy test. Which task should the nurse prioritize to assist this client who is denying any possibility that she is pregnant? a. Accepting the pregnancy b. Accepting the baby c. Preparing for parenthood d. Telling her partner and family

a. Acceptance of pregnancy is multi-factorial, and how the woman responds to the pregnancy is certainly influenced by her age and if the pregnancy was planned. As a teenager, she may not have been trying to get pregnant and may not want to accept the role and experience. Baby and parenthood decisions should all occur later.

A client presents at the clinic and is interested in obtaining emergency contraception (EC). The nurse explains that EC must be used within 72 hours of unprotected sex to be effective. This is because: a. ECs simply prevent embryo creation and uterine implantation from occurring in the first place. b. ECs can induce an abortion (elective termination of pregnancy) of a recently implanted embryo. c. ECs can help prevent STIs. d. ECs are more effective than regular birth control.

a. ECs prevent the embryo creation and uterine implantation from occurring. There is no evidence that ECs have any effect on an already-implanted ovum or that they induce abortion (elective termination of pregnancy). They do not protect against STIs and are less effective than regular birth control.

At 32 weeks' gestation a client with a BMI of 23 has gained 24 lb (11 kg). What is the nurse's recommendation for weight gain for the remainder of this pregnancy? a. Continue to gain approximately 1 lb (.45 kg) per week during this pregnancy. b. Watch the diet so no additional weight is gained during this pregnancy. c. Limit weight gain to less than 5 lb (2 kg) for the remainder of this pregnancy. d. Increase weight gain to 1.5 lb (0.68 kg) per week during this pregnancy.

a. Expected weight gain is 1.5 lb (0.68 kg) per month in the first trimester and 1 lb (.45 kg) per week for the second and third trimester. This client needs to continue to gain 1 lb (.45 kg) per week. Restricting weight gain near the end of pregnancy can negatively impact fetal growth.

A nurse is reviewing the history and physical examination of a client diagnosed with secondary dysmenorrhea for possible associated causes. Which etiology would the nurse need to keep in mind as being the most common? a. endometriosis b. multigravida status c. hormonal imbalance d. perimenopause

a. Secondary dysmenorrhea is painful menstruation due to pelvic or uterine pathology. Endometriosis is the most common cause of secondary dysmenorrhea. Other recognized causes include adenomyosis, fibroids, pelvic infection, an intrauterine device, cervical stenosis, or congenital uterine or vaginal abnormalities.

Which medication is used to suppress viral load of the HSV-2 infection? a. acyclovir b. penicillin c. metronidazole d. clindamycin

a. The antiviral agents acyclovir, valacyclovir, and famciclovir are recommended to suppress the viral load and decrease recurrence and shedding. Metronidazole and clindamycin are not used for this action.

A client who has just given a blood sample for pregnancy testing in the health care provider's office asks the nurse what method of confirming pregnancy is the most accurate. The nurse explains the difference between presumptive symptoms, probable signs, and positive signs. What should the nurse mention as an example of a positive sign, which may be used to diagnose pregnancy? a. visualization of the fetus by ultrasound b. laboratory test of a urine specimen for hCG c. laboratory test of a blood serum specimen for hCG d. absence of a period

a. There are only three documented or positive signs of pregnancy: 1) demonstration of a fetal heart separate from the mother's, 2) fetal movements felt by an examiner, and 3) visualization of the fetus by ultrasound. The absence of a period is an example of a presumptive symptom, which is a symptom that, when taken as a single entity, could easily indicate other conditions. Laboratory tests of either urine or blood serum for human chorionic gonadotropin (hCG) are examples of probable signs of pregnancy, which are objective and so can be verified by an examiner.

Amniotic fluid is produced throughout the pregnancy by the fetal membranes. Amniotic fluid has four major functions. What is one of these functions? a. Medium in which to test organ maturity b. Physical protection c. Provide fluid to keep the fetus hydrated d. Restriction of movement

b. Amniotic fluid serves four main functions for the fetus: physical protection, temperature regulation, provision of unrestricted movement, and symmetrical growth.

While assessing a client's breast during the third trimester, which finding would the nurse expect? a. breasts becoming soft b. colostrum from the nipples c. pink-colored nipples d. pain in the nipple area

b. During the third trimester, the nipples express colostrum. Areolae and nipples appear enlarged with darker pigmentation during the third trimester. The nurse assesses for the softness of the breast, color, and pain in the nipple area in nursing mothers.

A group of students is reviewing class material on sexually transmitted infections in preparation for a test. The students demonstrate understanding of the material when they identify which cause of condylomata? a. herpes virus b. human papillomavirus c. Treponema pallidum d. Haemophilus ducreyi bacillus

b. Genital warts or condylomata are caused by the human papilloma virus (HPV). Herpes virus causes genital herpes. Treponema pallidum is the cause of syphilis. Haemophilus ducreyi bacillus is the cause of chancroid.

Which information is important for a woman to understand before undergoing a scheduled tubal ligation? a. She will have lessened dysmenorrhea following the procedure. b. She must think of the procedure as irreversible. c. The procedure will reduce her menstrual flow in amount. d. She should schedule it to be done just before a menstrual flow.

b. Sterilization by fallopian tube obstruction does not alter menstrual symptoms or flow. Ectopic pregnancy could result if it is done following ovulation; reversing the process is difficult.

The nurse working in a free health clinic assesses a 17-year-old client interested in contraceptives. Which statement by the client would indicate that female or male condoms would be the appropriate recommendation? a. "I have no problems swallowing medications." b. "Last year I was diagnosed with HPV." c. "I am leaving for college in just a few months." d. "I do not want to follow in my sister's footsteps."

b. The client has already contracted HPV; to prevent further spread of it—or contracting other STIs—the nurse should recommend the client use male or female condoms.

A pregnant client in her first trimester visits the health care facility for regular checkups. The nurse instructs the client to increase her dietary intake of folic acid based on the understanding that folic acid is important for which action? a. increasing maternal energy level b. decreasing incidence of birth defects c. increasing resistance to diseases d. decreasing risk of breast cancer

b. The nurse should inform the pregnant client that folic acid is important because it decreases the risk of birth defects, including neural tube defects. Folic acid also helps to synthesize DNA and supports placental and fetal growth. Folic acid has no effect on increasing the mother's energy level or resistance to diseases or decreasing her risk of breast cancer.

A woman is 20 weeks pregnant. The nurse would expect to palpate the fundus at which location? a. symphysis pubis b. between the symphysis and umbilicus c. at the umbilicus d. just below the ensiform cartilage

c. At 20 weeks, the fundus can be palpated at the umbilicus. A fundus of 12 weeks' gestation is palpated at the symphysis pubis. At 16 weeks' gestation, the fundus is midway between the symphysis pubis and umbilicus. At 36 weeks' gestation, the fundus can be palpated just below the ensiform cartilage.

A client is to take clomiphene citrate for infertility. Which outcome should the nurse explain is the expected action of this medication? a. Reduce inflamed endometrium b. Break down scar tissue c. Stimulate the release of ova d. Decrease testosterone levels

c. Clomiphene citrate stimulates the release of FSH and LH which is responsible for the maturity and release of ovum from the ovary.

A pregnant client arrives for her first prenatal appointment. She reports her previous pregnancy ended at 19 weeks, and she has 3-year-old twins born at 30 weeks' gestation. How will the nurse document this in her records? a. G2 T2 P1 A0 L2 b. G2 T1 P1 A1 L1 c. G3 T0 P1 A1 L2 d. G3 T2 P2 A0 L1

c. G indicates the total number of pregnancies (2 prior, now pregnant = 3); T indicates term deliveries at or beyond 38 weeks' gestation (none = 0); P is for preterm deliveries (at 20 to 37 weeks = 1; multiple fetus delivery are scored as 1); A is for abortions or pregnancies ending before 20 weeks' gestation (1); and L refers to living children which is 2. Thus, G3 T0 P1 A1 L2 is what the nurse should note in the client's record.

The nurse is reviewing the medical records of several infants. Which infant is at highest risk for death according to the infant mortality rate in the United States? a. An infant born to adolescent parents b. An infant born at 38 weeks' gestation c. An infant born at a low birth weight d. An infant born to a 43-year-old mother

c. Infant mortality in the United States is associated with low birth weight, prematurity, and congenital abnormalities, among other concerns. An infant born to young or older parents, or at 38 weeks' gestation, would not be considered low birth weight or premature nor considered to be at higher risk for congenital abnormalities.

Which change related to the vital signs is expected in pregnant women? a. Pulse decreases. b. Lung space increases. c. Blood pressure decreases. d. Temperature decreases.

c. Pulse and temperature often increase, while lung space is decreased in pregnant women. It is common for blood pressure to decrease during pregnancy.

A pregnant client in her third trimester, lying supine on the examination table, suddenly grows very short of breath and dizzy. Concerned, she asks the nurse what is happening. Which response should the nurse prioritize? a. Cerebral arteries are growing congested with blood. b. The uterus requires more blood in a supine position. c. Blood is trapped in the vena cava in a supine position. d. Sympathetic nerve responses cause dyspnea when a woman lies supine.

c. Supine hypotension syndrome, or an interference with blood return to the heart, occurs when the weight of the fetus rests on the vena cava. Cerebral arteries should not be affected. Mean arterial pressure is high enough to maintain perfusion of the uterus in any orientation. The sympathetic nervous system will not be affected by the supine position.

A nurse is teaching a client who is 30 weeks' pregnant about ways to deal with pyrosis (heartburn). The nurse determines a need for additional teaching based on which client statement? a. "I need to cut out caffeine." b. "I should chew my food slowly." c. "I should lie down for 1/2 hour after eating." d. "I need to raise the head of my bed about 15 to 30 degrees."

c. The client should remain sitting for 1 to 3 hours after eating and avoid lying down within 3 hours of eating. Cutting out caffeine, chewing food slowly, and raising the head of the bed are helpful in reducing pyrosis (heartburn) of pregnancy.

A client in her third trimester reports to the nurse shortness of breath when sleeping. The nurse informs the client that this is normal and occurs because the growing fetus puts pressure on the diaphragm. Which measure should the nurse suggest to help alleviate this problem? a. Avoid overeating. b. Lie on a firmer mattress. c. Use extra pillows. d. Avoid spicy food.

c. The nurse should instruct the client to use extra pillows at night to keep her more upright. The nurse can instruct the client to use a firmer mattress if the client is experiencing backache. The nurse can ask the client to avoid overeating and ingesting spicy food in case the client is experiencing heartburn.

The nurse is assessing several pregnant women in a clinic setting. Which assessment finding would alert the nurse to notify the health care provider? a. increased nasal congestion b. increased urination and fatigue c. increased skin pigment d. blood pressure measured at 170/88 mm Hg

d. During pregnancy, women may expertise increased nasal stuffiness, increased urination, fatigue, and skin pigment increases. Elevated blood pressure is a concern during pregnancy and would be reported.

The nurse is describing pregnancy danger signs to a pregnant woman who is in her first trimester. Which danger sign might occur at this point in her pregnancy? a. dyspnea b. lower abdominal pressure c. swelling of extremities d. excessive vomiting

d. Excessive vomiting is a warning sign in the first trimester. Dyspnea, lower abdominal pressures, and swelling of face or extremities may occur late in pregnancy.

A nurse is caring for a couple during a prenatal clinic visit. Which assessment finding would lead the nurse to suggest genetic counseling for the couple? a. The mother is from a larger family, including seven brothers and three sisters. b. This is the second marriage for the 48-year old father. c. The mother expresses concern about her stepchildren accepting the baby. d. The mother just turned 39 years of age.

d. Genetic counseling is indicated for any woman older than 35 years of age and any man older than 55 years of age. This is directly related to the association between advanced parental age and the occurrence of Down syndrome. The mother's family history would be significant if there were indications of inherited diseases, congenital anomalies, or other such disorders. The father's age would not be a concern at 48 years old. The family may benefit from family counseling to assure the blended family is healthy, but genetic counseling cannot help with that issue.

A young woman presents with vaginal itching and irritation of recent onset. Her labia are swollen, and she has a frothy yellowish discharge with an unpleasant smell and a pH of 6.8. She has been celibate during the last six months and has been taking antibiotics for a throat infection. Which medication is most likely to clear her symptoms? a. azithromycin b. valacyclovir c. penicillin d. metronidazole

d. The character of the discharge, lack of recent sexual activity, and current antibiotic treatment point to infection with Trichomonas vaginalis, which can exist asymptomatically and flare up only if conditions, such as an imbalance in normal vaginal flora resulting from antibiotic treatment, enable the protozoan to proliferate. Trichomoniasis responds well to treatment with metronidazole.

A woman visits the family planning clinic to request a prescription for birth control pills. Which factor would indicate that an ovulation suppressant would not be the best contraceptive method for her? a. She is 30 years old. b. She has irregular menstrual cycles. c. She has a history of allergy to foreign protein. d. She has a family history of thromboembolism.

d. The estrogen content of birth control pills may lead to increased blood clotting, leading to an increased incidence of thromboembolism. Women who already are prone to this should not increase their risk further.

The nurse is providing education on the prevention of sexually transmitted infections (STIs) to clients. The nurse understands which client is at the highest risk of contracting an STI? a. A 17-year-old client who is in a monogamous relationship with a 25-year-old partner and engages in unprotected sex. b. A 20-year-old client who reports having protected sexual intercourse with three others, male and female, over the past five years. c. A 25-year-old client who refused to receive the human papillomavirus vaccine and whose parents both have HIV. d. A 30-year-old client who consumes alcohol multiple times per week, uses cocaine, and has a history of prostitution.

d. The nurse will look for the client with the most risk factors for contracting an STI. The 30-year-old client is at the highest risk due to the use of alcohol and drugs and a history of forced sexual activity (3 risk factors). Other risk factors include age of 24 years or less, multiple partners, unprotected intercourse, and a history of STIs. Although the 17-year-old client is young and having unprotected intercourse, the client is in a monogamous relationship, indicating only 2 risk factors. The 20-year-old client is young and has protected intercourse with multiple partners (2 risk factors). The 25-year-old has no risk factors.

The nurse is caring for a couple trying to conceive their first child. The couple state they have been attempting to become pregnant for the past year. Which finding in the couple's history most concerns the nurse? a. The male client drinks 4 to 5 beers per week. b. The female client's mother had breast cancer. c. The male client's sister experienced infertility. d. The female client has a history of pelvic inflammatory disease (PID).

d. The nurse would be most concerned learning the female client has a history of PID, because this can cause scarring and lead to subfertility. Consuming 4 to 5 alcoholic drinks per week is not concerning. Excessive alcohol consumption can interfere with fertility and would be addressed. The female client's mother having breast cancer does place the client at increased risk; however, this is not the primary concern at this time. The male client's sister having infertility does not indicate a genetic complication because one sibling is male and the other female.

A client in the third trimester of pregnancy has to travel a long distance by car. The client is anxious about the effect the travel may have on her pregnancy. Which instruction should the nurse provide to promote easy and safe travel for the client? a. Activate the air bag in the car. b. Use a lap belt that crosses over the uterus. c. Apply a padded shoulder strap properly. d. Always wear a three-point seat belt.

d. To promote easy and safe travel for the client, the nurse should instruct the client to always wear a three-point seat belt to prevent ejection or serious injury from collision. The nurse should instruct the client to deactivate the air bag if possible. The nurse should instruct the client to apply a nonpadded shoulder strap properly, ensuring that it crosses between the breasts and over the upper abdomen, above the uterus. The nurse should instruct the client to use a lap belt that crosses over the pelvis below—not over—the uterus.

For which client is preimplantation genetic diagnosis (PGD) a viable option? a. client who is currently 32 years of age b. client who had a miscarriage at 15 weeks' gestation c. client whose partner has cystic fibrosis d. client in the second week of pregnancy

c. Preimplantation genetic diagnosis (PGD) is a viable option when either parent is a carrier of a genetic disorder. It is also an option when the mother is of advanced maternal age or has a history of multiple miscarriages. PGD is not a viable option for pregnant clients.

A woman comes to the clinic reporting intense pruritus and a thick curd-like vaginal discharge. On examination, white plaques are observed on the vaginal wall. The nurse suspects which condition? a. trichomoniasis b. bacterial vaginosis c. vulvovaginal candidiasis d. chlamydia

c. Pruritus accompanied by a thick curd-like vaginal discharge and white plaques on the vaginal wall are characteristic of vulvovaginal candidiasis. Trichomoniasis is characterized by a heavy yellow or green or gray frothy or bubbly discharge. Bacterial vaginosis is characterized by a thin white homogeneous vaginal discharge. Chlamydia is usually manifested by a mucopurulent vaginal discharge.

A pregnant client is diagnosed with hydramnios. The nurse explains that further testing will be done to determine if which conditions are present? Select all that apply. a. Esophageal atresia b. Anencephaly c. Renal atresia d. Anuria e. Fetal hypoxia

a, b. Hydramnios or excessive amniotic fluid is found when the fetus has a condition such as esophageal atresia or anencephaly, which inhibit swallowing. Renal atresia or renal conditions that decrease urine production results in oligohydramnios. The volume of amniotic fluid protects the umbilical cord from pressure thus protecting the fetal oxygen supply.

Consent for urgent treatment is needed for a minor. The parents are unable to be at the hospital. What action by the nurse constitutes informed consent? a. Telephone consent with two witnesses listening simultaneously. b. Explaining the needed treatment to the minor and documenting this action. c. Contacting the institution's attorney to provide and document consent. d. Treating the minor and obtaining written informed consent when the parent arrives.

a. Telephone consent documented with two witness signatures is appropriate. None of the other options constitutes informed consent.

A 28-year-old client states that she has not had her menstrual period for the past 3 months and suspects she is pregnant. Which should the nurse do next? a. Determine at what age the client began menstruating. b. Have the client take a pregnancy test. c. Assess the client for a fetal heart tone. d. Ask the client the date her last period ended.

b. Absence of menstruation, along with consistent nausea, fatigue, breast tenderness, and urinary frequency, are the presumptive signs of pregnancy. To determine if the client may be pregnant, a pregnancy test is indicated.

At a prenatal checkup with a client at 7 weeks' gestation, the nurse would identify what as a normal finding? a. Quickening is detected by the mother. b. Fetal heart sounds are heard. c. Gender identity is determined with ultrasound. d. The fetus would have a startle reflex.

b. Although the heart is not fully developed, it begins to beat at week 5, and a regular rhythm and can be heard at week 7. Quickening is felt around week 13. Gender identity can be determined at weeks 9 to 12. The startle reflex can be seen around weeks 21 to 24.

A client is confused concerning her fertile period each month. The nurse would educate the client by providing which information? a. The female fertility period is from Day 12 to Day 21 of her menstrual cycle. b. Fertilization can occur only if intercourse occurs on the day of ovulation. c. The window of opportunity for conception is from 3 days before to 2 days following ovulation. d. Since a sperm can only live 12 to 24 hours after ejaculation, fertility is limited to 1 day before to 1 day after ovulation.

c. Since sperm are viable for 24 to 72 hours after ejaculation and the ovum remains fertile for a maximum of 48 hours after ovulation, the window is from 3 days before to 2 days after ovulation.

The nurse is preparing to administer a prescribed medication to the pregnant client. Which order should the nurse question? a. penicillin b. rubella c. acetaminophen d. folic acid

b. Most vaccines are contraindicated during pregnancy and are considered teratogenic, such as rubella. Penicillin and acetaminophen may be taken under provider supervision. Folic acid supplementation should be encouraged.

A nurse is conducting an orientation at a health clinic for newly hired nurses comparing normal and abnormal findings from a female physical assessment. The nurse determines the session is successful when they correctly point out which structure's secretions assist with sexual intercourse? a. introitus b. Bartholin glands c. vestibule d. labia minora

b. The Bartholin glands are located in the vestibule and secrete a mucus that moistens the vaginal mucosa during sexual intercourse. The introitus is the opening to the vagina. The vulva vestibule is the area between the labia minora. It contains the ureteral meatus, Skene glands, introitus, and Bartholin glands. The labia minora are paired erectile tissue folds that extend anteriorly from the clitoris and join at the perineum.

Which client should the nurse flag for the health care provider to assess first? a. the female client with an inverted, pear-shaped uterus b. the male client with three palpable masses in the testes c. the female client with a soft, doughnut-shaped cervix d. the male client whose scrotum is covered with hair

b. The testes should have two nut-like structures present in males. The third mass could indicate a tumor or other anomaly. All the remaining clients have normal, expected findings.

The nurse is assessing a client at her first prenatal visit and notes that she is exposed to various chemicals at her place of employment. Which statement by the client would indicate she needs additional health education to protect her and her fetus? a. "I only work four hours a day so I don't get exposed too much." b. "The gloves they provide irritate my hands, so I don't use them." c. "There hasn't been a chemical spill in three years." d. "I have an assistant helping me now to handle the chemicals."

b. There are various chemicals which are recognized for their teratogenic effects and must be avoided during pregnancy. The nurse should find out which chemicals the client is exposed to and determine the risk factor. The greatest danger is the client handling chemicals without a barrier protection such as gloves. The other issues may also be dangers depending on the chemicals and the environment in which the client is working and should also be evaluated.

The nurse is planning to speak at a local community center to a group of middle-aged women about osteoporosis. Which measure would the nurse be sure to include as effective in reducing the risk for osteoporosis? a. decreasing vitamin D intake b. drinking at least 1 glass of wine per day c. engaging in daily weight-bearing exercise d. limiting intake of cholesterol and saturated fats

c. Engaging in daily weight-bearing exercise such as walking helps to reduce a woman's risk for osteoporosis. To prevent osteoporosis, women should increase their calcium and vitamin D intake. Avoiding excessive alcohol ingestion helps prevent osteoporosis. Limiting intake of cholesterol and saturated fats helps reduce the risk for cardiovascular disease, not osteoporosis.

The nurse is completing the initial assessment at the prenatal visit of a pregnant client. Which question should the nurse prioritize when completing the review of systems? a. "Have you ever had a heart attack?" b. "Do you have a peptic ulcer?" c. "Have you had any urinary tract infections?" d. "Have you had any neurologic diseases?"

c. It is important to ensure the woman does not have any current infections as they can all contribute to adverse effects in the pregnancy. Any conditions the woman has had in the past may recur or be exacerbated during pregnancy. It is also possible for the woman to currently have a low-grade infection and not be aware of it. A urine culture may be required to ensure the woman does not currently have an infection. UTIs can contribute to premature labor.

A woman seen in the emergency department is diagnosed with primary syphilis. What finding is most likely? a. foul-smelling discharge b. vaginal bleeding c. vesicles on the labia d. chancres at the vaginal site

d. Syphilis is divided into four stages: primary, secondary, latency, and tertiary. Primary syphilis is characterized by a chancre at the site of bacterial entry. Foul-smelling discharge would not be noted. Vesicles on the labia are not associated with syphilis. Bleeding from the vagina could be normal menses and does not correlate with syphilis.

A nursing student is preparing a presentation illustrating the Human Genome Project. Which function will the student point out as being the primary focus? a. Genetic testing in adults b. Detection of genetic mutations in children c. Identification of human genes and functions d. Treatment of gene mutations

c. The Human Genome Project was founded in 1990 and its primary purpose was to identify and label all human genes and their functions. Information from the HGP may eventually be used in genetic testing, the detection of mutations or variations in children, and gene therapy.

The nurse is counseling a young woman who has just entered her second trimester, after an uneventful first trimester. She tells the nurse, "It still doesn't seem real. It's just hard to believe that I will really have a baby." Which future events should the nurse point out that will help the young woman come to believe it is real? Select all that apply. a. Feeling the baby kick b. Seeing an ultrasound image of the baby c. Giving up alcohol d. Receiving a positive result on a pregnancy test e. Taking prenatal vitamins

a, b. During the second trimester, the psychological task of a woman is to accept she is having a baby, a step up from accepting the pregnancy. This change usually happens at quickening, or the first moment a woman feels fetal movement. Shopping for baby clothes for the first time, setting up the crib, seeing a blurry outline on a sonogram screen: any of these small actions may suddenly make the coming baby seem real and desired. Clearly, receiving a positive result on a pregnancy test was not enough to help this woman accept that she was having a baby, as this has already happened. Taking prenatal vitamins and giving up alcohol are more likely to be indicators that the woman has accepted the pregnancy rather than aiding her in accepting the baby.

On a routine hematocrit screen during a prenatal visit, the nurse notices that the client is mildly anemic. When discussing this with the couple, the husband hints that she might be eating unusual things. The nurse recognizes the need for the woman to be evaluated for which condition? a. pica b. vegan diet c. food cravings d. food allergy

a. Pica is the compulsive ingestion of nonfood substances. Pregnant women who develop a pica habit typically have one or two specific cravings. The three main substances consumed by women with pica are soil or clay, ice, and laundry starch. These substances replace nutritive sources and can lead to complications such as iron-deficiency anemia, infection, and constipation.

A mother has come to the clinic with her 13-year-old daughter to find out why she has not started her menses. After a thorough examination and history, genetic testing is prescribed to rule out which abnormality? a. Turner syndrome b. Klinefelter's syndrome c. fragile X syndrome d. cri du chat syndrome

a. Turner syndrome is a common abnormality of the sex chromosome in which a portion or all of the X chromosome is missing. Only about one third of the cases are diagnosed as newborns; the remaining two thirds are diagnosed in early adolescence when they experience primary amenorrhea. No cure exists for this syndrome. Hormone replacement therapy may be used to induce puberty.

A fertilized ovum is known as which structure? a. fetus b. zygote c. embryo d. chorion

b. A fertilized ovum is known as a zygote and is the beginning of potential individual human development. The developing human organism is known as an embryo from the time it implants on the uterine wall until the eighth week after inception and as a fetus from the beginning of the ninth week after fertilization through birth. The chorion is the outermost cell layer that surrounds the embryo and fluid cavity.

The nurse is preparing a presentation for a local community group about sexually transmitted infections (STIs). Which most common bacterial STI in the United States would the nurse expect to include? a. gonorrhea b. chlamydia c. syphilis d. genital herpes

b. Chlamydia is the most common and fastest-spreading bacterial STI in the United States, with 2.8 million new cases occurring each year. Gonorrhea is the second most frequently reported communicable disease in the U.S. The incidence of syphilis had been increasing for the past 6 years. Genital herpes is a viral STI. One in five people older than age 12, however, is infected with the virus that causes genital herpes.

A woman has come to the clinic for her first prenatal visit. Which method would be the most effective way for the nurse to initiate data gathering for a health history? a. Ask her to complete a written questionnaire concerning her past and present status. b. Conduct an interview in a private room to obtain her health history. c. Wait until she is in the examining room and prepared for her physical examination. d. Ask her some basic questions in the waiting room before taking her to the examining room.

b. Health interviewing is always conducted best in a quiet, private setting before examination procedures begin.

During a routine visit to the clinic, a client tells the nurse that she thinks she may be pregnant. The physician prescribes a pregnancy test. The nurse should know the purpose of this test is to determine which change in the client's hormone level? a. decrease in human chorionic gonadotropin (hCG) b. increase in human chorionic gonadotropin (hCG) c. increase in luteinizing hormone (LH) d. decrease in LH

b. Human chorionic gonadotropin (hCG) increases in a woman's blood and urine to fairly large concentrations until the 15th week of pregnancy. The other hormone values are not indicative of pregnancy.

A nurse is preparing a presentation for a group of nurses about genetic discoveries and advances. When discussing this topic, the nurse would most likely include which topic area related to the potential for misuse? a. gene replacement therapy for defective genes b. individual risk profiling and confidentiality c. greater emphasis on the causes of diseases d. slower diagnosis of specific diseases

b. Individual risk profiling based on an individual's genetic makeup can raise issues related to privacy and confidentiality. Gene replacement therapy for defective genes and a greater emphasis on looking at the causes of disease are considered benefits associated with genetic advances. Rapid, more specific diagnosis of diseases would be possible.

A 20-year-old female comes to the sexual health clinic for follow up related to a positive test for the human papillomavirus (HPV). The client asks the nurse, "Is there anything I can do to get rid of this?" What is the nurse's best response? a. "The lumps on your cervix can be cured by cryosurgery." b. "There is currently no medical treatment to cure HPV." c. "This can be cured with medications such as the recombinant human papillomavirus quadrivalent vaccine." d. "The health care provider will prescribe antibiotics to cure this infection."

b. It is a lifelong recurrent viral disease treated but not cured with medical treatment. Immunization regimes such as the recombinant human papillomavirus quadrivalent vaccine are for HPV prevention not cure. Cryosurgery will eliminate HPV warts but not cure it. Antibiotics will not be effective for a virus.

Which information is most important in order to decrease the risk of complications if the client decides to work until her due date? a. Eat light meals b. Frequent rest periods c. Adequate sleep d. Flat shoes

b. It is common to have a client work until she goes into labor as long as she has had a low-risk pregnancy. Frequent rest periods are stressed, if possible, as the client progresses throughout the work day. The other options are good suggestions for any client at the end of pregnancy.

A nurse is assessing a female client. When obtaining the client's history, the client tells the nurse that her menstrual cycle is usually 30 days and that she expects her next menstruation on March 20. Based on the client's information and the nurse's understanding of the female reproductive cycle, the nurse would identify ovulation as most likely occurring at which time? a. March 1 b. March 6 c. March 13 d. March 27

b. Ovulation usually occurs on day 14 in a 28-day cycle; however, whether a woman's cycle is 28 days or 120 days, ovulation takes place 14 days before menstruation.

The nurse explains to a client diagnosed with trichomoniasis that the disease is caused by the parasitic protozoan Trichomonas vaginalis. What would the nurse say is a factor that triggers growth of trichomoniasis? a. trauma to the bladder b. being postmenopausal c. irritation of vaginal walls d. spontaneous abortion (miscarriage)

c. Factors that trigger growth of trichomoniasis include irritation of vaginal walls, pregnancy, sexual activity, trauma to the vaginal walls, systemic illness, menstruation, and emotional upsets.

Before becoming pregnant, a woman's heart rate averaged 72 beats per minute. The woman is now 15 weeks' pregnant. The nurse would expect this woman's heart rate to be approximately: a. 85 beats per minute. b. 90 beats per minute. c. 95 beats per minute. d. 100 beats per minute.

a. During pregnancy, heart rate increases by 10 to 15 beats per minute between 14 and 20 weeks of gestation, and this elevation persists to term. Therefore, a prepregnancy heart rate of 72 would increase by 10 to 15 beats per minute to a rate of 82 to 87 beats per minute.

A young couple are very excited to discover they are pregnant and ask the nurse when to expect the baby. Based on a July 20 LMP, which day will the nurse predict for delivery? a. April 27 b. March 13 c. April 13 d. May 20

a. Naegele rule is to subtract 3 months and add 7 days from the first day of the last menstrual period to determine an expected due date, making the client's due date April 27.

A couple who is in for fertility testing ask the nurse what tests are commonly performed to assess fertility. The nurse replies that there are only three primary tests that are used. What are these tests? a. semen analysis, ovulation monitoring, and tubal patency assessment b. semen analysis, urinalysis, and ovulation monitoring c. serologic test for syphilis, semen analysis, and tubal patency assessment d. pelvic sonogram, ovulation monitoring, and semen analysis

a. Only three tests are commonly used to test fertility: semen analysis in the male and ovulation monitoring and tubal patency assessment in the female. Additional testing may be performed both on the man and the woman, which may include urinalysis, serologic test for syphilis, and pelvic sonogram, but these are not the primary tests used.

An older couple approaches the nurse's station seeking information about their hospitalized grandchild. What should be the nurse's response? a. Tell the couple to seek information directly from the child's parents. b. Answer the grandparents' questions in a less public place. c. Ask to see identification from the two individuals. d. Assume "busy-looking" behaviors and move away.

a. To protect the child's privacy, information can be shared only with legal guardians, parents, or those identified in writing. "Looking busy" only temporarily avoids confronting the privacy rights.

A woman calls the prenatal clinic and says that she thinks she might be in labor. She shares her symptoms over the phone with the nurse and asks what to do. The nurse determines that she is likely in true labor and that she should head to the hospital. Which symptom is an indicator of true labor? a. contractions beginning in the back and sweeping forward across the abdomen b. lightening (descent of the fetus into the pelvis) c. intermittent backache stronger than usual d. increase in fetal kick count

a. True labor contractions usually begin in the back and sweep forward across the abdomen similar to tightening of a rubber band. They gradually increase in frequency and intensity over a period of hours. Lightening and intermittent backache are preliminary signs of labor but do not indicate true labor. Increase in fetal kick count does not indicate true labor.

The nurse is reviewing prenatal charts in the clinic and notes some clients report infections during their pregnancies. Which maternal infection(s) places the fetus at high risk for developmental abnormalities? Select all that apply. a. urinary tract infections b. rubella c. sinus infections d. varicella e. Zika virus

b, d, e. The Zika virus, varicella, and rubella are known as infectious teratogens. A urinary tract infection and a sinus infection would likely not alone cause fetal abnormalities.

A woman comes to the clinic for an exam and says that she is considering trying to become pregnant in the next few months. What would the nurse encourage the client to begin taking now? a. iron b. folic acid c. calcium d. magnesium

b. All women of childbearing age are recommended to consume 400 micrograms of folic acid daily to help prevent neural tube defects. Iron supplementation is used during pregnancy to maintain body stores and minimize the occurrence of iron deficiency anemia. A client's balanced diet should provide an adequate supply of vitamins and minerals for pregnancy.

During an exam, the nurse notes that the blood pressure of a client at 22 weeks' gestation is lower, and her heart rate is 12 beats per minute higher than at her last visit. How should the nurse interpret these findings? a. The heart rate increase may indicate that the client is experiencing cardiac overload. b. The blood pressure should be higher since the cardiac volume is increased. c. Both findings are normal at this point of the pregnancy. d. Combined, both of these findings are very concerning and warrant further investigation.

c. A pregnant woman will normally experience a decrease in her blood pressure during the second trimester. An increase in the heart rate of 10 to 15 beats per minute on average is also normal, due to the increased blood volume and increased workload of other organ systems. Hormonal changes cause the blood vessels to dilate, leading to a lowering of blood pressure.

A nurse is performing a physical assessment on a normal male client. Which finding would be considered abnormal in the exam? a. Rugae are noted on the scrotum. b. Absence of the prepuce c. Testes measure 6 cm long by 4 cm wide. d. The epididymis is palpable on the posterior portion of the testes.

c. The normal size of adult male testes are 4 cm x 2.5 cm, so testicular measurements of 6 cm x 4 cm would be considered enlarged and would warrant further diagnostics. Rugae, which are grooves on the scrotal tissue, are normal findings, as is a palpable epididymis and the absence of the prepuce or foreskin if the client has been circumcised.

A nurse is teaching a female client who is unable to conceive how to monitor her basal body temperature. Which instruction would the nurse prioritize for this client? a. Record body temperature every night b. Record menses and time of intercourse c. Chart body temperature for at least a month d. Record body weight along with the temperature

c. The nurse should instruct the client to record her temperature each morning immediately upon awakening (at rest) using the same thermometer. The nurse should also instruct the client to record her menses, any events that would alter her temperature (e.g., infections, insomnia), and when she has intercourse. To maintain a basal body temperature chart, the client has to monitor her basal body temperature for several months. It is not necessary to record body weight along with the temperature.

A nurse caring for a 43-year-old client explains to the client that ovarian functions gradually decrease around this age. What is the implication of decreased ovarian function? a. susceptibility to fractures b. increases the risk of heart disease c. changes in sexual response d. inability to become pregnant

d. Because of the decrease in ovarian functions, the client loses the ability to become pregnant. Loss of estrogen makes the client susceptible to fractures and also increases the risk of heart disease. Changes in sexual response are usually related to physical changes in the vagina.

A client asks the nurse about cervical mucus changes that occur during the menstrual cycle. Which statement should the nurse expect to include in the client's teaching plan? a. Cervical mucus disappears immediately after ovulation, resuming with menses. b. About midway through the menstrual cycle, cervical mucus is clear and sticky. c. During ovulation, the cervix remains dry with scant mucus secretion. d. As ovulation approaches, cervical mucus is abundant and stretchable.

d. During ovulation, the cervix produces thin, clear, stretchy, slippery mucus that is designed to capture the man's sperm, nourish it, and help the sperm travel up through the cervix to meet the ovum for fertilization.

A pregnant client is undergoing a fetal biophysical profile. Which parameter of the profile helps measure long-term adequacy of the placental function? a. amniotic fluid volume b. fetal heart rate c. fetal breathing record d. fetal reactivity

a. A biophysical profile combines five parameters (fetal reactivity, fetal breathing movements, fetal body movement, fetal tone, and amniotic fluid volume) into one assessment. The fetal heart and breathing record measures short-term central nervous system function; the amniotic fluid volume helps measure long-term adequacy of placental function.

A nurse is preparing a teaching plan for a woman who is planning to become pregnant. Which hormone secreted by the corpus luteum would the nurse include in the discussion when describing how the endometrium is prepped for egg implantation? a. progesterone b. prolactin c. testosterone d. luteinizing hormone

a. After the follicle ruptures as it releases the egg, it closes and forms a corpus luteum. The corpus luteum secretes increasing amounts of progesterone and estrogen, which interact with the endometrium to prepare it for implantation. The corpus luteum does not secrete prolactin, testosterone, or luteinizing hormone.

A male client visits a fertility clinic after one year of attempting unsuccessfully to impregnate his wife. What is a risk factor associated with male infertility? a. no or few sperm cells produced b. 2 to 5 ml of semen ejaculated c. sperm density about 20 million/ml d. morphology of sperm within normal limits

a. Azoospermia or oligospermia (no or few sperm cells produced) is a risk factor affecting male infertility. Normally, the volume of semen ejaculated needs to be about 2-5 ml, with a density of sperm at about 20 million/ml. In addition to the number of sperm, the motility (movement), viability, and morphology (shape) of the sperm must be within normal limits.

A nurse is explaining to a pregnant client about the various events associated with pregnancy and its progression. Which placental hormone would the nurse identify as playing a key role in stimulating myometrial contractility? a. estrogen b. progesterone c. relaxin d. human chorionic gonadotropin

a. Estrogen causes enlargement of a woman's breasts, uterus, and external genitalia and stimulates myometrial contractility. Progesterone maintains the endometrium, decreases uterine contractility, stimulates maternal metabolism and breast development, and provides nourishment for the early conceptus. Relaxin acts synergistically with progesterone to maintain pregnancy, causes relaxation of the pelvic ligaments, and softens the cervix in preparation for birth. Human chorionic gonadotropin preserves the corpus luteum and its progesterone production so that the endometrial lining is maintained. It is the basis for pregnancy tests.

A nurse is teaching personal hygiene care techniques to a client with genital herpes. Which statement by the client indicates the teaching has been effective? a. "I will wear loose cotton underwear." b. "I will apply a water-based lubricant to my lesions." c. "I should rub rather than scratch in response to itching." d. "I can pour hydrogen peroxide and water over my lesions."

a. Wearing loose cotton underwear promotes drying and helps avoid irritation of the lesions. The use of lubricants is contraindicated because they can prolong healing time and increase the risk of secondary infection. Lesions should not be rubbed or scratched because of the risk of tissue damage and additional infection. Cool, wet compresses can be used to soothe the itch. The use of hydrogen peroxide and water on lesions is not recommended.

A nurse is assessing a 45-year-old client. The client asks for information regarding the changes that are most likely to occur with menopause. Which should the nurse tell the client? a. The uterus tilts backward. b. The uterus shrinks and gradually atrophies. c. Cervical muscle content increases. d. The outer layer of the cervix becomes rough.

b. After menopause, the uterus shrinks and gradually atrophies. A full bladder, not menopause, causes the uterus to tilt backward. Cervical muscle content does not increase during menopause. Menopause has no significant effect on the outer layer of the cervix.

The nurse discovers a new prescription for Rho(D) immune globulin for a client who is about to undergo a diagnostic procedure. The nurse will administer the Rho(D) immune globulin after which procedure? a. contraction stress test b. amniocentesis c. nonstress test d. biophysical profile

b. Amniocentesis is an invasive procedure whereby a needle is inserted into the amniotic sac to obtain a small amount of fluid. This places the pregnancy at risk for a woman with Rh(D)-negative blood, since the puncture can allow the seepage of blood and amniotic fluid into the woman's system. She should receive Rho(D) immune globulin after the procedure to protect her and future babies. The CST, NST, and a biophysical profile are noninvasive tests.

The nurse is preparing to teach a client how to conduct the basal body temperature method to determine her fertile window. Which instruction should the nurse prioritize? a. It depends on the average temperature taken each morning. b. Temperature should be taken prior to any activity every morning. c. Avoid intercourse only on the days the temperature drops. d. It is the best method for predicting ovulation.

b. BBT is the basal body temperature method and requires the woman to take her temperature and record it every morning. This should be the first activity of the day before exiting the bed or doing other activities. To prevent conception, avoid unprotected intercourse from the day the BBT drops through the fourth day of temperature elevation. The BBT alone is not a reliable method for predicting ovulation. Use BBT along with calendar or cervical mucus methods to increase effectiveness.

A baby is born with what the primary care provider believes is a diagnosis of trisomy 21. This means that the infant has three number 21 chromosomes. What factor describes this genetic change? a. The mother also has genetic mutation of chromosome 21. b. The client has a nondisjunction occurring during meiosis. c. During meiosis, a reduction of chromosomes resulted in 23. d. The client will have a single X chromosome and infertility.

b. During meiosis, a pair of chromosomes may fail to separate completely, creating a sperm or oocyte that contains either two copies or no copy of a particular chromosome. This sporadic event, called nondisjunction, can lead to trisomy. Down syndrome is an example of trisomy. The mother does not have a mutation of chromosome 21, which is indicated in the question. Also, trisomy does not produce a single X chromosome and infertility. Genes are packaged and arranged in a linear order within chromosomes, which are located in the cell nucleus. In humans, 46 chromosomes occur in pairs in all body cells except oocytes and sperm, which contain only 23 chromosomes.

The nurse is caring for a client at 8 weeks' gestation who states, "I did not plan for this right now and I am not happy or excited about this pregnancy. I am not sure what to do." Which response by the nurse is best? a. "You will become excited and happy when you feel the baby move." b. "Many women feel this way during the first trimester." c. "We can refer you to a clinic for potential termination if you desire." d. "Do not worry. Once you hold this baby, everything will be fine."

b. The best response is to let the client know this is a common feeling among all pregnant women. Most women experience ambivalence during the first trimester whether the pregnancy was planned or not. Acceptance of the pregnancy commonly occurs during the second trimester when quickening, or feeling the baby move, occurs. However, it is not appropriate for the nurse to assume the client will become excited as each pregnancy is unique and a time of dramatic alterations. Stating not to worry and everything will be fine is nontherapeutic communication and does not focus on the client's concern. The nurse would discuss the client's feelings and concerns before making a referral.

The nurse is assessing a primipara's fundal height at 36 weeks' gestation and notes the fundus is now located at the xiphoid process of the sternum. The client asks if this is normal. Which response to the client would be best? a. "By this time, the fundus should drop down lower because the baby is moving towards the pelvic inlet." b. "At 36 weeks' gestation, the fundus is in the normal expected location." c. "To be honest, the fundus should be lower since you have gained minimal weight." d. "Just get prepared, the fundus might actually get a little higher until a few days before you go into labor."

b. The fundus grows to reach the umbilicus at 20 to 22 weeks and the xiphoid process of the sternum at 36 weeks. Therefore, this fundus is in the normal, expected location. After 36 weeks' gestation, lightening occurs and the fundus will drop ~4 cm below the xiphoid process. Once the fundus reaches the xiphoid process, it cannot go higher without severely compromising maternal respiratory efforts.

A client diagnosed with human papillomavirus (HPV) asks the nurse if she will be at risk for developing cervical cancer. The nurse best responds by making which statement? a. "Cervical cancer is typically caused by the herpes simplex virus, not the human papillomavirus (HPV)." b. "Certain strains of the human papillomavirus (HPV) have been associated with causing cervical cancer." c. "If you get the human papillomavirus (HPV) vaccine you will no longer be at risk for developing cervical cancer." d. "You will likely contract cervical cancer so frequent screening testing will be very important."

b. The human papillomavirus (HPV) is associated with causing both genital warts and cervical cancer. There is not enough information to definitively indicate that this client will or will not contract cancer. The herpes simplex virus is associated with shingles and genital herpes, not cervical cancer. The HPV vaccine prevents certain strains of HPV, but does not decrease the risk for cervical cancer if the person has already been exposed to the virus.

The nurse is educating a pregnant woman who is planning to breast feed her newborn about the events associated with it. Which hormone would the nurse explain as being reponsible for the let-down reflex? a. Progesterone b. Oxytocin c. Testosterone d. Estrogen

b. The nipple, on stimulation, transmits sensations to the posterior pituitary gland to release oxytocin, which then acts to constrict milk glands and push milk forward into the ducts that lead to the nipple (a let-down reflex). Estrogen is responsible for the increase in size of the breasts at puberty. Progesterone and testosterone are not involved with the breasts or the let-down reflex.

A woman is diagnosed with primary dysmenorrhea and is prescribed ibuprofen as part of her treatment plan. When teaching the woman about using this medication, which instruction would be important for the nurse to emphasize? a. "Take the medication on an empty stomach." b. "Start taking the medication when you first get your period." c. "Add an aspirin to the regimen if you do not get relief right away." d. "It is normal for your bowel movements to be black."

b. When taking a nonsteroidal anti-inflammatory drug (NSAID), such as ibuprofen, for dysmenorrhea, it is important for the woman to start therapy prophylactically and use sufficient doses to maximally suppress prostaglandin production. NSAIDs should be taken with food to prevent gastrointestinal upset. They should not be taken with aspirin because doing so can increase the risk of bleeding, which would be noted with black stools.

The nurse is assessing a client's risk for sexually transmitted infections. Which statement by the client would be cause for concern? a. "I am not sure if I want to keep the baby. It is a hard decision." b. "I am unsure who the father of the baby is. I will be raising it alone." c. "I needed Rho(D) immune globulin after my last pregnancy. Will I need it again?" d. "I only want my family to see the baby after it is born."

b. While many individuals have complex social issues, if a client states that she is unsure of the father of the baby, it is understood that she has had recent, multiple sex partners. Sex with multiple partners places the client and fetus at risk for a sexually transmitted infection. Not wanting to keep the baby, needing Rho(D) immune globulin, and having social issues does not place the client at risk for sexually transmitted infections.

A pregnant client and her husband have had a session with a genetic specialist. What is the role of the nurse after the client has seen a specialist? a. Identify the best decision to be taken for the client. b. Refer the client to another specialist for a second opinion. c. Review what has been discussed with the specialist. d. Refer the client for further diagnostic and screening tests.

c. After the client has seen the specialist, the nurse should review what the specialist has discussed with the family and clarify any doubts the couple may have. The nurse should never make the decision for the client but rather should present all the relevant information and aid the couple in making an informed decision. There is no need for the nurse to refer the client to another specialist or for further diagnostic and screening tests unless instructed to do so by the specialist.

An 18-year-old pregnant woman asks the nurse why she has to have a routine alpha-fetoprotein serum level drawn. The nurse explains that this: a. is a screening test for placental function. b. tests the ability of her heart to accommodate the pregnancy. c. may reveal chromosomal abnormalities. d. measures the fetal liver function.

c. An alpha-fetoprotein analysis is a cost-effective screening test to detect chromosomal and open-body-cavity disorders.

A pregnant woman asks the nurse about medications taken during pregnancy and if they cross the placental barrier. What response by the nurse is appropriate? a. "Yes, all medications cross the placental barrier." b. "Medications taken during the first trimester are typically safe." c. "Some medications cross the placental barrier, so be sure to discuss medications with your provider." d. "Medications taken orally typically do not cross the placental barrier because of the gastric absorption."

c. Some medications cross the placental barrier, so the nurse will encourage the woman to talk more specifically with her provider. The nurse would be in error to state that all or none cross the placental barrier. Regardless of the route, some medications cross the barrier and are unsafe during pregnancy. The first trimester is the most dangerous time to take a medication that crosses the placental barrier.

A nurse is obtaining the genetic history of a pregnant client by eliciting historical information about her family members. Which question is most appropriate for the nurse to ask? a. "Were there any instances of premature birth in the family?" b. "Is there a family history of alcohol or substance use disorder?" c. "What was the cause and age of death for deceased family members?" d. "Were there any instances of depression during pregnancy?"

c. The nurse should find out the age and cause of death for deceased family members, as it will help establish a genetic pattern. Although inquiry of a history of premature birth or depression during pregnancy are important and should be included in the data collection, they do not relate to genetically inherited disorders. A family history of alcohol or substance use disorder does not increase the risk of genetic disorders.

A nurse is preparing a client for intrauterine device (IUD) insertion. Which education will the nurse provide to the client? a. "An IUD will prevent sexually transmitted infections (STIs)." b. "The insertion procedure for an IUD is painless." c. "Checking the strings is recommended following insertion." d. "Expect to have continuous uterine cramps after insertion."

c. The nurse should instruct the client to check the strings/threads regularly to make sure that the IUD is in place. Current recommendations are to check the strings monthly after insertion, but some clients may be instructed to check less frequently after the first 3 months following insertion because expulsion of the IUD is less common after that time. An IUD does not protect against STIs, and the insertion procedure may involve mild to moderate cramping pain; however, the client should not expect to have continuous cramping pain following insertion.

A client with a family history of cervical cancer is to undergo a Papanicolaou test. During the client education, what group should the nurse include as at risk for cervical cancer? a. clients with irregular menstrual cycles b. clients who have not had babies c. clients who have genital warts d. clients with fibrocystic breast disease

c. The presence of genital warts (condyloma) increases the risk of developing cervical cancer. Women with metrorrhagia or irregular menstrual cycles are at an increased risk of developing breast cancer, not cervical cancer. Clients who have never had a baby or those with a history of fibrocystic breast disease have an increased risk of developing breast cancer, but not cervical cancer.

Why is fertility testing initiated with sperm analysis of the male partner? a. Asking the male partner to undergo diagnostic procedures is the best way to assess his interest in treatment of subfertility. b. The man is found to be fertile in over 75% of couples with subfertility problems. c. Male fertility testing is time-consuming and, therefore, should be initiated early. d. Sperm analysis is one of the easiest tests to complete.

d. Because sperm analysis is noninvasive, it is one of the easiest tests (and therefore most cost-effective) to complete.

The nurse assesses a 20-week gestational client at a routine prenatal visit. What will the nurse predict the fundal height to be on this client experiencing an uneventful pregnancy? a. 24 cm b. 12 cm c. 16 cm d. 20 cm

d. Between weeks 18 and 32 the fundal height in centimeters should match the gestational age of the pregnancy. At 20 weeks' the fundal height should be at the umbilicus. A fundal height smaller than expected can indicate that the original dates were miscalculated, oligohydramnios, or that the fetus is smaller than expected. If the fundal height is larger than expected this can indicate multiple gestation, the original dates were miscalculated, polyhydramnios, or a molar pregnancy.

Which hormone is called the hormone of pregnancy because it reduces uterine contractions during pregnancy? a. luteinizing hormone (LH) b. estrogen c. follicle-stimulating hormone (FSH) d. progesterone

d. Progesterone is called the hormone of pregnancy because it reduces uterine contractions, thus producing a calming effect on the uterus, allowing pregnancy to be maintained. FSH is primarily responsible for the maturation of the ovarian follicle. LH is required for both the final maturation of preovulatory follicles and the luteinization of the ruptured follicle. Estrogen is crucial for the development and maturation of the follicle.

Which genetic condition is caused by a small gene mutation that affects protein structure, producing hemoglobin S? a. Marfan syndrome b. hemophilia c. Tay-Sachs disease d. sickle cell anemia

d. Sickle cell anemia is an example of a genetic condition caused by a small gene mutation that affects protein structure, producing hemoglobin S. Marfan syndrome is an autosomal dominant inherited condition. Hemophilia is an X-linked recessive inherited condition. Tay-Sachs disease is an autosomal recessive inherited condition.

Which instruction should be given to a woman newly diagnosed with genital herpes? a. Obtain a Papanicolaou test every 3 years. b. Have your partner use a condom when lesions are present. c. Use a water-soluble lubricant for relief of pruritus. d. Limit stress and emotional upset as much as possible.

d. Stress, anxiety, and emotional upset seem to predispose a client to recurrent outbreaks of genital herpes. Sexual intercourse should be avoided during outbreaks, and a condom should be used between outbreaks; it is not known whether the virus can be transmitted at this time. During an outbreak, creams and lubricants should be avoided because they may prolong healing. Because a relationship has been found between genital herpes and cervical cancer, a Papanicolaou test is recommended every year.

A pregnant client is undergoing an amniocentesis in her third trimester and is worried why she is undergoing this procedure. Which statement by the nurse would best alleviate this client's anxieties? a. "This procedure is not very dangerous and you should not worry about it." b. "We are concerned that your baby may have some genetic disorder." c. "The doctor can explain everything to you when we are done with the procedure." d. "We want to be sure your baby's lungs are mature and this measures the development of the baby's lungs.

d. The purpose of an amniocentesis is to determine if the lungs of the fetus are mature enough to support respirations outside the womb. Additionally, an amniocentesis is done for genetic testing for a variety of disorders. Telling the client that there might be a genetic disorder would only escalate the mother's concerns, not alleviate them. Telling the client the procedure is not dangerous or that the doctor will explain things after the amniocentesis does not address her concerns; rather, it minimizes them.

A client in her second trimester of pregnancy reports discomfort during sexual activity. Which instruction should a nurse provide? a. Perform frequent douching, and use lubricants. b. Modify sexual positions to increase comfort. c. Restrict contact to alternative, noncoital modes of sexual expression. d. Perform stress-relieving and relaxing exercises.

b. The nurse should instruct the client to change sexual positions to increase comfort as the pregnancy progresses. Although the nurse should also encourage her to engage in alternative, noncoital modes of sexual expression, such as cuddling, caressing, and holding, the client need not restrict herself to such alternatives. It is not advisable to perform frequent douching, because this is believed to irritate the vaginal mucosa and predispose the client to infection. Using lubricants or performing stress-relieving and relaxation exercises will not alleviate discomfort during sexual activity.

A nurse is working as part of a larger community group to develop programs to address current barriers to health care being experienced by women. The community is in the city with a large population of low-income families. On which factor should the group focus their efforts to address a main barrier to access to care? a. health insurance b. transportation c. language d. low health literacy

a. Although transportation, language and culture, and low health literacy are barriers to health care, health insurance is a major factor affecting access to health care. The existence of financial barriers is one of the most important factors that limits access to care. Many families do not have health insurance so they cannot afford to see health care providers for maintenance and prevention services. Many do not have enough health insurance to cover services they need or cannot pay for services.

A client is about 16 weeks' pregnant and is concerned because she feels her "abdomen" contracting. She calls the primary care provider's office and speaks to the nurse. What is the nurse's most appropriate response to this client's concern? a. "What you are feeling are called Braxton Hicks contractions. They are considered practice contractions during pregnancy." b. "You need to go to the emergency room right away." c. "You need to come to the office to be examined." d. "You have nothing to be concerned about. I am sure you are not feeling contractions at this point in your pregnancy."

a. Braxton Hicks contractions are the painless, intermittent, "practice" contractions of pregnancy.

A pregnant client asks the nurse if the fetus can be tested for Down syndrome. Which statement by the nurse is most appropriate? a. "Yes, a chromosomal analysis can be done to assess for Down syndrome." b. "Of course. The primary health care provider can prescribe a DNA analysis for this." c. "You need to be sure you want to know this information before you think about being tested." d. "When you have ultrasounds done, they look for signs of Down syndrome."

a. Chromosomal analysis is part of the genetic testing for Down syndrome. The nurse would state this for the client to directly answer the client's question. A DNA analysis may be used in the detection of genetic diseases. The nurse should not avoid answering the client's question. While features of Down syndrome may be seen on ultrasound, the client asked if testing could be completed and the nurse should address this question.

During a preconception visit, the nurse teaches the client about the risk of inadequate nutrition in early pregnancy. Which statement by the client indicates understanding of the nurse's teaching? a. "If I have inadequate nutrients in the first half of pregnancy, an inadequate number of fetal cells are formed." b. "If I have inadequate nutrients in the first half of pregnancy, the fetal cells are unable to grow to their full potential." c. "If I have inadequate nutrients in the last half of pregnancy, the blood flow to the placenta will be decreased." d. "If I have inadequate nutrients in the last half of pregnancy, my fetus will develop cellular hyperplasia."

a. Early in pregnancy, fetal growth occurs largely by an increase in the number of cells formed (hyperplasia); late in pregnancy, it occurs mainly by enlargement of existing cells (hypertrophy). This means a fetus deprived of adequate nutrition early in pregnancy could be small for gestational age because of an inadequate number of cells formed in the body. Later on, although the number of cells may be normal, restricted growth can occur because cells cannot grow to their full potential. Blood flow to the placenta is related to cardiovascular status, not nutrients. Cellular hyperplasia is seen when maternal blood glucose levels are high and the fetus is producing excessive levels of insulin.

The public health nurse is teaching a community class of couples on fertility awareness-based methods. The nurse determines that additional teaching is needed when one of the couples states that they will be using which method? a. coitus interruptus method b. cervical mucus ovulation method c. basal body temperature method d. symptothermal method

a. Fertility awareness refers to any natural contraceptive method that does not require hormones, pharmaceutical compounds, physical barriers, or surgery to prevent pregnancy. Techniques used to determine fertility include the cervical mucus ovulation method, the basal body temperature (BBT) method, the symptothermal method, standard days method, and 2-day method. Coitus interruptus or withdrawal is not considered a fertility awareness based method.

Which assessment finding in the pregnant woman at 12 weeks' gestation should the nurse find most concerning? The inability to: a. detect fetal heart sounds with a Doppler. b. feel fetal movements. c. hear the fetal heartbeat with a stethoscope. d. palpate the fetal outline.

a. Fetal heart sounds are audible with a Doppler at 10 to 12 weeks' gestation but cannot be heard through a stethoscope until 18 to 20 weeks' gestation. Fetal movements can be felt by a woman as early as 16 weeks of pregnancy and felt by the examiner around 20 weeks' gestation. The fetal outline is also palpable around 20 weeks' gestation.

A woman is going to have in vitro fertilization. When preparing her for this, the nurse would make which statement? a. "It can be done with frozen donor sperm." b. "You will need to select a surrogate mother." c. "Most procedures are effective the first time tried." d. "This is dangerous if there is ovarian cancer in your family."

a. Fresh or frozen sperm may be used. The success of in vitro fertilization is not related to the incidence of ovarian cancer in the family. Quite often, more than one attempt is needed before successful implantation.

A nurse working in the newborn nursery hears an innocent murmur on auscultation of a 24-hour-old infant's chest. The nurse recognizes this as most likely the result of which condition? a. delayed fetal shunt closure b. dysfunctional foramen ovale c. congenital defect d. attached umbilical cord stump

a. Functional closure of all fetal shunts usually occurs anywhere from the first hour to three to four weeks after birth. These delayed fetal shunt closures are usually not associated with a heart lesion. If they are still present at a later date, evaluation may be warranted.

A mother brings her 10-year-old daughter to the clinic for a routine visit. The mother tells the nurse that she has noticed her daughter beginning to develop and asks the nurse about when her daughter may begin menstruating. The nurse understands that many factors are involved in determining the age at which menstruation begins. Which factor would the nurse include in the response as most important? a. genetics b. nutrition c. weight d. cultural practices

a. Genetics is the most important factor in determining the age at which menarche starts, but geographic location, nutrition, weight, general health, nutrition, cultural and social practices, the girl's educational level, attitude, family environment, and beliefs are also important.

A nurse is providing home care to a pregnant woman who is on bed rest. The woman has two other children. During her assessment, the nurse asks the woman how she occupies her time. What is the best rationale for asking this question? a. to ensure that the woman is not engaging in activities that would disrupt her rest b. to learn about the client's hobbies c. to pick up tips to pass on to other clients who are on bed rest d. to build rapport with the client

a. If bed rest is required, ask how the client occupies her time. A woman is not really resting if she is concerned about her family or finances, is caring for older children, or is so bored that she is frequently turning or sitting up. The other answers are legitimate reasons for asking the question but are not the best rationale for it.

A nurse is caring for a pregnant client in her second trimester of pregnancy. The nurse educates the client to look for which danger sign of pregnancy needing immediate attention by the primary care provider? a. vaginal bleeding b. painful urination c. severe, persistent vomiting d. lower abdominal and shoulder pain

a. In a client's second trimester of pregnancy, the nurse should educate the client to look for vaginal bleeding as a danger sign of pregnancy needing immediate attention from the primary care provider. Generally, painful urination, severe/persistent vomiting, and lower abdominal and shoulder pain are the danger signs that the client has to monitor for during the first trimester of pregnancy.

During a routine health visit for an 11-year-old girl, her mother asks the nurse, "My daughter just got her period about 4 months ago, but they haven't been very regular so far. How long might it take until she gets regular?" Which response by the nurse would be most appropriate? a. "It can take up to 2 years once she starts for the periods to become regular." b. "She should start getting regular after about 2 to 3 more months." c. "That's odd. Her periods should be getting regular by now." d. "If her periods are not regular now, odds are they will be irregular from now on."

a. Once menarche has occurred, cycles may take up to 2 years to become regular, ovulatory cycles. Telling the mother that her daughter's periods would get regular in 2 to 3 months or that she should be having regular periods by now is incorrect. Also, telling the mother that her daughter's periods will continue to be irregular is untrue and inappropriate.

A pregnant woman at her first prenatal visit asks the nurse if it is safe to have sex during her pregnancy. Which client statement alerts the nurse to the need for further teaching? a. "I should substitute intercourse with nonsexual touch to avoid harming the fetus." b. "I will experience a heightened need for touch throughout my pregnancy." c. "If I experience bleeding, I will abstain from vaginal intercourse." d. "I will avoid having intercourse following the rupture of the membranes."

a. Sexual needs may be met through sexual intercourse with a partner as long as the pregnancy is healthy and there are no other risk factors, such as bleeding or rupture of membranes. Pregnancy is a time of a heightened need for touch, which may be met partially by sexual expression, but which can also be met through nonsexual touch, such as massage, caressing, or holding.

The nurse cares for a pregnant client at the first prenatal visit and reviews expected changes that will occur during pregnancy. Which information will the nurse include in the education? a. During pregnancy blood volume can increase by at least 40%. b. Pregnancy typically causes a decrease in respiratory rate. c. Hemoglobin levels rise significantly during pregnancy. d. Blood pressure decreases in the third trimester.

a. The pregnant woman can experience a blood volume increase by approximately 40% to 50% above prepregnancy levels by the end of the third trimester. Pregnancy results in an increased respiratory rate to provide oxygen to both the mother and fetus. Hemoglobin levels are usually low during pregnancy because of hemodilution of red blood cells, which is termed physiologic anemia of pregnancy. Blood pressure usually reaches a low point mid-pregnancy and, thereafter, increases to prepregnancy levels by the third trimester.

The nurse is teaching a pregnant teenager the importance of proper nutrition and adequate weight gain throughout the pregnancy. What is the best response when the client refuses to eat due to fear of possible weight gain? a. The infant will be small and could have problems. b. There may be little impact on the infant, but the mother can suffer complications. c. It will just make the baby smaller, but there are no other problems associated. d. The infant will be smaller but should quickly gain weight.

a. Women who gain less than 16 pounds (7257 g) are at risk of giving birth to small infants, which is associated with poor neonatal outcomes. The infant may not quickly gain weight but continue to slowly put on weight.

A client reports that she has multiple sex partners and has a lengthy history of various pelvic infections. She would like to know if there is any temporary contraceptive method that would suit her condition. Which method should the nurse suggest for this client? a. intrauterine device (IUD) b. condoms c. oral contraceptive pills (OCPs) d. tubal ligation

b. Because the client has multiple sex partners, condoms will help offer protection against sexually transmitted infections (STIs) and are best suited for her needs. The client cannot use an IUD because of her history of various pelvic infections. Although OCPs will help the client as a means of contraception, this method is not the best choice for her because it does not offer protection against STIs. Tubal ligation is a sterilization procedure and does not suit the client's purpose.

The nurse is caring for a client requesting oral contraceptive pills (OCPs). She has multiple sexual partners. The nurse recommends condoms to the client, but the client states, "I cannot use condoms because I am allergic to latex." Which response by the nurse is appropriate? a. "I will let the health care provider know you desire oral contraceptive pills." b. "You can still use condoms because they make latex-free condoms." c. "We can have you allergy tested to see if certain condoms may work for you." d. "Do you understand the potential ramifications of having multiple sexual partners?"

b. Because this client has multiple partners, there is concern over the risk for sexually transmitted infections (STIs) as well as pregnancy. Condoms are the only method of contraception that protects against STIs, which is an important concern if the client is not in a monogamous relationship. The nurse would inform the client of latex-free condoms and where to purchase them first. Regardless of whether the client chooses to use latex-free condoms, the client can discuss other options for contraception with the health care provider at that time. There is no need to have the client tested for latex sensitivity. The nurse should educate the client on risks associated with multiple sexual partners; however, using therapeutic language will be most effective.

During a prenatal visit, a pregnant woman tells the nurse, "I have had to buy some new bras because my breasts are getting bigger. I know this is supposed to happen, but what is going on in my body?" Which response by the nurse would be appropriate? a. "Your breasts are growing because fatty tissue is replacing the gland tissue." b. "Your hormones are causing the glands in your breasts to grow." c. "Breast milk is being made already so it will be ready for your newborn." d. "All the good things in breast milk are collecting in your breasts."

b. During pregnancy, placental estrogen and progesterone stimulate the development of the mammary glands. Because of this hormonal activity, the breasts may double in size during pregnancy in preparation for milk production. At the same time, glandular tissue replaces the adipose tissue of the breasts. Following childbirth and the expulsion of the placenta, levels of placental hormones (progesterone and lactogen) fall rapidly, and the action of prolactin (milk-producing hormone) is no longer inhibited. Prolactin stimulates the production of milk within a few days after child birth, but in the interim, dark yellow fluid called colostrum is secreted. The substances in breast milk are not present until breast milk is produced.

A client at 40 weeks' gestation informs the nurse that she is tired of being pregnant. What is the best response from the nurse? a. "Do you need to speak with someone about your feelings?" b. "That is a very normal feeling, especially at this point in pregnancy." c. "Most woman would have asked to be induced by this point. Is that what you want?" d. "Are you getting enough rest? If you don't take time for rest, that is why you might be tired."

b. During the third trimester, the client is preparing for parenthood and is often tired and ready for a break. The woman may feel large and unable to do any normal activities, and may feel ready to have the baby in her arms rather than in her uterus. This is not an abnormal statement, and the provider should not overreact. Deciding to induce labor is something that should be done in consultation with the health care provider and only when it is necessary for the health/safety of the mother or baby.

A nurse is collecting data during an admission assessment of a client who is pregnant with twins. The client has a 4-year-old child who was delivered at 38 weeks' gestation and tells the nurse that she does have a history of spontaneous abortion (miscarriage) within the first trimester. The nurse is correct to document the history as: a. G = 4, T = 2, P = 0, A = 0, L = 1 b. G = 3, T = 1, P = 0, A = 1, L = 1 c. G = 1, T = 1, P = 1, A = 0, L = 1 d. G = 2, T = 0, P = 0, A = 0, L = 1

b. The GTPAL stands for Gravida -- number of pregnancies, which is 3 (current, 4-year-old, and miscarriage); Term -- only one pregnancy thus far carried to term; Preterm deliveries -- 0; Abortions (either elective or miscarriage) -- 1; Living children -- 1. Do not be distracted by the twins. That is still one pregnancy.

The nurse is assessing a pregnant client in her third trimester who is reporting a first-time occurrence of constipation. When asked why this is happening, what is the best response from the nurse? a. There is not enough fiber in your diet. b. The intestines are displaced by the growing fetus. c. This shouldn't be happening. d. hCG is delaying peristalsis.

b. The growing fetus is displacing the intestines and interfering with peristalsis, delaying the passage of fecal matter and resulting in constipation. This is common and expected; however, the client should take measures to prevent hemorrhoids that can occur as the result of the pressure and straining. Progesterone, not hCG, can delay gastric emptying and decrease peristalsis.

A woman is confused after finding out the ultrasound results predict a different due date for the birth of her baby. Which factor should the nurse point out is most likely the reason for the miscalculation of the fetal age? a. an error in math when calculating b. mistaking implantation bleeding for last menstrual period (LMP) c. amount of weight gain of mother in early weeks of pregnancy d. not seeking prenatal care in the beginning

b. The most common cause is implantation bleeding, which can occur as the blastocyst implants itself into the endometrium. This bleeding can be mistaken for a scanty menstrual period and can lead to miscalculation of fetal age by 2 weeks. The other choices might also contribute, especially the math miscalculation, but are not the primary reason.

During which stage of fetal development is exposure to teratogens most damaging? a. pre-embryonic stage b. embryonic stage c. fetal stage d. mitosis stage

b. The most sensitive period of fetal development related to teratogens is during the embryonic period when the different body systems are developing. During the pre-embryonic stage, the fetal stage, and the mitosis stage, the risk of teratogenic exposure is not as influential on the fetus.

A nurse who is conducting sessions on preventing the spread of sexually transmitted infections (STIs) discovers that there is a very high incidence of hepatitis B in the community. Which measure should the nurse take to ensure the prevention of the disease? a. Ensure that the drinking water is disease-free. b. Instruct people to get vaccinated for hepatitis B. c. Educate about risks of injecting drugs. d. Educate teenagers to delay onset of sexual activity.

b. The nurse should instruct all community members to get vaccinated for prevention of hepatitis B. Ensuring that drinking water is disease-free and educating people about the risks involved with injecting drugs may help prevent hepatitis A, not hepatitis B. Delaying the start of sexual activity by teenagers may not protect them from hepatitis B in the long run.

The nurse is conducting an annual examination on a young female who reports her last menses was 2 months ago. The client insists she is not pregnant due to a negative home pregnancy test. Which assessment should the nurse use to assess confirm the pregnancy? a. Chadwick sign b. fetal heartbeat c. positive urine human chorionic gonadotropin (hCG) d. uterine size and shape changes

b. The only positive sign of pregnancy is a sign or symptom that could only be attributable to the fetus; thus, fetal heartbeat can have no other origin. Chadwick sign is a color change in the cervix, vagina, and perineum; these could all be the result of other causes. A positive urine hCG is a probable sign as it can be related to causes other than pregnancy. A change in the size and shape of the uterus can occur due to other causes.

A client calls to cancel an appointment for the first prenatal visit after reporting a home pregnancy test is negative. Which instruction should the nurse prioritize? a. Use a diluted urine specimen. b. Wait until after two missed menstrual periods. c. Keep the appointment. d. Refrain from eating for 4 hours before testing.

c. Although home pregnancy tests are accurate 95% of the time, they may still have false positives or false negatives, and the client needs to seek prenatal care and confirmation from her health care provider. Diluting the urine, waiting to miss a second period, or eating before the test would have no effect. The tests look for hCG, which is not affected.

The nurse is creating an educational pamphlet for pregnant mothers. Which is the best description of fetal development for the nurse to emphasize? a. age in weeks and systems developed b. length, weight, sex c. gestational age, length, weight, and systems developed d. sex and systems developed

c. Client education is a major component of maternal-child nursing. During pregnancy, nurses provide anticipatory guidance to prepare the woman and her significant other for the changes each month brings. Clients most often want to know gestational age in weeks, length, weight, and systems developed; the client is then able to visualize what the fetus looks like

The nursing instructor is teaching a class on the nutritional needs of the pregnant client. The instructor determines the session is successful when the students correctly choose which supplement as being known to prevent up to 70% of CNS birth defects? a. iodine b. zinc c. folic acid d. vitamin A

c. Folic acid is noted to help prevent up to 70% of CNS birth defects; however, the folic acid needs to be in the body prior to the pregnancy to be most effective. Iodine affects thyroid development. Zinc is required for enzyme formation and gene expression. Vitamin A helps develop vision.

The nurse at a health fair is teaching about the various changes of puberty. Which sequence of events will be best for the nurse to present when illustrating pubertal changes in females? a. menarche, breast budding, appearance of pubic hair b. Appearance of pubic hair, menarche, breast budding c. breast budding, appearance of pubic hair, menarche d. appearance of pubic hair, breast budding, menarche

c. Secondary sexual characteristics develop in an orderly sequence with variance in the timing for individuals. Breast budding in the female is usually the first physical sign noted, and occurs between the ages of 10 and 12 years on average. Appearance of pubic hair usually occurs just before menarche, the first menstrual period. From the onset to the start of menarche is typically 2 years.

A client, 6 hours postpartum, has a severe postpartum hemorrhage that the health care providers are unable to control. She succumbed to the hemorrhage in the intensive care unit. The client's death would become part of the: a. neonatal morbidity rate. b. neonatal mortality rate. c. maternal mortality rate. d. maternal morbidity rate.

c. The maternal mortality ratio is the annual number of deaths from any cause related to or aggravated by pregnancy or its management. Morbidity refers to diseases. The infant mortality rate is the number of deaths occurring in the first 12 months of life. Neonatal mortality is reflected in the infant mortality rate.

The nurse is examining a woman who came to the clinic because she thinks she is pregnant. Which data collected by the nurse are presumptive signs of her pregnancy? Select all that apply. a. breast changes b. ultrasound pictures c. fetal heartbeat d. amenorrhea e. hydatidiform mole f. morning sickness

a, d, f. Presumptive signs are possible signs of pregnancy that appear in the first trimester, often only noted subjectively by the mother (e.g., breast changes, amenorrhea, morning sickness). Probable signs are signs that appear in the first and early second trimesters, seen via objective criteria, but can also be indicators of other conditions (e.g., hydatidiform mole). Positive signs affirm that proof exists that there is a developing fetus in any trimester and are objective criteria seen by a trained observer or diagnostic study, (e.g., ultrasound.)

In developing a plan of care for a 5-year-old child with Down syndrome, what should be a priority for the nurse in meeting the expected growth and development outcomes for this child? a. The child is enrolled in kindergarten. b. The child is kept in a day care setting. c. The child stays at home with a parent. d. The child is placed in a home facility.

a. All children are granted the opportunity to learn in the public school system regardless of their intellectual ability. To meet the child's growth and development outcomes, the child should be enrolled in kindergarten so that he or she has an opportunity to learn and play with others of the same age.

A female client states, "I am concerned because when I have sex with my spouse, I have an area near my vagina that is really sensitive to touch." Which action will the nurse complete? a. Provide education to the client on the female anatomy. b. Determine if the client has a sexually transmitted infection (STI). c. Have the client provide a urine specimen for testing. d. Refer the client for psychological counseling.

a. The nurse understands this is a normal, expected experience and would provide education to the client on the clitoris and its function. The clitoris is a small, cylindrical mass of erectile tissue and nerves that is highly sensitive and analogous to the head of the male penis. There is no indication for counseling because no signs of current or past abuse are present. This is not an indication of an STI or urinary tract infection; therefore, no additional testing or specimens are needed.

A nursing instructor is explaining the stages of fetal development to a group of nursing students. The instructor determines the session is successful after the students correctly choose which time period as representing the pre-embryonic stage? a. From fertilization to the end of the second week after fertilization b. Approximately 2 weeks after fertilization to the end of the eighth week c. Approximately 9 weeks after fertilization to birth d. Approximately 6 weeks after fertilization to the end of 8 weeks

a. The pre-embryonic stage begins at fertilization and lasts through the end of the second week after fertilization. The embryonic stage begins approximately 2 weeks after fertilization and ends at the conclusion of the eighth week after fertilization. The fetal stage begins at 9 weeks after fertilization and ends at birth. There is no distinct stage recognized approximately 6 to 8 weeks after fertilization. This is part of the embryonic stage period.

When educating parents on recessive genetic disease statistics, the nurse understands that which statement by the parents indicates an accurate understanding of genetic inheritance? a. "My children will have a 50/50 chance of having a genetic disease." b. "If I have four children, only one will have the disease." c. "With each child the odds of inheritance decrease significantly." d. "Each child will have a 25% chance of developing the disease."

d. The statistics of inheritance are reset with each pregnancy; therefore, each pregnancy has the same statistical probability of displaying the genetic disease.

A nurse is helping a couple choose a contraception method that is right for them. Which is the least likely factor the nurse should consider? a. the popularity of the method among the couple's friends b. ability to use a method correctly c. financial factors d. whether the method will affect sexual enjoyment

a. Important things to consider when helping a couple to choose a method that will be right for them include the following: personal values, ability to use a method correctly, whether the method will affect sexual enjoyment, financial factors, whether a couple's relationship is short-term or long-term, prior experiences with contraception, and future plans. The popularity of the method is the least likely factor the nurse should consider.

The school nurse is preparing a teaching session for a group of adolescent girls explaining the menstrual cycle. The nurse determines the session is successful when the young students correctly choose which fact? a. The average menstrual flow lasts 4 to 6 days. b. The average age for menarche is 11 to 13 years. c. Menarche is the start of puberty. d. Total blood loss each month averages 45 to 80 ml of blood.

a. The average flow lasts 4 to 6 days and is caused by casting away of blood, tissue, and debris from the uterus. The average age for menarche is from 12 to 14 years. Menarche is the end of puberty, which follows breast budding and the appearance of pubic hair. The total blood loss each month is 25 to 60 ml.

A client asks the nurse about how the testes function in reproduction. Which is the nurse's best reply? a. The testes serve as the production center for sperm. b. The testes serve as a reservoir for seminal fluid. c. The hormone estrogen is produced there. d. The testes provide stimulus for sexual activity.

a. The testes serve two purposes: production of testosterone and formation and maturation of sperm. Seminal fluid is located in prostate and seminal vesicles. The testes produce testosterone, not estrogen, which is a female hormone. The testes have no connection to sexual stimulation.

A client presents with vulvar itching and diffuse green vaginal discharge. Upon evaluation, she is prescribed metronidazole. What is the paramount nursing intervention in discharge planning? a. Instruct the client not to drink alcohol with this treatment. b. Counsel the client to refrain from sex for one week. c. Advise the client to take medication with a glass of milk. d. Reassure the client further STI testing is not indicated.

a. While counseling to abstain from sex for one week is appropriate, the most important intervention is counseling to avoid alcohol during metronidazole treatment. Alcohol consumption while taking metronidazole creates a severe gastrointestinal reaction of nausea, vomiting, and flushing. Metronidazole does not have to be taken with milk. Further STI testing is indeed recommended with the diagnosis of an STI.

A pregnant client reports an increase in a thick, whitish vaginal discharge. Which response by the nurse would be most appropriate? a. "You should refrain from any sexual activity." b. "You need to be assessed for a fungal infection." c. "This discharge is normal during pregnancy." d. "Use a local antifungal agent regularly."

c. During pregnancy, vaginal secretions become more acidic, white, and thick. Most women experience an increase in a whitish vaginal discharge, called leukorrhea, during pregnancy. The nurse should inform the client that the vaginal discharge is normal except when it is accompanied by itching and irritation, possibly suggesting Candida albicans infection (a monilial vaginitis), which is a very common occurrence in this glycogen-rich environment. Monilial vaginitis is a benign fungal condition and is treated with local antifungal agents. The client need not refrain from sexual activity when there is an increase in a thick, whitish vaginal discharge.

A 25-year-old woman who underwent genetic testing has learned that she is heterozygous dominant for Huntington disease. Her male partner, however, who also underwent the testing, is free from the trait. What statement by the woman would indicate an understanding of the risk of their child developing Huntington disease? a. "Since my partner does not carry the trait and I do, there is a 50% chance that each of our children will either carry the trait or develop the disease." b. "Because I am heterozygous dominant and my partner is not, each of our children will not develop or be a carrier of the disease." c. "Each of our children has a 75% chance of developing Huntington disease." d. "Since one of us carries the trait and the other does not, there is a 25% chance each of our children will be a carrier of the trait."

a. Huntington disease is inherited in an autosomal dominant manner. This means that a person with the mutation in only one of the two copies of the gene will have the condition. This client is heterozygous dominant. Her male partner has no copies of the mutated gene. Thus, each of the couple's children has a 50% chance to have Huntington disease and a 50% chance to be a carrier of the disease.

The school nurse is teaching a health education class on the male reproductive system. The nurse determines the class is successful when the students correctly point out which structure produces sperm? a. testes b. seminal vesicles c. scrotum d. prostate gland

a. The testes are responsible for sperm production. The seminal vesicles produce nutrient seminal fluid. The scrotum surrounds and protects the testes. The prostate gland and the seminal vesicles produce fluid to nourish the sperm.

At the first prenatal visit, the client reports her last menstrual period (LMP) was November 16. The nurse determines the estimated due date to be: a. August 23 b. August 13 c. August 3 d. September 1

a. There are several methods to determine the estimated date of birth. Naegele rule can be used, which involves subtracting 3 months and then adding 7 days to the first day of the LMP. Then correct the year by adding 1 where necessary. Another method is to add 7 days and then add 9 months and add 1 to the year where needed. Thus the client reports her LMP was November 16 subtract 3 months (August), add 7 days (23), and adjust the year by adding 1 year. This client's estimated date of birth is August 23, in the following year.

The nurse is providing contraception counseling to a perimenopausal woman who has had negative reactions to oral contraceptives (OCs) in the past and would like a long-term, nonhormone-based method that has a high rate of success. Neither she nor her husband wants to undergo surgery, however. Which method should the nurse recommend? a. intrauterine device b. transdermal patch c. subdermal progestin implant d. tubal ligation

a. Women who are perimenopausal are, overall, good candidates for intrauterine devices (IUDs). In this case, the IUD is the best choice because this method is almost 100% effective, is long-term, and does not involve any hormones. Both the transdermal patch and the subdermal progestin implant involve hormones, which rules each out in this case. Also, tubal ligation is a surgical procedure, which also rules it out for this client.

After teaching a class about conception, the nursing instructor asks the attendees to explain the process of fertilization. Which statements indicate the students need additional teaching? Select all that apply. a. The ovum is receptive to conception 12 to 23 hours after ovulation. b. Sperm are viable for only about 48 hours after ejaculation. c. Sperm are able to fertilize the ovum from 3 days before until 2 days after ovulation. d. Sperm are able to fertilize the ovum up to 12 hours after ejaculation, and the ovum remains fertile for at least 24 hours after ovulation. e. This activity usually occurs in the ampulla of the fallopian tube.

b, d. Once the mature ovum is released (i.e., ovulation), fertilization must occur fairly quickly because an ovum is capable of fertilization for only about 24 to 48 hours. After that time, it atrophies and becomes nonfunctional. Because the functional life of a spermatozoon is also about 24 to 72 hours, the total critical time span during which sexual relations must occur for fertilization to be successful is from 3 days (72 hours) before until 2 days (48 hours) after ovulation.

The nurse is caring for a female client who is a victim of intimate partner violence (IPV). The client has experienced traumatic injury to the clitoris. Which statement will the nurse include when providing discharge education to this client? a. "The damage to your clitoris could cause you to experience incontinence in the future." b. "Damage to your clitoris may lead to decreased stimulation in future sexual experiences." c. "The clitoris functions to provide cushioning during intercourse. You might now have increased pain." d. "There is a chance you will experience increased vaginal discharge due to the injury experienced."

b. At the superior junction of the labia, there is a fleshy protrusion of tissue called the clitoris. The clitoris is an erectile tissue that enlarges and becomes sensitive when stimulated by the penis or touching that accompanies sexual foreplay. Depending on the severity of the damage, the clitoris may have decreased ability to function with future sexual experiences. The clitoris does not affect bladder function or provide cushioning during intercourse. The mons pubis is responsible for providing a cushion. Vaginal discharge should not increase. There may be a decrease in vaginal secretions related to sexual arousal if permanent damage to the clitoris occurs due to decreased sexual stimulation.

A nurse is discussing the importance of good nutrition to a young pregnant client. The nurse would point out that the growing fetus is getting nutrition from the mother via which structure? a. decidua b. placenta c. amniotic fluid d. umbilical arteries

b. The placenta is a flat, round structure which forms on the decidua and attaches to the fetus by the umbilical cord. The placenta is the organ responsible for supplying nutrients and oxygenated blood to the fetus. The amniotic fluid surrounds the fetus and provides protection, temperature regulation, allows movement, and symmetric growth. It collects urine and other waste products from the fetus. The decidua is the name given to the endometrium after the pregnancy starts. The umbilical arteries carry waste products away from the fetus to the placenta, where they are filtered out into the maternal body for proper disposal.

A client is being discharged from the gynecological unit after treatment for acute pelvic inflammatory disease (PID). What priority instruction regarding disease management should the nurse include? a. Provide the client with brochures on sexually transmitted infection prevention. b. Arrange for follow-up visits to her health care provider. c. Discuss the necessity of completing the antibiotic therapy. d. Refer the client to the women's sexual health clinic.

c. It is the priority that the client understands the need to finish the antibiotic therapy required to treat PID in order to eradicate the offending bacterial infection. The STI brochures may be appropriate, but they are not the priority and neither is arranging for follow-up health care provider visits. If the client can access the women's health clinic, a referral can be made, but it is not the priority.

The community nurse is preparing a presentation for a health fair illustrating successful pregnancies. Which component should the nurse prioritize as the most critical to ensure a positive psychological experience with the pregnancy by the mother? a. Early prenatal care b. Age at the time of pregnancy c. Having a planned pregnancy d. Social support

d. All options are correct and play a role pregnancy, but the most critical for a positive psychological experience is for the woman to have a social support system. Early care, maternal age, and planned pregnancy all affect fetal and maternal health, but are not necessarily linked to positive psychological experiences.

During an examination, a client at 32 weeks' gestation becomes dizzy, lightheaded, and pale while supine. What should the nurse do first? a. Listen to fetal heart tones. b. Take the client's blood pressure. c. Ask the client to breathe deeply. d. Turn the client on her left side.

d. As the enlarging uterus increases pressure on the inferior vena cava, it compromises venous return, which can cause dizziness, light-headedness, and pallor when the client is supine. The nurse can relieve these symptoms by turning the client on her left side, which relieves pressure on the vena cava and restores venous return. Although they are valuable assessments, fetal heart tone and maternal blood pressure measurements do not correct the problem. Because deep breathing has no effect on venous return, it cannot relieve the client's symptoms.

A gravida woman in her second trimester has shared that she still enjoys a glass of wine about once a week with dinner. What response by the nurse is most appropriate? a. "Now that you have reached the second trimester you are at a reduced risk for causing complications to your fetus." b. "There is no amount of alcohol consumption in pregnancy that is considered safe for the fetus." c. "As long as you do not increase the amount of alcohol you are drinking there is little risk." d. "The best thing for you to do is to reduce the amount of alcohol you are drinking."

b. Alcohol ingestion during the pregnancy is considered unsafe at all points in the pregnancy. Alcohol can impact the fetus during each of trimester of pregnancy. There are no exact amounts of alcohol that can be ingested safely. Alcohol impacts each pregnancy and fetus differently. The best course of action is to share the dangers with the woman.

When describing the role of a doula to a group of pregnant women, the nurse would include which information? a. The doula is a professionally trained nurse hired to provide physical and emotional support. b. The doula can perform any necessary clinical procedures. c. The doula primarily focuses on providing continuous labor support. d. The doula is capable of handling high-risk births and emergencies.

c. Doulas provide the woman with continuous support throughout labor. The doula is a laywoman trained to provide women and families with encouragement, emotional and physical support, and information through late pregnancy, labor, and birth. A doula does not perform any clinical procedures and is not trained to handle high-risk births and emergencies.

A pregnant client reports occasional headaches. She wants to know what she can take to alleviate the discomfort. What would be the best response by the nurse? a. "You don't want to harm the baby by taking medications now, do you?" b. "Wait until you reach your third trimester. You can take something to relieve headaches then." c. "Acetaminophen is considered relatively safe to take for your headaches during your pregnancy." d. "Ibuprofen is considered safe to take for your headaches during your pregnancy."

c. The medication that is approved for the treatment of headaches in pregnant women is acetaminophen. Acetaminophen is considered relatively safe to take during pregnancy.

The nursing instructor is teaching a class on the structures vital to the development of the fetus. The instructor determines the class is successful when the class correctly chooses which facts concerning amniotic fluid? a. produced by fetal brain b. amniotic fluid is 90% water c. production stops in the 8th month d. helps the fetus regulate body temperature

d. Amniotic fluid is formed by the fetal membranes, the amnion and chorion, on a constant basis until birth. It is 98% to 99% water, with the remaining 1% to 2% composed of electrolytes, creatinine, urea, glucose, hormones, fetal cells, lanugo, and vernix. It serves four main functions: physical protection, temperature regulation, provision of unrestricted movement, and symmetrical growth. The fetus is unable to regulate its own body temperature so the amniotic fluid provides this function.

A client is suspected of having herpes simplex viral infection. The nurse would expect to prepare the client for which diagnostic test to confirm the infection? a. viral culture of vesicular fluid b. Papanicolaou smear c. IgG/IgM antibody testing d. treponemal testing

a. Diagnosis of HSV is often based on clinical signs and symptoms and is confirmed by viral culture of fluid from the vesicles. The IgG/IgM antibody testing is frequently done for screening purposes. Papanicolaou (Pap) smears are an insensitive and nonspecific diagnostic test for HSV and should not be relied on for diagnosis. Treponemal testing is used to diagnose syphilis.

A nurse is preparing a presentation for a health fair on the topic of vasectomy. Which information should the nurse point out in the presentation? a. Relatively easy procedure with few complications b. Regular sperm counts are not essential c. Procedure is effective immediately d. Birth control measures are not required

a. The nurse should inform the individuals that vasectomy is a relatively easy procedure with few complications. Regular sperm counts following a vasectomy are important, as it will take approximately a month for the remaining sperm to exit the body. The client should use birth control measures until his sperm count remains at zero for 6 weeks.

The nurse is caring for a client diagnosed with candida vaginitis. The nurse knows additional teaching is needed when the client makes which statement? a. "I should wear cotton underwear to prevent future infections." b. "Douching is an effective way to cleanse the vaginal area after intercourse." c. "Eating yogurt can may help limit future infections." d. "Low blood sugar levels are a way to control vaginal infections."

b. An acidic environment helps protect against ascending infections in the vagina. Douching can alter the pH of the vagina, causing an alkaline environment where candida infections thrive. The client should be cautioned against using douches, hygiene sprays, or deodorants that might upset the acid balance. Cotton underwear promotes vaginal dryness, which will help prevent vaginal infections. Probiotics can help balance vaginal bacteria and limit infections. Foods such as yogurt and fermented foods contain probiotics. Infections also thrive when blood glucose levels are high; therefore, clients with diabetes must maintain balanced levels to promote healing and limit infections.

Nausea and vomiting are common reports during pregnancy. What nutritional action can be used to lessen nausea and vomiting? a. drinking liquids with meals b. limiting intake of heavy, greasy foods c. increasing fluid intake d. limiting carbohydrate intake

b. Nausea and vomiting can be lessened by limiting intake of fatty and greasy foods and eating small frequent meals every 2 to 3 hours. Other interventions include eating carbohydrate foods such as dry crackers, Melba toast, dry cereal, or hard candy before getting out of bed in the morning. Avoid drinking liquids with meals; avoid coffee, tea, and spicy foods; and eliminate individual food intolerances. Drinking liquids, increasing fluid intake, and limiting carbohydrate intake does not lessen nausea and vomiting.

The nurse is explaining the menstrual cycle to a group of older adolescents. Which factor should the nurse point out will lead to menstruation? a. Follicle stimulating hormone by the anterior pituitary b. Decreasing production of estrogen by the follicle c. Decreasing production of progesterone by the corpus luteum d. Another ovum that begins immediately

c. If the ovum is not fertilized, the production of progesterone and estrogen by the corpus luteum begins to decrease. Without the high levels of progesterone and estrogen, the endometrium degenerates and sheds, a process referred to as menstruation. The follicle becomes the corpus luteum after ovulation and continues to secrete estrogen and progesterone. The production of another ovum is not the factor that causes endometrium degeneration. Production of follicle stimulating hormone by the anterior pituitary initiates ovulation.

A mother comes in with her 17-year-old daughter to find out why she has not had a menstrual cycle for a few months. Examination confirms the daughter is pregnant with a fundal height of approximately 24 cm. The nurse interprets this finding as indicating that the daughter is approximately how many weeks pregnant? a. 24 b. 22 c. 20 d. 18

a. By 20 weeks' gestation, the fundus of the uterus is at the level of the umbilicus and measures 20 cm. A monthly measurement of the height of the top of the uterus in centimeters, which corresponds to the number of gestational weeks, is commonly used to date the pregnancy. Therefore for this client, the additional 4 cm would be the equivalent of 4 additional weeks making the gestational age of 24 weeks.

A woman with both heart disease and osteoarthritis has come to the genetics clinic for genetic screening. What would the nurse know about these two diseases? a. They are multifactorial. b. They are a direct result of the client's lifestyle. c. They are caused by a single gene. d. They do not have a genetic basis.

a. Genomic or multifactorial influences involve interactions among several genes (gene-gene interactions) and between genes and the environment (gene-environment interactions), as well as the individual's lifestyle.

What instruction should a nurse offer to a pregnant client or a client who wishes to become pregnant to help her avoid exposure to teratogenic substances? a. Avoid medications. b. Eat a well-balanced diet. c. Maintain personal hygiene. d. Avoid intake of coffee.

a. The nurse should instruct a client who is pregnant or one who wants to conceive to avoid medications and thus avoid exposure to any kind of teratogenic substance. Eating a well-balanced diet and maintaining personal hygiene, though important during pregnancy, will not prevent a client's exposure to teratogenic substances. Coffee is not a teratogenic substance, so the client need not avoid coffee. However, coffee is not recommended during pregnancy because it may increase the risk of spontaneous abortion (miscarriage).

The nurse is reviewing the functions of the ovaries, uterus, clitoris, and vagina with a group of nursing students. What would be the correct response by a nursing student about the function of the ovaries? a. They produce female gametes or ova and secrete female sex hormones. b. They receive the fertilized ovum and provide housing and nourishment for a fetus. c. They receive sperm, provide an exit for menstrual flow, and serve as the birth canal. d. They are a small erectile structure that responds to sexual stimulation.

a. The ovaries' functions are to produce female gametes or ova and secrete female sex hormones. The uterus's functions are to receive the fertilized ovum and provide housing and nourishment for a fetus. The vagina's functions are to receive sperm, provide an exit for menstrual flow, and serve as the birth canal. The clitoris is a small erectile structure that responds to sexual stimulation.

A couple is seeking guidance for their inability to conceive a child after trying for 15 months. They are morbidly obese but state they have friends with the same weight problem who have had no difficulty conceiving. What education can the nurse provide this couple to increase their chances of success? a. "If you have been trying for this long without success, something must be wrong. So we will need to run a series of tests to see what that is." b. "Obesity may interfere with effective penetration and deposition of sperm. We will look at several factors to discover what issues you may be encountering." c. "Being this overweight is unhealthy for you and for any child that you may bring into the world. Before you consider conceiving, you should lose weight." d. "Weight has no bearing on the ability to conceive. Something else must be going on with your reproductive organs."

b. Extreme obesity in a male may interfere with effective penetration and deposition of sperm, compromising the ability to conceive.

While talking with a pregnant woman who has undergone genetic testing, the woman informs the nurse that her baby will be born with Down syndrome. The nurse understands that Down syndrome is an example of a: a. multifactorial inheritance. b. X-linked recessive inheritance. c. trisomy numeric abnormality. d. chromosomal deletion.

c. Down syndrome is an example of a chromosomal abnormality involving the number of chromosomes (trisomy numeric abnormality), in particular chromosome 21, in which the individual has three copies of that chromosome. Multifactorial inheritance gives rise to disorders such as cleft lip, congenital heart disease, neural tube defects, and pyloric stenosis. X-linked recessive inheritance is associated with disorders such as hemophilia. Chromosomal deletion is involved with disorders such as cri-du-chat syndrome.

The nurse is caring for a client experiencing perimenopausal symptoms. The client states intercourse has become painful. Which nursing action is appropriate? a. Tell the client to ask for a referral to see a specialist. b. Determine the client's frequency of intercourse. c. Recommend the client use lubrication during intercourse. d. Request the health care provider assess the client's hormone levels.

c. During sexual activity, the walls of the vagina secrete a thin watery material, necessary for the comfortable placement and movement of the penis in the vagina during intercourse. Decreased lubrication is a symptom of perimenopause, which often results in painful intercourse. The nurse would first recommend the client use lubrication with intercourse before invasive procedures are performed. Hormone levels may be assessed if lubrication is not effective. There is no indication the client needs to see a specialist at this time. Intercourse frequency does not need to be assessed based on the client's symptoms. The findings do not indicate a concern with frequency.

Semen analysis has been ordered for the partner of a client who has been unable to become pregnant. What instructions should the nurse provide to the partner? a. "Collect the sample after two consecutive days of ejaculating." b. "Keep the sample refrigerated to ensure the sperm survive." c. "Bring the sample to the lab immediately after you collect it." d. "If possible, collect the sample first thing in the morning."

c. Semen samples must be delivered to a lab no more than 30 minutes after collection. Storage in the refrigerator would violate this directive. The client should abstain from ejaculating for 3 to 5 days before collecting the sample. The semen does not need to be collected in the morning.

A woman is 10 weeks' pregnant and tells the nurse that this pregnancy was unplanned and she has no real family support. The nurse's most therapeutic response would be to: a. encourage her to identify someone that she can talk to and share the pregnancy experience. b. tell her to move home so her family will be nearby to help her. c. remind her that she is still early in the pregnancy and she will feel better about it as the pregnancy progresses. d. offer to meet with the client on a regular basis to provide her someone to talk to about her concerns.

a. A pregnant woman without social support needs to identify someone with whom she can share the experience of pregnancy because social support is a crucial part of adapting to parenthood. Telling her to move home and telling her that she will feel better as the pregnancy progresses do not address the issue of isolation. Also, moving home may not be a possibility for this woman. The nurse should maintain a professional relationship and not commit to a long-term relationship with a client.

The nurse has assessed several clients who have arrived for routine appointments. The nurse predicts the health care provider will prioritize a bone density scan for which client? a. a 40-year-old black client, 5 ft 4 in (1.62 m) tall, 172 lbs (78 kg), inactive lifestyle b. a 25-year-old Asian client, 5 ft 7 in (1.7 m) tall, 129 lbs (58.5 kg), with two children c. a 55-year-old white client who smokes and has family history of osteoporosis d. a 45-year-old Hispanic client with a vitamin D deficiency

c. Risk factors for osteoporosis include female gender, white or Asian ethnicity, slender build, advanced age, estrogen deficiency because of menopause, low bone mass density, family history of osteoporosis, personal history of fracture as an adult, smoking, excessive alcohol intake, low dietary intake of calcium, vitamin D deficiency, inactive lifestyle, and use of glucocorticoids and anticonvulsants. The more of these risk factors an individual has, the higher the risk for osteoporosis. The 55-year-old white smoker with a family history of osteoporosis should be referred for bone scan or density testing. The other individuals have fewer overall risk factors.

Which statement about time orientation most accurately exemplifies a person who is oriented in the present? a. The person is making plans to attend college in the next 2 years. b. The person is focused on preserving cultural traditions with minimal variation in practice. c. The person places a low priority toward saving money for college. d. The person focuses on what food to buy and prepare for a holiday celebration next month.

c. Some cultures are oriented toward the past: they carefully preserve traditions, allowing only the slightest changes or variations in practices. Still others are oriented toward the present; saving money for college (a future-oriented action) might seem important to an American nurse but would not be a high priority in these cultures. Preparing for an upcoming celebration is a future-oriented focus.

A young male asks the nurse at the clinic about what he can do to maintain his sperm production. The nurse would instruct him to take which action? a. Wear snug fitting pants to support the scrotum. b. Buy only cotton underwear for use. c. Take long, hot baths to encourage sperm production. d. It is recommended that men wear boxer-style underwear.

d. It is recommended that men wear loose fitting pants and underwear in order to keep the testes cooler than body temperature to best support sperm production. Tight pants and hot baths are counterproductive to sperm production; wearing cotton underwear does not affect it.

A female client with genital herpes is prescribed acyclovir as treatment. After teaching the client about this treatment, which statement by the client indicates effective teaching? a. "If I use this drug, I will be cured of the infection." b. "This drug will help reduce my risk for a recurrence after discontinuing it." c. "The severity of future attacks will be much less after using this drug." d. "This drug will help to suppress any symptoms of the infection."

d. No cure exists, but antiviral drug therapy helps to reduce or suppress symptoms, shedding, and recurrent episodes. Advances in treatment with acyclovir 400 mg orally three times daily for 7 to 10 days, famciclovir 250 mg orally three times daily for 7 to 10 days, or valacyclovir 1 g orally twice daily for 7 to 10 days have resulted in an improved quality of life for those infected with herpes simplex virus (HSV). However, according to the CDC, these drugs neither eradicate latent virus nor affect the risk, frequency, or severity of recurrences after the drug is discontinued.

A pregnant woman has been diagnosed with pica since she eats lead paint chips for their sweetness. The nurse educating this woman should strongly encourage her to abandon this practice because it may have which consequence to the fetus? a. neurological challenges b. cataracts c. fetal growth restriction d. spontaneous abortion

a. Lead ingestion during pregnancy may lead to a newborn who is both cognitively and neurologically challenged. Formaldehyde exposure can lead to spontaneous abortions (miscarriages). Breathing air filled with pollutants (such as carbon monoxide) has been shown to lead to fetal growth restriction. The rubella virus' teratogenic effects on a fetus can be devastating, such as hearing impairment, cognitive and motor challenges, cataracts, and cardiac defects.

The nurse is assisting a pregnant client who underwent a nonstress test that was ruled reactive. Which factor will the nurse point out when questioned by the client about the results? a. There is no evidence of congenital anomalies or deformities. b. The fetal heart rate increases with activity and indicates fetal well-being. c. The fetus is developing at a fast rate but doing fine. d. The results indicate a stress test is needed for further evaluation.

b. A nonstress test is a noninvasive way to monitor fetal well-being. A reactive NST is a positive sign the fetus is tolerating pregnancy well by demonstrating heart rate increase with activity, and this indicates fetal well-being. This test is not used to determine congenital anomalies or deformities. It does not determine the speed by which fetus is developing. Further evaluation would be necessary if the results were nonreactive.

The nurse is reviewing the role of the mammary glands in the reproductive process with a birth class. What should the nurse tell the class is the name of the ducts in the mammary glands that form a small reservoir for milk? a. areola b. lactiferous c. lobules d. nipple

b. Each breast is divided into 15 to 20 lobes of glandular tissue, covered by adipose (fat) tissue, which gives the breast its shape. The lobes are made up of lobules, which consist of milk-secreting cells in glandular alveoli. From the alveoli, small lactiferous ducts converge toward each nipple like the spokes of a wheel. Each lactiferous duct forms a small reservoir for milk. The structures of the breast include the nipple, the areola, and the areolar glands. The nipple is a circular projection containing some erectile tissue. It is surrounded by the pigmented areola. Areolar glands, which are close to the skin's surface, make the areola appear rough.

A woman experiencing menopausal symptoms asks the nurse about herbal remedies for managing her symptoms. Which response by the nurse would be most appropriate? a. "Herbal remedies have been proven quite reliable in alleviating the symptoms of menopause." b. "You should avoid herbal remedies and ask your primary care provider for hormone replacement therapy." c. "Most of the information about herbal therapies is based on reports by those who have used them, not scientific studies." d. "Your symptoms are not severe enough to warrant any treatment or therapy."

c. Information about the efficacy of herbal therapies is largely anecdotal. Research to validate the efficacy, safety, and potential harmful effects of herbal therapies is lacking. Until recently, hormone replacement therapy was the mainstay of treatment, but in light of the results of the Women's Health Initiative trial, it has become controversial. Telling the woman that her symptoms are not severe enough is demeaning and inappropriate: the woman's symptoms are significant to her, regardless of what the nurse may think.

A young couple who underwent preconceptual genetic testing is at high risk for having a child with Down syndrome and have decided not to have children. Which response by the nurse is most appropriate? a. "If you would like to discuss this further, here is the contact information for the genetic counseling center." b. "You always have the option to adopt a child if you are really wanting to become parents." c. "The health care provider can refer you for a second opinion if you are not happy with these results." d. "To still have a child of your own, you should consider asking a friend or family member to be a surrogate."

a. Even if a couple decides not to have more children, the nurse should be certain they know genetic counseling is available for them should their decision change. It is not appropriate for the nurse to discuss adoption or surrogacy at this time. The couple needs time to process this information first. The nurse should avoid using terms such as "always" as the couple may have barriers that would prevent them from being able to adopt. The nurse should not state the client should approach family or friends for surrogacy at this time. If the couple would request information, the nurse would list all potential possibilities for surrogacy and not place emphasis/pressure on a certain group. Being referred for a second opinion is providing the couple with false hope. If the couple would request a referral, it would not be denied.

A pregnant client in the first trimester asks the nurse about taking medications while she is pregnant. She tells the nurse that she heard that it can be harmful to the fetus if medications are taken at certain times during pregnancy. What is the best response by the nurse? a. "Exposure to certain substances during the embryonic phase may be harmful to the developing fetus." b. "As long as you are past 4 weeks of pregnancy, you should be able to take most medications." c. "There is no need for you to worry; you are not far enough along in your pregnancy for this to be a problem." d. "You cannot drink alcohol, but you can take some medications, such as cold preparations and over-the-counter medications."

a. Exposure to a teratogen during the embryonic stage produces the greatest damaging effects because cells are rapidly dividing and differentiating into specific body structures.

A young couple is exploring their contraceptive options and are curious about using an intrauterine contraceptive device. The nurse explains that there are two types, one that uses hormones and one that uses: a. copper. b. magnesium. c. silicone. d. potassium.

a. The implants contain either copper or progesterone to enhance their effectiveness. One or two attached strings protrude into the vagina so that the user can check its placement. Four IUCs are currently available in the United States: the copper ParaGard-TCu-380A, the levonorgestrel-releasing intrauterine systems (LNG-IUSs) marketed as Mirena and Kyleena, and the LNG-IUD marketed as Jaydess. The ParaGard-TCu-380A is approved for 10 years of use and is nonhormonal. Its mechanism of action is based on the release of copper ions, which alone are spermicidal. Additionally, the device causes an inflammatory action leading to a hostile uterine environment.

A pregnant client wishes to know if sexual intercourse would be safe during her pregnancy. Which should the nurse confirm before educating the client regarding sexual behavior during pregnancy? a. Client does not have cervical insufficiency. b. Client does not have anxieties and worries. c. Client does not have anemia. d. Client does not experience facial and hand edema.

a. The nurse should inform the client that sexual activity is permissible during pregnancy unless there is a history of cervical insufficiency, vaginal bleeding, placenta previa, risk of preterm labor, multiple gestation, premature rupture of membranes, or presence of any infection. Anemia and facial and hand edema would be contraindications to exercising but not intercourse. Freedom from anxieties and worries contributes to adequate sleep promotion.

A nurse is caring for a pregnant client who has been diagnosed with lordosis. The nurse offers preventive measures for which consequence of lordosis when caring for this client? a. melasma (chloasma) b. edema in lower extremities c. chronic backache d. diastasis recti

c. The nurse should provide preventive measures for chronic backache as a consequence of lordosis when caring for this client. Melasma (chloasma) is characterized by darkened areas on the face, particularly over the nose and cheeks. It is also known as the mark of pregnancy. Chloasma is not caused by lordosis. Diastasis recti occurs as the pregnancy progresses when the rectus muscle stretches to the point that it separates. It is not caused by lordosis. Edema in lower extremities occurs due to an impeded venous return caused by the pressure of the growing fetus on pelvic and femoral areas. It is not caused by lordosis.

The nurse educates a 15-year-old female client on ways to prevent pregnancy. Which statement will the nurse include in the teaching? a. "Oral contraceptive pills are effective at preventing pregnancy as long as you take one each week." b. "The withdrawal method is effective as long as you have a monogamous partner." c. "The use of a condom is a method often recommended to prevent pregnancy for people your age." d. "If you do not want to become pregnant within the next 10 years, an intrauterine device would be best."

c. The nurse would state condoms as the best option for preventing pregnancy for this age group. Oral contraceptives are also effective but must be taken daily at approximately the same time. The withdrawal method is not effective, regardless of the number of sexual partners because sperm can be released before ejaculation occurs. An intrauterine device could be removed if the client desired pregnancy in the upcoming years.

The nurse is conducting the first prenatal assessment on a newly pregnant client. She shares with the nurse that she has 3 children, 2 born at full-term and one at 34 weeks' gestation. Her last pregnancy ended in a miscarriage. How should the nurse document this client's obstetrical history? a. G3 T2 P1 A1 L3 b. G4 T3 P0 A1 L3 c. G4 T2 P1 A1 L3 d. G5 T2 P1 A1 L3

d. One of the most common methods of recording the obstetric history is to use the acronym GTPAL. "G" stands for gravida, the total number of pregnancies including the current one. "T" stands for term, the number of pregnancies that ended at term (at or beyond 38 weeks' gestation); "P" is for preterm, the number of pregnancies that ended after 20 weeks and before the end of 37 weeks' gestation. "A" represents abortions, the number of pregnancies that ended before 20 weeks' gestation to include miscarriage. "L" is for living, the number of children delivered who are alive at the time of history collection. . For this client, G5 = current pregnancy (1) + children (3) + miscarriage (1); T2 = children born at 38+ weeks (2); P1 = children born between 20 and 37 weeks (1); A1 = abortion (0) + miscarriage (1); L3 = number of living children at time of assessment (3).

The nurse is teaching a group of middle school students about sexually transmitted infections (STIs). Which statement by the students indicates a need for additional teaching about condom use? a. "I will put the condom on just before orgasm." b. "Latex condoms provide the best protection against STIs." c. "I will put the condom on just before insertion." d. "I will not store condoms in my wallet."

a. Condoms should be placed on the penis before any genital contact occurs. Applying them at a later time decreases their effectiveness. Latex condoms are best to protect against infection. Condoms should be stored in a cool, dry place that is away from direct sunlight. A wallet is subject to extreme changes in temperature and is an inappropriate place to store a condom.

A nurse is preparing for a class discussion on sexually transmitted infections (STIs) to be given at a local high school. Which would the nurse include as a discussion priority? a. Adults aged 21 to 30 years are at greater risk of contracting an STI. b. Adolescents and young adults are the largest age group diagnosed with an STI. c. College students are more likely to engage in risky sexual behaviors leading to an STI. d. More single mothers are diagnosed with an STI causing sterility.

b. Individuals aged 15 to 24 years represent almost half of all cases of new STIs. Four in ten sexually active teen girls, not single mothers, have an STI that can cause infertility. In the United States, teens who are sexually active, not college-aged students, experience high rates of STIs.

An 18-year-old female client who is sexually active asks the nurse for information on barrier contraceptives. Which statement is appropriate for the nurse to include in the response? a. "The use of a diaphragm is a good choice if you have frequent urinary tract infections." b. "A cervical cap is a barrier contraceptive that you can purchase over the counter." c. "A diaphragm must be inserted 1 hour prior to sexual intercourse." d. "Cervical caps are most effective when used along with a spermicidal agent."

d. The nurse would include information on using spermicidal agents along with barrier contraceptives. A cervical cap is a mechanical barrier contraceptive that interferes with conception by physically preventing sperm from fertilizing ova. Cervical caps cannot be purchased over the counter, because the client must be properly fitted for the device to work effectively. Frequent urinary tract infections are a contraindication to the use of diaphragms because diaphragm use can place pressure placed on the urinary tract. Diaphragms can be placed up to 18 hours before intercourse; however, 6 hours is the most commonly recommended time. Spermicide should not be placed more than 2 hours before intercourse.

The nurse is providing prenatal education in the community. The nurse advises the pregnant women to check with their health care provider before what activity(ies)? Select all that apply. a. receiving immunizations b. taking over-the-counter herbs c. taking "natural" medications d. eating spicy foods e. drinking bottled water

a, b, c. The pregnant woman is taught to consider that substances she takes into her body may pass to the fetus. These include immunizations, over-the-counter herbs, and all medications, even the ones labeled as "natural." The woman should verify with her health care provider before any of those things are taken. Eating spicy food and drinking specific brands of bottled water would not need to be cleared with the provider unless the woman experienced gastrointestinal symptoms following ingestion.

Which occupation may expose a fetus to environmental hazards? Select all that apply. a. nurse anesthetist working in a busy oral surgeon's office b. accountant c. short-order cook for a busy deli d. nurse working for a pulmonologist who administers inhalation ribavirin routinely to the client population e. carpenter

a, c, d. Some occupations are hazardous during pregnancy and should be discontinued because they bring women into contact with harmful substances. For example, nurses working with anesthetic gases in operating rooms or dental offices are reported to have a higher incidence of spontaneous miscarriage and, possibly, congenital anomalies in children than nurses working in other locales, probably because of exposure to nitrous oxide. Nurses working with chemotherapy agents may also be at risk and should wear gloves to protect themselves from exposure to these drugs. Ribavirin, an antiviral formerly used to treat respiratory syncytial virus (RSV) infections, is also apparently teratogenic. It is administered by inhalation to individuals with RSV in hematopoietic cell or heart and lung transplant recipients.

A nurse is caring for a client with vaginitis. What teaching(s) should the nurse provide to the client to prevent recurrent vaginal infections? Select all that apply. a. Avoid using deodorant tampons. b. Clean the perineal area from back to front. c. Avoid douching. d. Wear only cotton panties and ventilated pantyhose. e. Take a bubble bath once a week.

a, c, d. The nurse should teach the client ways to help prevent vaginitis. Prevention activities include avoiding douching to prevent altering the vaginal environment; using condoms to avoid spreading the organism; urinating with knees spread wide apart; avoiding tights, nylon underpants, and tight clothes; wiping from front to back after using the toilet; washing only with hypoallergenic bar soap, avoiding liquid soaps or body washes; avoiding powders, bubble baths, and perfumed vaginal sprays; wearing clean cotton underpants; and washing and drying the vulvar area gently after baths or showers.

The nurse is preparing a teaching plan for a pregnant woman about the signs and symptoms to be reported immediately to her health care provider. Which signs and symptoms would the nurse include? Select all that apply. a. headache with visual changes in the third trimester b. urinary frequency in the third trimester c. sudden leakage of fluid during the second trimester d. nausea with vomiting during the first trimester e. lower abdominal pain with shoulder pain in the first trimester f. backache during the second trimester

a, c, e. Danger signs and symptoms that need to be reported immediately include headache with visual changes in the third trimester; sudden leakage of fluid in the second trimester; and lower abdominal pain accompanied by shoulder pain in the first trimester. Urinary frequency in the third trimester, nausea and vomiting during the first trimester, and backache during the second trimester are common discomforts of pregnancy.

The police have brought a young female to the emergency department after they raided a "crack" house and found this female passed out and bleeding from her "bottom." This female is pregnant and is likely bleeding related to which complication of cocaine use during pregnancy? Select all that apply. a. premature dislodgement of the placenta b. rupture of cervical lesions c. rupture of uterus d. dislodgement of fibroid cysts e. spontaneous miscarriage

a, e. Cocaine, particularly in crack form, is potentially harmful to a fetus because it causes severe vasoconstriction in the mother, thus compromising placental blood flow and perhaps dislodging the placenta. Its use is associated with spontaneous miscarriage, preterm labor, meconium staining, and intrauterine growth restriction. Cocaine is not associated with development of cervical lesions, rupture of uterus, or dislodgement of fibroid cysts.

The 36-year-old client with subfertility has been tracking menstrual cycles with basal body temperature. The client finds that the luteal phase is typically 8 to 9 days long. What treatment does the nurse anticipate? a. progesterone vaginal b. conjugated estrogen c. clomiphene citrate d. bromocriptine

a. A luteal phase is the postovulation phase of the menstrual cycle; a luteal phase of less than 10 days can indicate a luteal phase defect. This is commonly treated with progesterone vaginal suppositories to support the development of the endometrium for implantation. Both clomiphene citrate and bromocriptine are ovulation stimulating medications; these are not indicated. Estrogen does not support the luteal phase or endometrium development.

The mother of a 9-year-old girl is concerned because her daughter has already begun menstruating. The daughter is of normal height for her age and weighs 150 lbs (68 kg). The mother is concerned that this is not normal and asks about the factors that could be involved. Which response by the nurse would be best? a. "It has become more common for girls to start their periods as early as 9; her weight may be a contributing factor." b. "You are right to be concerned, as 9 is not a normal age to begin menstruation; lack of proper nutrition may be a contributing factor." c. "Nine is an early age to begin menstruation; anorexia nervosa may be a contributing factor." d. "Nine is a completely normal age to begin menstruation; the age of menarche is governed completely by genetics."

a. Although it is not proven, the general consensus is a girl must reach a critical weight of approximately 95 lb (43 kg) or develop a critical mass of body fat before the hypothalamus is triggered to send initial stimulation to the anterior pituitary gland to begin the formation of FSH and LH. Probably because of the combination of better nutrition and increased obesity, girls are beginning puberty at earlier ages than ever before (9 to 12 years). Nine is an early age to begin menstruation, but it is within the normal range. Anorexia nervosa is associated with delays or halts in menstruation, not early menstruation. The age of menarche is not governed completely by genetics; other factors are involved.

A client in her 39th week of gestation arrives at the maternity clinic stating that earlier in her pregnancy, she experienced shortness of breath. However, for the past few days, she has been able to breathe easily, but she has also begun to experience increased urinary frequency. A nurse is assigned to perform the physical examination of the client. Which observation is most likely? a. Fundal height has dropped since the last recording. b. Fundal height is at its highest level at the xiphoid process. c. The fundus is at the level of the umbilicus and measures 20 cm. d. The lower uterine segment and cervix have softened.

a. Between 38 and 40 weeks of gestation, the fundal height drops as the fetus begins to descend and engage into the pelvis. Because it pushes against the diaphragm, many women experience shortness of breath. By 40 weeks, the fetal head begins to descend and engage into the pelvis. Although breathing becomes easier because of this descent, the pressure on the urinary bladder now increases, and women experience urinary frequency. The fundus reaches its highest level at the xiphoid process at approximately 36, not 39, weeks. By 20 weeks' gestation, the fundus is at the level of the umbilicus and measures 20 cm. At between 6 and 8 weeks of gestation, the cervix begins to soften (Goodell sign) and the lower uterine segment softens (Hegar's sign).

A community health nurse is conducting an educational session at a local community center on sexually transmitted infections (STIs). The nurse considers the session successful when participants identify which statement as correct? a. "Human papillomavirus is the cause of essentially all cases of cervical cancer." b. "At least antibiotics will cure a sexually transmitted infection." c. "Gonorrhea and syphilis are infections seen only in men." d. "STIs can't be transmitted through oral sexual intercourse."

a. Clinical studies have confirmed that HPV is the cause of essentially all cases of cervical cancer, which is the fourth most common cancer in women in the United States. Up to 95% of cervical squamous cell carcinomas and nearly all preinvasive cervical neoplasms are caused by the HPV. Antibiotics will only cure those STIs caused by bacterial infections. A viral STI infection is a lifetime infection. Gonorrhea and syphilis affect both men and women. Sexually transmitted infections are infections of the reproductive tract caused by microorganisms transmitted through oral sexual intercourse.

The nurse is conducting an initial history and assessment on a client at 10 weeks' gestation who is pregnant with her first child. Which question is a priority for the nurse to ask the client at this time? a. "Does anyone in your or the father's family have any genetic disorders?" b. "Do you plan to breastfeed or bottle feed your new baby?" c. "Are you planning to develop a birth plan for use during the birthing process?" d. "Was your mother or grandmother diagnosed with gestational diabetes?"

a. Due to the timing of the client's pregnancy, it is a priority for the nurse to assess for potential risk factors for genetic abnormalities. The client is in the appropriate time frame to begin genetic testing if desired. The nurse will determine the client's feeding preference and plans to have a birth plan closer to birth to allow the client time to research these topics and make an informed decision about both. Gestational diabetes is not thought to be genetic in nature. The client will be tested for gestational diabetes regardless of family history during her second trimester.

A multigravida client is concerned that she may deliver early. When asking the nurse what is the earliest her baby can be delivered and survive, which time frame would the nurse point out? a. The end of the second trimester b. The end of the first trimester c. The end of the third trimester d. The end of the fourth trimester

a. During the second trimester, fetal growth is significant. The fetus begins this trimester 3 inches long and weighing less than 1 oz (0.8 gm). By the end of the second trimester, the fetus is about 15 inches long and weighs more than 2 lbs (1000 gm). Major organs develop to the point that the fetus may survive (with help) outside the womb. The fetus would not be able to survive at the end of the first trimester. The end of the third trimester is the expected time of deliver. The fourth trimester involves the time after delivery.

A nurse is assessing a male client recently diagnosed with genital herpes. Which finding would most likely correlate with this diagnosis? a. reports of itching, tingling and pain in genital area b. painful urination with a penile discharge present c. wart-like flesh-colored lesions on the scrotal area d. a chancre on the penis

a. Initial symptoms for a male with genital herpes would include itching, tingling, and pain in genital area followed by small pustules and blister-like genital lesions. Gonorrhea presents with a penile discharge (pus). Wart-like lesions that are soft, moist, or flesh-colored and appear on the scrotum with HPV. Syphilis in both male and female presents with a chancre.

A woman in the third trimester of her first pregnancy expresses fear about the birth canal being wide enough for her to push the baby through it during labor. She is a petite person, and the baby seems so large. She asks the nurse how this will be possible. To help alleviate the client's fears, the nurse should mention the role of the hormone that softens the cervix and collagen in the joints, which allows dilation (dilatation) and enlargement of the birth canal. What is this hormone? a. relaxin b. progesterone c. estrogen d. human placental lactogen

a. Relaxin, secreted by the corpus luteum of the ovary as well as the placenta, is responsible for helping to inhibit uterine activity and to soften the cervix and the collagen in joints. Softening of the cervix allows for dilation (dilatation) at birth; softening of collagen allows for laxness in the lower spine and so helps enlarge the birth canal. The effect of estrogen is to cause breast and uterine enlargement. Progesterone has a major role in maintaining the endometrium, inhibiting uterine contractility, and aiding in the development of the breasts for lactation. Human placental lactogen (hPL), also known as human chorionic somatomammotropin, serves as an antagonist to insulin, making insulin less effective and thereby allowing more glucose to become available for fetal growth.

A community health nurse is actively involved in various community projects. The nurse is providing a secondary prevention activity by organizing which event? a. a skin cancer screening fair b. sexually transmitted infection (STI) education c. internet instruction for older adults d. application of a colostomy device

a. Secondary prevention measures are those taken to screen for diseases (such as skin cancer screening), delayed development according to criteria, or use of medication. Primary prevention involves health promotion activities to prevent the development of illness or injury. This level of prevention includes giving information which could include teaching older adults how to use the internet to find reliable information concerning various diseases, or providing STI education to prevent the spread of the disease. Tertiary prevention includes health promotion activities that focus on rehabilitation and that provide information to prevent further injury or illness, such as teaching a client how to properly apply a colostomy device.

A pregnant client at 24 weeks' gestation calls the clinic crying after a prenatal visit, where she had a pelvic exam. She states that she noticed blood on the tissue when she wiped after voiding. What initial statement by the nurse would explain this finding? a. The cervix is very vascular during pregnancy, so spotting after a pelvic exam is not unusual. b. She may have a bleeding disorder so she needs to come back to the clinic for blood work. c. It is possible she is losing her mucus plug, which can cause bloody show. d. Some bleeding during pregnancy is not uncommon and this finding is expected.

a. Slight bleeding after a pelvic exam in a pregnant woman is common due to the vascularity of her cervix during pregnancy. Suggesting a bleeding disorder is frightening and not substantiated by the data. Bleeding is not a normal finding during pregnancy and losing the mucus plug occurs at the end of pregnancy, just prior to labor.

A client at 32 weeks' gestation is admitted to labor and delivery with vaginal bleeding and contractions. The physician orders a course of two steroid injections. The client asks why she needs steroids. What is the best explanation by the nurse? a. The steroids speed up the development of the lungs. b. The steroids will help to slow the development of infection. c. The steroids will increase the baby's muscle mass. d. The steroids will create a layer of fat to help with temperature regulation.

a. Steroids given to the mother before birth help to speed up the development of the fetal lungs. The use of prenatal steroids has decreased the mortality rate in preterm infants. Prenatal steroids do not increase muscle mass or amount of fat tissue to aid in temperature regulation. Prenatal steroids do not have an impact on the development of sepsis in either the mother or neonate.

A 24-year-old female presents with vulvar pruritus accompanied by irritation, pain on urination, erythema, and an odorless, thick, acid vaginal discharge. She denies sexual activity during the last six months. Her records show that she has diabetes mellitus and uses oral contraceptives. Which category of antimicrobial medication is most likely to clear her symptoms? a. an azole antifungal agent b. a penicillin antibiotic c. a quinolone antibiotic d. an antiviral agent

a. The character of the discharge and the lack of recent sexual activity suggest infection with Candida, which can exist asymptomatically and flare up only if conditions, such as an imbalance in normal vaginal flora resulting from antibiotic treatment, diabetes, or oral contraceptive use, enable the fungus to proliferate. Candidiasis responds well to treatment with azole antifungal agents.

A 12-year-old girl expresses concern to the nurse because she has not begun her period yet. The nurse notes that the girl has grown 6 in (15 cm) in the past year and a half and is beginning to develop breast buds. Which response by the nurse is appropriate? a. "It is normal for females to begin menstruating between the ages of 12 and 15." b. "It is difficult to say when you will get your period, but you should not be concerned at this time." c. "The average age to begin your period is 12. I will notify the health care provider." d. "You sound concerned about not having your period. Is there a bigger issue bothering you?"

a. There is wide variation in the time required for adolescents to move through sexual developmental stages; however, the sequential order is fairly constant. In girls, pubertal changes typically occur in the following order: growth spurt, increase in transverse diameter of the pelvis, breast development, growth of pubic hair, onset of menstruation, growth of axillary hair, and vaginal secretions. Thelarche, which is the beginning of breast development, usually starts 1 to 2 years before menstruation. Given the signs of development that this client has already exhibited, it seems likely that she will begin her period in the coming year. Her development is completely normal, so the nurse should not suggest something is wrong by stating she will notify the health care provider. The client does not indicate another issue is concerning her; therefore, the nurse would address the client's current statement. The nurse would not tell the client to not be concerned as this statement is not therapeutic and does not address the client's concern.

A client in her third month of pregnancy arrives at the health care facility for a regular follow-up visit. The client reports discomfort due to increased urinary frequency. Which instruction should the nurse offer the client to reduce the client's discomfort? a. Avoid consumption of caffeinated drinks. b. Drink fluids with meals rather than between meals. c. Avoid an empty stomach at all times. d. Munch on dry crackers and toast in the early morning.

a. To reduce the client's urinary frequency, the nurse should instruct the client to avoid consuming caffeinated drinks, since caffeine stimulates voiding patterns. The nurse instructs the client to drink fluids between meals rather than with meals if the client complains of nausea and vomiting. The nurse instructs the client to avoid an empty stomach at all times, to prevent fatigue. The nurse also instructs the client to munch on dry crackers or toast early in the morning before arising if the client experiences nausea and vomiting; this would not help the client experiencing urinary frequency.

The nurse is explaining the phases of the uterine cycle to a client. What action during the proliferative phase of the uterine cycle should the nurse include in the teaching session with the client? a. While the ovarian follicles are producing increased amounts of estrogen, the endometrium prepares for possible fertilization with pronounced growth. b. If fertilization does not occur, the corpus luteum degenerates, and hormonal levels fall. c. Withdrawal of hormones causes the endometrial cells to change, and menstruation begins. d. An ovum matures and is released.

a. While the ovarian follicles are producing increased amounts of estrogen, the endometrium prepares for possible fertilization with pronounced growth during the proliferative phase. If fertilization does not occur, the corpus luteum degenerates, and hormonal levels fall during the secretory phase. Withdrawal of hormones causes the endometrial cells to change, and menstruation begins. Maturation of an ovum occurs during the ovarian cycle; growth of the lining (endometrium) of the uterus occurs during the uterine cycle. Together, these cycles are known as the menstrual cycle.

A client at a preconception screening appointment indicates to the nurse that she is a carrier of muscular dystrophy, an X-linked recessive trait. Her partner does not have the trait. Which statement made by the client indicates an understanding of the implications? a. "Each time I have a child, there is a 25% chance a female child will be a carrier." b. "Because this is an X-linked recessive trait, any male children will not be affected." c. "There are more affected females than males with this particular trait." d. "There is a good chance all my future children will be affected with this trait."

a. X-linked recessive inheritance affects more males than females. There is no male-to-male transmission but any man who is affected with an X-linked recessive disorder will have carrier daughters. If a woman is a carrier, there is a 25% chance she will have an "affected son," a 25% chance that her daughter will be a "carrier," a 25% chance that she will have an "unaffected" son, and a 25% chance her daughter will be a "noncarrier."

The nurse is caring for four pregnant clients. Which client will the nurse highlight for the health care provider as being at highest risk for carrying a child with a genetic abnormality? a. 38-year-old client whose partner is 43 years of age and who have a child diagnosed with hypoplastic left heart syndrome b. 36-year-old client with thalassemia whose partner is 55 years of age and who have a son with Duchenne muscular dystrophy c. 28-year-old client diagnosed with hemophilia whose partner is 38 years of age and has hypertension d. 17-year-old client diagnosed with schizophrenia whose partner is 15 years of age and has type 2 diabetes

b. The nurse knows the 36-year-old client with thalassemia whose partner is 55 years of age and have a son with Duchenne muscular dystrophy is at highest risk with 3 risk factors. Maternal age of 35 or older, paternal age 50 or older, having a genetic disorder (such as thalassemia or hemophilia) already having a child with a genetic disorder (such as Duchenne muscular dystrophy), family history of genetic disorders, and exposure to a known risk factor (such as certain drugs or infections) are all known risk factors. Nongenetic disorders which are not risk factors include type 2 diabetes, hypoplastic left heart syndrome, hypertension, and schizophrenia.

A client in her second trimester of pregnancy arrives at the health care facility for a routine follow-up visit. The nurse is required to educate the client so that the client knows what to expect during her second trimester. Which information should the nurse offer? a. "You may feel physical discomfort as the baby inside grows." b. "You will experience quickening, and you will actually feel the baby." c. "You will be more conscious of the changes taking place in your body now." d. "You may have mood swings that could overwhelm your partner."

b. The nurse should inform the client that quickening occurs in the second trimester. The client will be able to physically feel the fetal movements, which will help her bond with her developing fetus. Physical discomfort actually starts to increase in the third trimester as the fetus grows rapidly. The client feels conscious of the changes taking place in her body due to her pregnancy primarily in the first trimester, not the second. The client is likely to have mood swings in the first trimester of the pregnancy, which can be very overwhelming for the client as well as her partner.

The nurse is providing care for a pregnant client who has been given the necessary requisitions for laboratory work by the primary care provider. The client notices that the lab tests include testing for HIV and other sexually transmitted infections, and expresses alarm, stating, "I don't understand why the doctor would suspect that I've got these diseases." What is the nurse's most therapeutic statement? a. "Unfortunately, these infections have the potential to harm the fetus. It's important that the doctor identifies them early in your pregnancy." b. "Every pregnant client is tested for these diseases; it doesn't necessarily suggest that the doctor suspects that you have them." c. "Pregnancy is a major change, so every member of the care team makes sure that your health is assessed carefully." d. "Sexually transmitted infections are much more common than most people believe."

b. The nurse should reassure the client that these lab tests are ordered for all clients, not only those who are at high risk for sexually transmitted infections. Making general statements about the incidence of sexually transmitted infections or the need for thorough care does not address the client's expressed concern.

A pregnant client has been diagnosed with gonorrhea. Which nursing interventions should be performed to prevent gonococcal ophthalmia neonatorum in the baby? a. Administer cephalosporins to mother during pregnancy. b. Instill a prophylactic agent in the eyes of the newborn. c. Perform cesarean birth to prevent infection. d. Administer an antiretroviral syrup to the newborn.

b. To prevent gonococcal ophthalmia neonatorum in the baby, the nurse should instill a prophylactic agent in the eyes of the newborn. Cephalosporins are administered to the mother during pregnancy to treat gonorrhea but not to prevent infection in the newborn. Performing a cesarean birth will not prevent gonococcal ophthalmia neonatorum in the newborn. An antiretroviral syrup is administered to the newborn only if the mother is HIV positive and will not help prevent gonococcal ophthalmia neonatorum in the baby.

A client at 39 weeks' gestation calls the OB triage and questions the nurse concerning a bloody mucus discharge noted in the toilet after an OB office visit several hours earlier. What is the best response from the triage nurse? a. "It might be nothing. If it happens again call your provider who is on-call." b. "If the provider did an exam, it might be just normal vaginal secretions, so don't worry about it." c. "A one time discharge of bloody mucus in the toilet might have been your mucus plug." d. "Bloody mucus is a sign you are in labor. Please come to the hospital."

c. Bloody mucus can either be a mucus plug or bloody show. The one-time occurrence would be more likely to be the mucus plug. A bloody show would continue if her cervix was changing, but this usually does not occur until after contractions start. It is a sign that something is happening and should be reported to the health care provider. The bloody mucus is not a sign of labor, but it can be an early sign that labor is coming soon.

A 28-year-old primigravida client with diabetes mellitus, in her first trimester, comes to the health care clinic for a routine visit. The client reports frequent episodes of sweating, giddiness, and confusion. What should the nurse tell the client about these experiences? a. tissue sensitivity to insulin increases as pregnancy advances b. use of insulin needs to be reduced as pregnancy advances c increased secretion of insulin occurs in the first trimester d. insulin resistance becomes minimal in the latter half of the pregnancy

c. Increased secretion of insulin in the maternal body in the first trimester is due to the rise in serum levels of estrogen, progesterone, and other hormones. During the second half of pregnancy, tissue sensitivity to insulin progressively decreases, producing hyperglycemia and hyperinsulinemia. Use of insulin needs to be increased not reduced as pregnancy advances. Insulin resistance becomes maximal not minimal in the latter half of the pregnancy.

The nurse is teaching an adolescent client about fertility and the various phases of the menstrual cycle. The client has a 28-day menstrual cycle. Which statement is a priority for the nurse to include in the teaching? a. "Days 1 through 5 are the days you will typically experience menstrual flow." b. "Day 10 is part of the proliferative phase of your menstrual cycle." c. "Day 14 of your menstrual cycle is normally when you will ovulate." d. "Days 27 and 28 are when you have declining hormonal secretions."

c. It is a priority for the nurse to teach the client when ovulation will occur to assist in pregnancy prevention at this age. While the other statements are all true, not knowing the specific information for those days does not hold the same consideration or life-altering potential as when pregnancy can occur. The menstrual cycle is based on an average of 28 days and divided into four phases based on the hormones secreted and their actions. Days 1 through 5 are the menstruation phase. Days 6 through 14 are the proliferative phase, ending with ovulation on the 14th day. Days 15 through 26 are the secretory phase. Days 27 and 28 are the ischemic phase when hormonal secretions decline.

A 27-year-old client is in the first trimester of an unplanned pregnancy. She acknowledges that it would be best if she were to quit smoking now that she is pregnant, but states that it would be too difficult given her 13 pack-year history and circle of friends who also smoke. She asks the nurse, "Why exactly is it so important for me to quit? I know lots of smokers who have happy, healthy babies." What can the nurse tell the client about the potential effects of smoking in pregnancy? a. "Smoking is unhealthy for anyone's heart, but your baby faces an especially high risk of heart trouble if you smoke while you're pregnant." b. "Smoking during pregnancy places your baby at an increased risk of intellectual disability." c. "Babies of women who smoke tend to weigh significantly less than other infants." d. "Smoking during pregnancy means that your child will be born with a dependence on nicotine and will have to endure a period of withdrawal in his or her first days of life."

c. Smoking during pregnancy is linked with low birth weight but not cardiac anomalies, intellectual disability, or nicotine dependence.

The nurse is teaching a prenatal class about preparing for their expanding families. What is helpful advice from the nurse? a. "Expect your other children to react positively to their new brother/sister." b. "Your old coping methods will adequately get you through this period of adjustment." c. "The hormones of pregnancy may cause anxiety or depression postpartum." d. "Caring for your new infant is instinctual and will come naturally to you."

c. The "raging hormones" of pregnancy can keep the woman slightly out of touch with her usual methods of coping. Although she may normally interact and communicate in quite mature ways, during a pregnancy she may become depressed, anxious, withdrawn, or angry as she accomplishes her own developmental tasks. Siblings often react to a pregnancy by regression in behavior and attitude because they fear they will be replaced or unloved. In addition to anticipatory guidance concerning the alterations in family structure and functioning, prenatal preparation for first-time parents involves learning the basics of infant care and preparing for infant feeding, particularly for women who plan to breastfeed.

A community health nurse is visiting a 16-year-old new mother. The nurse explains to the client and her mother the genetic screening that is required by the state's law. The client asks why it is important to have the testing done on the infant. What is the nurse's best response? a. "Genetic testing is a way to determine the rate of infectious disease." b. "It is important to test newborns for PKU, congenital hypothyroidism, and galactosemia." c. "PKU, congenital hypothyroidism, and galactosemia are conditions that could result in disability or death if untreated." d. "This testing is required and you will not be able to refuse it. It usually is free so there is no reason to refuse it."

c. The first aim is to improve management, that is, identify people with treatable genetic conditions that could prove dangerous to their health if left untreated. The other answers are incorrect because genetic testing does not determine the rate of infectious disease. The other answers do not adequately explain the rationale for newborn testing.

The nurse is providing teaching to an adolescent girl on the function of the female reproductive system. Which statement by the client best reflects understanding of the material presented? a. "I will always count from the end of my last menstrual cycle to determine when I will ovulate." b. "A vaginal discharge that is thick and white in color indicates that I am ovulating." c. "My menstrual cycle lasts about 28 days each month but varies sometimes, which is normal." d. "During ovulation, the egg is released directly into my uterus to help with fertilization."

c. The normal menstrual cycle lasts an average of 28 days, but there is a variance between women and even between cycles in some women, which is normal. Counting to determine ovulation time is based upon the beginning of the menstrual cycle, not the end. The mucus during ovulation is thin, clear and somewhat stretchable. At ovulation, the ova are released into the abdominal cavity and are propelled along by tiny finger-like projections that direct it to the fallopian tube.

The nurse is caring for a couple trying to conceive. The nurse learns the male client relaxes in the couple's home hot tub daily for 30 to 60 minutes and the female client uses the hot tub once a week for 5 to 10 minutes. What information is best for the nurse to include when providing education to the couple at this time? a. "Hot tubs can spread infections and should be avoided." b. "It is dangerous for a fetus to be exposed to high temperatures." c. "Men desiring to conceive should limit their time in a hot tub." d. "It is great to find ways to relax as stress can lead to subfertility."

c. The nurse would educate the male client to limit time in the hot tub to promote an adequate sperm count and development. Prolonged exposure of the testes to high temperatures can lead to decreased sperm counts. The male's sperm count will rise once the hot tub is avoided. The couple states using their personal home hot tub; therefore, the risk of infection is not a concern. There is limited research showing a risk to the fetus exposed to high temperatures. Current recommendations state the pregnant client should not spend more than 10 minutes in a hot tub. The nurse would provide education on this information once the client is pregnant. Although it is healthy for the couple to limit stress, this is not the most appropriate response from the nurse. The nurse would address the issue of hot tub use because stress is not mentioned as an issue for the couple at this time.

The nurse is caring for a female client diagnosed with vulvovaginal candidiasis. Which instruction will the nurse include in the teaching session with the client? a. "Use superabsorbent tampons and change frequently." b. "Notify your health care provider if you note thick, white vaginal discharge." c. "It is best for you to wear cotton underwear." d. "It is okay for you to take your oral fluconazole tablet with or without food."

c. The nurse would inform the client fluconazole can be taken with or without food. As a preventive measure for the client with frequent vulvovaginal candidiasis, the nurse should instruct the client to wear cotton underwear, not silk. The nurse should instruct the client to use pads instead of superabsorbent tampons, to avoid douching the affected area (as it washes away protective vaginal mucus), and to reduce dietary intake of simple sugars and soda. There is no need to notify the health care provider of expected symptoms such as thick, white vaginal discharge, itching, or inflammation.

The health care provider has prescribed a karyotype for a newborn. The mother questions the type of information that will be provided by the test. What information should be included in the nurse's response? a. The karyotype will provide information about the severity of your baby's condition. b. A karyotype is useful in determining the potential complications the baby may face as a result of its condition. c. The karyotype will assess the baby's chromosomal makeup. d. The karyotype will determine the treatment needed for the infant.

c. The pictorial analysis of the number, form, and size of an individual's chromosomes is referred to as a karyotype. This analysis commonly uses white blood cells and fetal cells in amniotic fluid. The chromosomes are numbered from the largest to the smallest, 1 to 22, and the sex chromosomes are designated by the letter X or the letter Y. The severity and related complications of a disorder are not determined by the karyotype. Condition management is not determined by the karyotype.

The nurse-midwife is performing a pelvic examination on a client who came to her following a positive home pregnancy test. The nurse checks the woman's cervix for the probable sign of pregnancy known as Goodell sign. Which description illustrates this alteration? a. The cervix looks blue or purple when examined. b. The lower uterine segment softens. c. The fundus enlarges. d. The cervix softens.

d. At about the 8th week of gestation, the cervix softens, a probable sign known as Goodell sign. The cervix also looks blue or purple when examined; this is Chadwick sign, and may occur as early as the 6th week of pregnancy. At about 6 weeks, the lower uterine segment softens, a probable sign called Hegar sign. A softening of the uterine fundus, where the embryo has implanted, also occurs by about the 7th week, and the fundus enlarges by the 8th week.

A nurse is caring for a client who is 8 months pregnant. Which instruction is the nurse most likely to give her? a. Perform nipple exercises and stimulation on a regular basis. b. Take a hot water bath or shower daily to maintain hygiene. c. Apply lanolin ointment to the nipple and areola to prevent cracking. d. Rest on the left side for at least 1 hour in the morning and afternoon.

d. During the last months of pregnancy, the nurse should instruct the woman to rest on her left side for at least 1 hour in the morning and afternoon. This position relieves fetal pressure on the renal veins, helps the kidneys excrete fluid, and increases flow of oxygenated blood to the fetus. The body's oil and sweat glands are more active than usual during pregnancy. Thus, a daily warm bath or shower is important, rather than a hot bath, which may produce hyperthermia. Nipple exercises and stimulation should not be done, especially in the third trimester, when they can cause uterine contractions and premature labor. Lanolin ointment may damage the areola and nipple. It has not been shown to be effective in preventing sore and cracked nipples. Lanolin is also a common allergen and may contain insecticide residuals such as DDT.

After teaching a group of college-aged students about condom use, the nurse determines that additional teaching is needed when the students make which statement? a. "Latex condoms are the best protection from STIs." b. "Condoms should be applied before any genital contact." c. "Condoms should be stored in a cool, dry place." d. "It's okay to use petroleum jelly with a latex condom."

d. If external lubricants are used, use only water-based lubricants with latex condoms. Oil-based or petroleum-based lubricants, such as body lotion, massage oil, or cooking oil, can weaken latex condoms. Latex condoms are the best protection from STIs. Condoms are applied before any genital contact because sperm is present in preejactulate fluid. Condoms also should be stored in a cool, dry place away from direct sunlight to prevent deterioration.

When providing preconception care to a client, which instruction will the nurse to provide about medications during pregnancy? a. "You need to avoid all prescription, over-the-counter, and herbal medications when you are pregnant." b. "It is safe for you to take over-the-counter medications." c. "You should switch to herbal remedies because they are safer to use than other types medicines." d. "You need to talk with your health care provider about using all prescription, over-the-counter, and herbal medications."

d. Medication use is common during pregnancy, with prevalence estimates generally exceeding 65% and increasing over the years. Pregnant women use a wide variety of both prescription and over-the-counter medications for both pregnancy-related conditions and conditions unrelated to pregnancy conditions. Little is known about the effects of taking most medications during pregnancy. It is best for pregnant women to not take any medications during their pregnancy. At the very least, they should be encouraged to discuss with the health care provider their current medications and any herbal remedies they take so that they can learn about any potential risks should they continue to take them during pregnancy. A common concern of many pregnant women involves the use of over-the-counter medications and herbal agents. Many women consider these products benign simply because they are available without a prescription. Although herbal medications are commonly thought of as "natural" alternatives to other medicines, they can be just as potent as some prescription medications. The nurse should encourage pregnant women to check with their health care providers before taking anything.

The nurse is caring for a client diagnosed with trichomoniasis. The health care provider has prescribed a single dose of metronidazole. Which information will the nurse provide this client? a. "You may crush or chew this tablet if you have difficulty swallowing medications." b. "Once you are healed, you need to come back to get the human papillomavirus (HPV) vaccination." c. "We need to complete a pregnancy test first, to be sure you can take this medication." d. "Avoid alcohol consumption for at least 24 hours after you take this medication."

d. Metronidazole is an antibiotic used to treat bacterial infections of the vagina. The HPV vaccine is not required for clients diagnosed with trichomoniasis. The nurse would educate the client that having one sexually transmitted infection (STI) places the client at higher risk for future STIs; however, the client does not have to receive the vaccine. A pregnancy test is not necessary, because metronidazole is safe to take during pregnancy. Clients should not consume alcohol or foods or medicines that contain propylene glycol while taking metronidazole and for at least 1 day after to avoid unpleasant side effects (tachycardia, flushed, nausea, vomiting). The nurse would instruct the client to not crush, chew, or break the extended-release tablet.

The clinic nurse is treating a client diagnosed with HIV. Which statement made by the client indicates an understanding of the diagnosis? a. "Right now I have HIV, but eventually it will develop into AIDS." b. "I will not spread HIV to others if I have intercourse with them, if I take my medication." c. "If I do not take my medications as prescribed, my red blood cell count will fall." d. "If my T-cell count drops below 200 cells/mm3 (0.2 × 109/l), my HIV has progressed to AIDS."

d. The client indicates an understanding of the diagnosis when reporting a CD4 T-cell count of at or below 200 cells/mm3(0.2 × 109/l) indicates the development of AIDS. Not all persons with HIV will develop AIDS. There are medications available to help slow the process within the body and limit the development of AIDS. While intercourse can lead to the spread of HIV or AIDS, clients are still able to be sexually active if appropriate measures are taken such as using barriers. Taking the prescribed medication may help prevent the spread but is not a definitive measure. The client should notify partners before intercourse occurs and use a condom. Not taking medications as prescribed will lead to a decrease in white blood cell counts initially, not red blood cells.

The nurse is seeing a client who is going to be married in a month. This client has a history of Huntington disease in her family. The genetic testing has come back, and the client has just been told she carries the gene for Huntington disease and will develop the disease when she gets older. The client asks the nurse if this information is confidential and if it will remain that way. The nurse explains to the client that her family should be told and so should her fiancé. The client forcefully tells the nurse "no." She is not going to tell either her family or her fiancé. What is the nurse's best response? a. "I am ethically bound to tell your family and your fiancé." b. "Your information will remain confidential until the geneticist reviews everything. Then he will have to tell your family." c. "Have you thought about what this disease will do to the person you are going to marry and any children you may have?" d. "I will respect your wishes and keep your information confidential. I do wish you would reconsider though."

d. The nurse must honor the client's wishes and should be sure the client is aware that this action will be done. Although the nurse may ask the client to consider the potential benefit this information may have for other family members, this reply is not the critical or best one. The other two replies should not be stated, because the nurse has to honor the wishes of the client.

Which of a client's cells are most likely to reproduce by meiosis? a. Ova b. Breast tissue c. Endometrium d. Cervix

a. Ova and other sex cells reproduce by meiosis. All of the other tissues are made up of somatic cells, which reproduce by mitosis.

A female sex trade worker has been diagnosed with secondary syphilis. Which findings would most likely correlate with this diagnosis? a. sore throat and flu-like symptoms b. pain-free crusty genital lesions c. yellow vaginal discharge d. painful dysuria

a. With a secondary infection of syphilis, there would be no evidence of vaginal lesions present. Present would be a maculopapular rash (hands and feet); a sore throat; lymphadenopathy; and flu-like symptoms. Dysuria is not seen in the secondary infection. A yellow vaginal discharge is for gonorrhea.

Cystic fibrosis is an example of which type of inheritance? a. autosomal dominant b. autosomal recessive c. X-linked recessive d. multifactorial

b. Cystic fibrosis is an autosomal recessive inherited condition. Huntington disease would be an example of an autosomal dominant inherited condition. Hemophilia is an X-linked recessive inherited condition. Cleft lip is a multifactorial inherited condition.

An anatomy professor is teaching a class of pre-nursing students about the female reproductive system. One student asks, "Where is the opening to the vagina?" The best answer the professor can give is: a. between where you pee and where you poop. b. in what is called the vestibule, which is located between the labia minora. c. by the Bartholin glands. d. between the labia majora and by the paraurethral glands.

b. The vestibule is the area between the labia minora. The urethral meatus (opening to the urethra), paraurethral (Skene) glands, vaginal opening or introitus, and Bartholin glands are located within the vestibule.

The client states that the first day of her last menstrual period is March 23. The nurse is most correct to calculate using Naegele rule that the estimated date of delivery is: a. January 30 b. December 30 c. December 16 d. November 23

b. Using Naegele rule, since the first day of the client's last menstrual period is March 23, 7 days are added leading to the 30th. Subtracting 3 months from March is December. Thus, December 30 is the estimated date of delivery.

A couple comes to the clinic and states to the nurse, "I don't think we are ever going to be able to have children. We have been trying but have had no luck." What assessments does the nurse anticipate will be performed for this couple? Select all that apply. a. in vitro fertilization counseling b. fertility drugs c. semen analysis d. ovulation monitoring e. tubal patency

c, d, e. Today, a subfertility investigation usually is limited to three assessments: semen analysis, ovulation monitoring, and tubal patency.

A client in her second trimester of pregnancy visits a health care facility. The client frequently engages in aerobic exercise and asks the nurse about doing so during her pregnancy. Which precaution should the nurse instruct the pregnant client to take when practicing aerobic exercises? a. Begin a new exercise regimen. b. Wear support hose when exercising. c. Maintain tolerable intensity of exercise. d. Reduce the amount of exercise.

c. Women accustomed to exercise before pregnancy are instructed to maintain a tolerable intensity of exercise. They are instructed not to begin a new exercise regimen. A nurse does not tell the client to wear a support hose when exercising or to reduce the amount of exercises.

The nurse is assessing a pregnant client at her 20-week visit. Which breast assessment should the nurse anticipate documenting? a. Slack, soft breast tissue b. Deeply fissured nipples c. Enlarged lymph nodes d. Darkened breast areolae

d. As part of the pigment changes that occur with pregnancy, breast areolae become darker. The breast tissue should not be softer or slacker than before. There should not yet be any lymph enlargement, and the nipples should not have fissures.

The nurse is assessing a pregnant client at 20 weeks' gestation and obtains a hemoglobin level. Which result would be a cause for concern? a. 12.8 g/dl b. 11.9 g/dl c. 11.2 g/dl d. 10.6 g/dl

d. The average hemoglobin level at term is 12.5 g/dl. The hemoglobin level is considered normal until it falls below 11 g/dl.

When conducting a health history with a couple who are experiencing subfertility, the nurse will include which of the following? Select all that apply. a. General health b. Medication history c. Sexual history d. Surgical history e. School history

a, b, c, d. The nurse will need to gather a thorough health history that will include information related to general health, medications, past sexual history, and surgical history.

A nurse is teaching a group of pregnant women about fetal development. When describing how the various organs form, the nurse describes the primary germ layers involved. Which organ(s) would the nurse explain as developing from the mesoderm? Select all that apply. a. bones b. kidneys c. heart d. brain e. lungs

a, b, c. The mesoderm forms the skeletal (bones), urinary (kidneys), circulatory (heart), and reproductive organs. The ectoderm forms the central nervous system (brain), special senses, skin, and glands. The endoderm forms the respiratory system (lungs), liver, pancreas, and digestive system.

What is the medication of choice for early syphilis? a. penicillin G benzathine b. doxycycline c. tetracycline d. ceftriaxone

a. A single dose of penicillin G benzathine intramuscular injection is the medication of choice for early syphilis or early latent syphilis of less than 1 year's duration. Clients who are allergic to penicillin are usually treated with doxycycline or tetracycline. Ceftriaxone is not the medication of choice for syphilis.

A 45-year-old client and her spouse are present in the clinic. Results of fertility testing indicate that the client has damage to her fallopian tubes. Which would be the most appropriate infertility option for this client? a. Gamete intrafallopian transfer b. In vitro fertilization c. Surrogate mother d. Ovulation stimulation

b, This client has damage to her fallopian tubes, so any procedure would need to bypass this structure. In vitro fertilization fertilizes an ovum and then inserts it into a women's uterus.

The nurse is assigned to clients who are having the following procedures: amniocentesis, fetal nonstress test, chorionic villus sampling, percutaneous umbilical blood sampling, and Doppler assessment of fetal heart rate. For which clients will the nurse ensure that signed informed consent has been given and is in the client's record? a. Amniocentesis, chorionic villus sampling, fetal nonstress test b. Amniocentesis, chorionic villus sampling, percutaneous umbilical blood sampling c. Fetal nonstress test, Doppler assessment of fetal heart rate d. Amniocentesis, percutaneous umbilical blood sampling, Doppler assessment of fetal heart rate

b. While the client ultimately consents to all procedures, some require signed documentation of consent within the client's record. An informed consent is needed for an amniocentesis, chorionic villus sampling and a percutaneous umbilical blood sampling due to the invasive nature of the procedures. Both the fetal nonstress test and the Doppler assessment of the fetal heart rate are non-invasive procedures.

A female client is having a procedure this morning that involves radiologic examination of the fallopian tubes using a radiopaque medium. What procedure should the nurse document as being performed? a. a uterine endometrial biopsy b. a sonohysterography c. a hysterosalpingography d. a hysteroscopy

c. A hysterosalpingography, a radiologic examination of the fallopian tubes using a radiopaque medium, is a frequently used technique.

The nurse is preparing a teaching session for a client considering tubal ligation. Which factor should the nurse prioritize in this session? a. Wait several months after birth, and schedule the surgery as an outpatient. b. The procedure is easy to perform and will be painless. c. This is a permanent and irreversible procedure for birth control. d. She must have signed consent from her partner.

c. The procedure is considered permanent and irreversible. This is a procedure not for routine birth control but for permanent birth control. If the women elects for this procedure, it can be done immediately following the birth of the child, lessening the inconvenience or hospitalization of the client. The procedure is not painless, nor easy. A consent form from the partner is not always required.

The nurse is teaching a male client about collecting a semen sample for analysis. The nurse determines that additional teaching is needed based on which client statement? a. "Once I collect the specimen, I need to take it to the laboratory immediately." b. "I should not have sexual intercourse for 2 to 4 days before getting the specimen." c. "I will be sure to collect the specimen in a plastic bag with a sealed closure." d. "I will carry the specimen in an inside pocket so it is close to my body."

c. To ensure an accurate analysis, semen specimens should be kept at body temperature during transportation. Clients can be advised to carry the specimen next to the body in an inside pocket to accomplish this goal. The semen sample should be taken to the laboratory or health care provider's office immediately so it can be analyzed within 1 hour of collection. The client should also be sexually abstinent 2 to 4 days prior to the analysis. The specimen is to be collected in a clean, dry specimen jar or a special condom (one without spermicide).

What is the most effective way for a nurse to assess a woman's usual food intake during her pregnancy? a. Assess a list she makes describing a good diet. b. Ask her to describe her total intake for a week. c. Assess her skin for hydration and color. d. Ask her to describe her intake for the last 24 hours.

d. A 24-hour food intake history is the best method to assess food intake in all individuals.

The term used to guide the cultural aspects of nursing care and respect individual differences is: a. diversity nursing. b. ethnicity nursing. c. family nursing. d. transcultural nursing.

d. Transcultural nursing is the nursing care method that is guided by cultural aspects and respects individual differences.

A woman tells the nurse that she is going to use a home pregnancy test to determine whether she is pregnant. Which precautions should the nurse give her? a. Use a diluted urine specimen. b. Wait until after two missed menstrual periods. c. Arrange for prenatal care if the test is positive. d. Refrain from eating for 4 hours before testing.

c. Home pregnancy testing can be accurate as soon as a period is missed; it should not take the place of prenatal care.

The nurse is conducting a health history for an infant with a presumed genetic disorder. On which family members will the nurse gather information? Select all that apply. a. siblings b. parents c. great-grandparents d. grandparents e. in-laws

a, b, d. The nurse should obtain information for three generations of family members related to health history. This would need to include half-brothers and half-sisters as well.

The nurse is explaining the latest laboratory results to a pregnant client who is in her third trimester. After letting the client know she is anemic, which heme iron-rich foods should the nurse encourage her to add to her diet? a. Legumes b. Dairy c. Grains d. Meats

d. Meats are the best source of heme-rich iron and should be included in her diet if she is not following a vegetarian diet. Grains and legumes are non-heme iron sources. Dairy products will add various vitamins and calcium to the diet.

A school health nurse is providing education to a group of adolescents regarding the proper procedure for male condom use. The nurse knows the teaching has been effective when which statement is made by a student? a. "Ensure your condom is always available, so store condoms in your wallet." b. "It is important to put the condom on just before the penis is erect." c. "Use only petroleum-based lubricants, such as body lotion or massage oil." d. "Withdraw the penis erect, holding the condom firmly against the penis."

d. The teaching guidelines for proper condom use include: ensure the condom has been stored in a cool, dry place away from direct sunlight. Do not store condoms in wallet; put the condom on before any genital contact; put the condom on when penis is erect; ensure adequate lubrication during intercourse. If external lubricants are used, use only water-based lubricants. Oil-based or petroleum-based lubricants, such as body lotion or massage oil, can weaken latex condoms. Withdraw while penis is still erect, and hold condom firmly against base of penis. Remove carefully to ensure no semen spills out.

The uterus has specific functions in the female body. Which would not be a function of the uterus? a. Aid in childbirth b. Nourish the growing fetus c. Prepare for pregnancy each month d. Stretch during childbirth to accommodate a full-term infant

d. The uterus does not stretch during childbirth to allow passage of the infant. It stretches during the pregnancy. The vagina is the structure that expands during childbirth.

During a routine prenatal visit, a pregnant woman reports a white, thick, vaginal discharge. She denies any itching or irritation. Which action would the nurse take next? a. Notify the health care provider of a possible infection. b. Tell the woman that this is entirely normal. c. Advise the woman about the need to culture the discharge. d. Check the discharge for evidence of ruptured membranes.

b. Vaginal secretions increase during pregnancy and this is considered normal leukorrhea based on the woman's report that she is not experiencing any itching or irritation. There is no evidence indicating the need to notify the health care provider, check for rupture of membranes, or advise her about the need for a culture.

A group of nursing students are preparing a presentation for a health fair illustrating the structures found during a pregnancy. Which structures should the students point out form a protective barrier around the developing fetus? a. ectoderm and amnion b. amnion and mesoderm c. chorion and amnion d. chorion and endoderm

c. The chorion and amnion are the two fetal membranes. The ectoderm, mesoderm, and endoderm are layers in the developing blastocyst.

The nurse is preparing to discuss birthing options with a 25-year-old female who is in a low-risk pregnancy with one older child. Which option will be best for the nurse to recommend for this client? a. Home setting b. Birthing center c. Hospital d. Any birthing settings

d. When a woman is low-risk and has no known medical needs, birth location is the choice of the mother. Home births attended with certified nurse midwives are less likely to have complications and require interventions. Birthing centers are an option for an alternative birth setting, and hospitals are required birth locations for women with potential needs or complications. The nurse should present the advantages and disadvantages of each option and allow the client to make the decision.

A nurse is interviewing the family members of a pregnant client to obtain a genetic history. While asking questions, which information would be most important? a. socioeconomic status of the family members b. avoidance of questions on race or ethnic background c. specific physical characteristics of family members d. if couples are related to each other or have blood ties

d. While obtaining the genetic history of the client, the nurse should find out if the members of the couple are related to each other or have blood ties, as this increases the risk of many genetic disorders. The socioeconomic status or the physical characteristics of family members do not have any significant bearing on the risk of genetic disorders. The nurse should ask questions about race or ethnic background because some races are more susceptible to certain disorders than others.

Many changes occur in the body of a pregnant woman. Some of these are changes in the integumentary system. What is one change in the integumentary system called? a. linea rubria b. Chadwick sign c. ballottement d. melasma (chloasma)

d. The so-called mask of pregnancy, melasma (also known as chloasma) can appear as brown blotchy areas on the forehead, cheeks, and nose of the pregnant woman. This condition may be permanent, or it may regress between pregnancies.

It is important that a nurse recognizes there are cultural differences that will have an influence on the care of the client. Some cultures practice female circumcision, which involves the removal of the: a. clitoris. b. labia minora. c. hymen. d. fourchette.

a. Some cultures practice female circumcision, which involves the removal of the clitoris. It is important for the nurse to remain open-minded and nonjudgmental with all clients.

Before beginning the initial prenatal examination, a nurse should instruct a client to complete what procedure before undressing? a. clean-catch urine b. initial blood tests c. measurement of fundal height d. ultrasound for fetal measurements

a. The first procedure a nurse should ask the client to do is obtain a clean-catch, midstream urine before undressing. Lab tests can be done after the examination is complete. At the first visit, the fetus is too small to be measured or have an ultrasound performed.

The nurse is answering questions from a newborn's parents concerning a circumcision. Which structure will the nurse point out is removed during the procedure? a. tunica albuginea b. corpus spongiosum c. rugae d. prepuce

d. The foreskin or prepuce is a fold of skin which covers the glans of the penis. This fold of skin is removed in a circumcision. The tunica albuginea is connective tissue found inside the penile shaft. The corpus spongiosum is erectile tissue that runs the full length of the penis. Rugae are folds that allow for stretching during an erection.

The nurse advises a pregnant client to keep a small high-carbohydrate, low-fat snack at the bedside. The nurse should point out this will assist with which condition? a. heartburn b. faintness c. slowed GI transit time d. nausea and vomiting

d. Women will commonly experience nausea and vomiting upon awakening first thing in the morning. Clients who experience this should be encouraged to have small snacks at their bedside for eating prior to moving from the bed. Heartburn is a result of pressure and hormone action. Faintness is due to pressure on the vena cava, not blood sugar. GI transit time is not affected.

A nurse educator is describing the structures of the fetal heart to a pregnant client. What information would the nurse likely include in the description? Select all that apply. a. The ductus venosus connects the umbilical vein to the inferior vena cava. b. The ductus arteriosus connects the main pulmonary artery to the aorta. c. The foramen ovale is an anatomic opening between the right and left atrium. d. The ductus arteriosus connects the main pulmonary vein to the vena cava. e. The foramen ovale is an anatomic opening between the right and left ventricles.

a, b, c. Three shunts are present during fetal life. The ductus venosus connects the umbilical vein to the inferior vena cava. The ductus arteriosus connects the main pulmonary artery to the aorta. The foramen ovale is the anatomic opening between the right and left atrium. The function of these shunts is to direct oxygen-rich venous blood to the systemic circulation and to ensure that oxygen-depleted venous blood bypasses the underdeveloped pulmonary circulation.

A nurse is conducting education classes at the local high school on reproductive life planning. Which would be appropriate for the nurse to implement during the teaching? Select all that apply. a. encouragement of abstinence b. proper condom application c. various religious viewpoints d. sexually transmitted infection statistics e. nurse's personal opinion on abortion (elective termination of pregnancy)

a, b, d. The nurse should talk about all safer sex options, including abstinence and its 100% effectiveness. Within that teaching it is important to educate adolescents on the proper techniques for application of a condom as well as to discuss statistics related to sexually transmitted infections.

A woman is in her early second trimester of pregnancy. The nurse would instruct the woman to return for a follow-up visit every: a. 4 weeks. b. 3 weeks. c. 2 weeks. d. 1 week.

a. The recommended follow-up visit schedule is every 4 weeks up to 28 weeks, every 2 weeks from 29 to 36 weeks, and then every week from 37 weeks to birth.

A nurse is reading a journal article about chlamydia. The nurse would expect to find that what percentage of women are asymptomatic when infected with chlamydia? a. 30% b. 50% c. 70% d. 90%

c. Asymptomatic infection is common among both men and women, affecting 50% and 70%, respectively.

After teaching a class on the various structures formed by the embryonic membranes, the nurse determines that the teaching was successful when the class identifies which structure as being formed by the ectoderm? a. lungs b. bones c. ears d. stomach

c. The ectoderm forms the structures of the special senses. The endoderm forms the structures of the respiratory and digestive systems. The mesoderm forms the structures of the skeletal system.

Morbidity rates among children are most highly associated with which cause? a. Firearms at home b. School violence c. Environmental factors d. Suicide and homicide

c. The factors most commonly associated with child morbidity are environmental and socioeconomic problems. The more difficult the societal issues and the more marked the environmental poverty, the higher the illness rates and childhood morbidity. Firearms, violence in schools, homicide, and suicide are all factors in morbidity, but they are not strictly related to children.

A pregnant client who is planning to have genetic testing asks the nurse when she should schedule her amniocentesis. What should the nurse tell the client? a. 10 weeks b. 24 weeks c. 30 weeks d. 16 weeks

d. The nurse should tell the client that an amniocentesis is typically scheduled between 15 and 18 weeks' gestation.

A female client has been trying to conceive for 3 months. She tells the nurse, "I just know something is wrong with me." Which response by the nurse is best? a. "It is time to talk to your health care provider about hormone testing." b. "Do you or your partner have a family history of subfertility?" c. "The problem can be with the man, woman, or both. We will explore all possibilities." d. "It can take up to 1 year of regular, unprotected intercourse to conceive."

d. The nurse would first provide accurate information by informing the client that 3 months is not an adequate time frame to indicate a problem is present. Most couples conceive within 1 year when they have regular intercourse without the use of contraceptives. There is no reason to assess for infertility issues with the female or with the couple's families. It is true that subfertility issues can be related to the man, woman, or both; however, it is not appropriate to indicate there is concern by stating "we will explore all possibilities."

A nurse assesses that a 15-year-old female client has not developed secondary sexual characteristics, is short in stature, and has a webbed neck. The nurse identifies this as being most likely related to which diagnosis? a. Edwards syndrome b. Klinefelter syndrome c. Patau syndrome d. Turner syndrome

d. The nurse should identify these findings as being consistent with Turner Syndrome, which is a gonadal dysgenesis or only having one functional X chromosome.

A nurse is providing genetic counseling to a pregnant client. Which are nursing responsibilities related to counseling the client? Select all that apply. a. explaining basic concepts of probability and disorder susceptibility b. ensuring complete informed consent to facilitate decisions about genetic testing c. instructing the client on the appropriate decision to be taken d. knowing basic genetic terminology and inheritance patterns e. avoiding explanation of ethical or legal issues and concentrating on genetic issues

a, b, d. The responsibilities of the nurse while counseling the client include knowing basic genetic terminology and inheritance patterns and explaining basic concepts of probability and disorder susceptibility. The nurse should ensure complete informed consent to facilitate decisions about genetic testing. The nurse should explain ethical and legal issues related to genetics as well. The nurse should never instruct the client on which decision to make and should let the client make the decision.

The nurse is assessing a client at 12 weeks' gestation who reports enjoying her usual slow, long daily walk. The nurse should point out which recommendation to this client? a. Reduce walking to half a block daily. b. Continue this as long as she enjoys it. c. Stop and rest every block. d. Engage in aerobics for greater benefits.

b. Walking is an excellent exercise during pregnancy because it is low impact and increases venous circulation. Exercise should be maintained as long as it is comfortable, but intensity should not increase over what is normally performed.

A female community health nurse is to be accompanied by a male student nurse for the day. The female nurse asks parents who are scheduled for an upcoming home visit if the student nurse can observe the breastfeeding assessment to be conducted for the new mother and baby. The couple decline. What is the nurse's most appropriate next action? a. Reassure the couple that the student nurse will be professional. b. Discuss acceptance and tolerance with the couple. c. Ask the couple about their concerns. d. Honor the couple's preference.

d. When providing services in the homes of clients, nurses need to respect the family's values and cultural preferences related to the provision of services. The other responses negate the couple's wishes and could be seen as confrontational and not client centered.

What is a positive sign of pregnancy? a. positive pregnancy test b. fetal movement felt by examiner c. Hegar sign d. uterine contractions

b. The positive signs of pregnancy are fetal image on sonogram, hearing a fetal heart rate, and the examiner feeling fetal movement.

In an effort to improve the health status of women and their children, several national campaigns have been implemented. The campaign that reduced the incidence of sudden infant death syndrome (SIDS) by 50% was the: a. Back to Front Campaign. b. Prone No More Campaign. c. Back to Sleep Campaign. d. Front to Back Campaign.

c. Another success is the 50% reduction in cases of sudden infant death syndrome (SIDS) after initiation of the "Back to Sleep" campaign.

In the resolution phase of sexual response, which response would be most commonly noted in the male? a. Testes are elevated against the perineum b. Scrotal sac thickens c. Erection is lost d. Rectal muscles contract

c. During the resolution phase, which is the final phase of the sexual response cycle, the male will lose his erection, his scrotal sac will thin back out and the testes will descend back down. Rectal contractions occur during the orgasm.

A pregnant mother may experience constipation and the increased pressure in the veins below the uterus can lead to development of what problem? a. Varicose veins b. Umbilical hernia c. Hemorrhoids d. Gastrointestinal reflux

c. The displacement of the intestines and possible slowed motility of the intestines can lead to constipation in the pregnant woman. This, along with elevated venous pressure, can lead to development of hemorrhoids.

The nurse is assessing a client at her first prenatal visit and reports her LMP started December 1. Which date will the nurse predict for the EDD? a. October 7 b. September 8 c. July 7 d. August 8

b. According to Naegele rule, the estimated date of birth is September 8. Add 7 days and subtract 3 months to the LMP to determine the estimated date of birth.

What anatomic area should be examined when assessing Montgomery glands (Montgomery tubercles)? a. thorax b. abdomen c. breasts d. perineum

c. Montgomery glands (Montgomery tubercles) are sebaceous glands on the areola of the breasts and are prominent during pregnancy

A 28-year-old client in her first trimester of pregnancy reports conflicting feelings. She expresses feeling proud and excited about her pregnancy while at the same time feeling fearful and anxious of its implications. Which action should the nurse do next? a. Schedule the client a consult with a psychiatric health care provider. b. Determine if the client's significant other is experiencing similar feelings about the pregnancy. c. Provide the client with information about pregnancy support groups. d. Inform the client this is a normal response to pregnancy that many women experience.

d. The maternal emotional response experienced by the client is ambivalence. Ambivalence, or having conflicting feelings at the same time, is universal and is considered normal when preparing for a lifestyle change and new role. Pregnant women commonly experience ambivalence during the first trimester.

A nurse is teaching a new mother about self-care measures. Which action would the nurse do as the final step in this process? a. Document the teaching and effectiveness. b. Reassess the teaching plan. c. Develop goals for the future. d. Reinforce positive behavioral changes.

a. The steps of client and family education are similar to the steps of the nursing process: the nurse must assess, diagnosis, plan, implement, and evaluate. Part of the evaluation process is to document all actions taken and outcomes achieved.

Which physician is known as the "Father of Pediatrics"? a. Joseph Brennaman b. Ren Spitz c. Abraham Jacobi d. John Bowlby

c. Many view Abraham Jacobi, a Prussian-born physician, as the father of pediatrics.

There are many steps in the process of genetic counseling and testing. Put these steps in the correct chronological order from first to last. All options must be used. 1 assessment of family history 2 physical examination of parents 3 nuchal translucency screening 4 amniocentesis 5 explain results of genetic testing 6 support couple in adjusting to diagnosis

The correct order is as follows: 1) assessment of family history; 2) physical examination of parents; 3) nuchal translucency screening; 4) amniocentesis; 5) explain results of genetic testing; 6) support couple in adjusting to diagnosis.

A nurse is working in a community setting and is involved in case management. In which activity would the nurse most likely be involved? a. helping a grandmother to learn a procedure b. assessing the sanitary conditions of the home c. establishing eligibility for a Medicaid waiver d. scheduling speech and respiratory therapy services

d. Case management focuses on coordinating health care services while balancing quality and cost outcomes. The nurse would be most likely involved with scheduling speech and respiratory services, ensuring these services are integrated into the client's plan of care in a coordinated manner. Helping a person learn a procedure or assessing the sanitary conditions of the home and establishing eligibility are not activities associated with case management.

A pregnant woman arrives in the L & D unit following premature rupture of membranes along with her spouse, who insists on being present and demands only female nurses assess his wife. When obtaining history and assessment data, the spouse provides all the answers. What is the best approach for the nurse to handle this situation? a. Call security and then inform the spouse that staff members need to speak with the woman alone. b. Evaluate whether it is essential to stand up to the spouse or pacify him. c. Educate the spouse about his expected role in the birthing process. d. Act as an advocate for the woman who has a male-dominant partner.

d. In many cultures, the man is the dominant figure. In a strongly male-dominant culture, if approval for hospital admission or therapy is needed, the man would prefer to give this approval. Since the woman is a nondominant member of the family, the nurse may have to act as an advocate for her with a more dominant partner. Security would be viewed as a threat and not appropriate at this time. To gain information from the woman, the nurse may need to confront the spouse about his role in the birthing process. It is not appropriate to educate the spouse because this is a long-standing family environment, and this is not the time to try to change the family culture toward females.

A new single mother is sharing her concerns about the cost of daycare with her nurse. She is new to the area. The nurse recognizes that which trend is contributing to this client's concerns? Select all that apply. a. fewer extended family members to help care for a child b. 90% of women working outside the home c. the increase in the number of single-parent families d. the increased tendency for families to move from one part of the country to another e. the increasing use of the Internet to monitor health and ask health questions f. the increase in child abuse (child mistreatment) and intimate partner violence

a, b, c, d. In this client's case, moving to a new a new area, lack of extended family members to help care for the child, having to work outside the home, and being a single parent all contribute to her concern over the cost of child care. Increased use of the Internet to monitor health or ask health questions would not contribute to her concern. There is no evidence that child abuse (child mistreatment) or intimate partner violence is occurring in this case.

The nurse is caring for a child from a different culture. Which statement(s) by the nurse demonstrates an understanding of how culture impacts a client? Select all that apply. a. "Culture can impact who a client chooses to see for health care needs." b. "Culture can impact the diet a client follows." c. "When a client moves to a different city, the client follows the cultural practices that are prevalent in the new city." d. "A client's cultural beliefs can impact spirituality and/or religion." e. "A client's cultural beliefs should be incorporated into the care a client receives.

a, b, d, e. Culture is a set of traditions and beliefs held by groups of people. Beliefs, diet, language, values, religion and spirituality are included in culture. Some cultures utilize folk healers to provide health care. The nurse should incorporate a client's beliefs as much as possible into care. When a client moves to a different location, the client tends to bring cultural beliefs and does not necessarily follow all the cultural practices of the new location.

A nurse is admitting a female client to the cardiac unit diagnosed with a possible myocardial infarction. During the admission assessment, which symptoms would the nurse expect to find? Select all that apply. a. nausea b. unbalanced c. temperature 40°C (104°F) d. heart rate of 50 bpm e. pain in the right arm f. crushing chest pain

a, b, e. The nurse should look beyond the obvious "crushing chest pain" symptom that heralds a heart attack in men. Clinical manifestations of a heart attack observed in women include nausea, dizziness, irregular heartbeat, unusual fatigue, sleep disturbances, indigestion, anxiety, shortness of breath, and pain/discomfort in one or both arms. Chest pain is the most common heart attack symptom, but some women may experience it differently than men. Bradycardia is defined as the heart beating fewer than 60 times a minute.

What societal issues greatly influence delivery of maternal and pediatric health care? Select all that apply. a. Cost of health care b. Increase in surrogacy and adoption c. Low income of families d. Increased cultural and ethnic diversity of clients e. Increased number of children born in the U.S.

a, c, d. Demographic trends such as a decreased number children being born, combined with an increase in a multicultural society seeking health care, are affecting the delivery of maternal-child health care. Poverty and the cost of health care also play a major role in influencing health care delivery for both women and children.

A nurse is conducting a teaching session on sudden infant death syndrome (SIDS) for expectant parents. Which information should the nurse include? Select all that apply. a. Sharing a room allows for monitoring of the infant. b. Co-bedding or sharing a bed creates parental bonding. c. Place the infant on his or her back to sleep. d. Maintain neutral temperatures and avoid overheating. e. Allow the infant to sleep with a bottle.

a, c, d. Sharing of a room allows for monitoring and bonding of the infant as well as ease of feeding. Placing the infant on his or her back to sleep is the recommended sleeping position for all infants until 12 months of age, or until they can change their own position during sleep. Infants are not able to regulate their temperature; therefore, overheating can increase their risk for SIDS. Co-bedding increases the risk for accidental suffocation, and allowing an infant to sleep with a bottle increases risk for aspiration and infection.

When caring for woman who speaks a different language than one's own, the nurse must ask for an interpreter before having the client sign which forms to ensure clarity can be confirmed? Select all that apply. a. surgical permit b. dietary preferences c. end-of-life care d. consent to blood transfusions e. smoking cessation policy

a, c, d. When caring for clients who speak a different dialect or language, the nurse should always ask them to repeat instruction to be certain it was interpreted correctly. The nurse should repeat what the client said so he or she can confirm understanding the nurse correctly. The nurse should not be reluctant to ask for an interpreter to help clarify forms, such as consent for blood transfusion, informed consent for surgery, or wishes regarding right-to-life care, as necessary. Dietary preferences and smoking cessation policy are not as important as the other forms that result in invasive procedures.

A nursing instructor is teaching a session investigating infant mortality as a standard measurement of the quality of health care in the country. The instructor determines the session is successful when the students correctly choose which factors as contributing to the high numbers of deaths in the United States? Select all that apply. a. Preterm births b. Maternal age 30 to 34 years c. Male gender of infant d. Maternal complications e. Congenital deformities and chromosomal abnormalities

a, d, e. Many factors contribute to the high infant mortality rates in the United States. Some of the most common factors are low gestational age and prematurity, congenital deformities and chromosomal abnormalities, and maternal complications such as cervical insufficiency, multiple births, and premature rupture of membranes. Female infants have lower mortality rates than males, and the best outcomes are in mothers who give birth between 30 and 34 years of age.

The nurse should carefully screen a client who insists on using only oral contraceptive pills (OCPs) for which contraindication? a. deep vein thrombosis b. gastrointestinal disease c. impaired absorption d. use of alcohol

a. The nurse should screen the client for deep vein thrombosis (DVT), migraine headaches, neurological symptoms, coronary artery disease or cerebral vascular disease, severe diabetes, hypertension, liver disease, breast or endometrial cancer, and unexplained vaginal bleeding when oral contraceptive pills (OCPs) are used. Gastrointestinal diseases are contraindicated in the use of progestin-only pills. OCPs are not contraindicated in clients who drink alcohol. Impaired absorption is a contraindication for the use of progestin-only pills (mini-pills).

A 4-year-old adopted child has begun to ask questions about when she was born. Which suggestions by the clinic nurse would be considered the most appropriate answer for this child related to her birth? Select all that apply. a. Explain to the child that she grew inside another woman, but after the birth she was given to her adoptive mom and dad to raise. b. Tell the child that her biological mom could not care for her after birth because she was HIV positive. c. Inform the child that her biological mom was in prison and would not be able to care for her for a long time. d. Explain that her biological mom could not care for her so she was given away. e. Avoid criticizing the biological parents but reinforce how much the adoptive mom and dad love them.

a, e. At least by 4 years, children are old enough to fully understand the story of their adoption: they grew inside the body of another woman who, because she could not care for them after they were born, gave them to the adopting parents to raise and love. It is important for parents not to criticize a birth mother as part of the explanation because children need to know, for their own self-esteem, that their birth parents were good people and they were capable of being loved by them, but things just did not work out that way. At age 4, children do not understand HIV status, not being able to provide for the needs of an infant, or prison terms.

A nurse is caring for a Turkish American client. The nurse understands that there could be major cultural differences between herself and the client. The nurse contemplates assigning this client to a staff member who is of the same culture as the client. What is a potential consequence? a. stereotyping of the client b. ensuring better care and understanding c. helping in assessing client's culture d. building a better nurse-client relationship

a. A nurse who thinks stereotypically may assign a client to a staff member who is of the same culture as the client because the nurse assumes that all people of that culture are alike. The nurse also may believe that clients with the same skin color may react in the same manner in similar social situations. Because stereotypes are preconceived ideas unsupported by facts, they may not be real or accurate. In fact, they can be dangerous because they are dehumanizing and interfere with accepting others as unique individuals.

Pediatric nurses are developing more home care and community-based services for children with chronic illnesses because: a. increasing numbers of children live with chronic disabilities due to advances in health care that allow children with formerly fatal diseases to survive. b. uninsured families can access these services. there are fewer positions for nurses in institutions providing acute care. c. disability and chronic illness have increased with the appearance of more genetic abnormalities. d. more disabilities are occurring in children because many women are delaying pregnancy and giving birth to babies later in life.

a. Advances in health care have led to more children living with chronic illness or disability. The statements about genetic disease and older women may contain some truth but have only added a few people to the chronic illness total. Acute care pediatric nursing positions are decreasing in community hospitals but are more available in medical centers. Uninsured families may or may not be able to access nonhospital care.

A nurse is conducting a presentation for a group of pregnant women about appropriate health promotion strategies to address issues related to infant mortality. Which strategy would the nurse encourage to reduce the infant's risk for infection after birth? a. breastfeeding b. sleeping on the back c. folic acid supplementation d. newborn development support groups

a. After birth, other health promotion strategies can significantly improve an infant's health and chances of survival. Breastfeeding has been shown to reduce rates of infection in infants and to improve long-term health. Emphasizing the importance of placing an infant on his or her back to sleep will reduce the incidence of SIDS. Newborn development support groups will help provide education about normal child development and child rearing. Folic acid supplementation is used during pregnancy to prevent neural tube defects.

The nurse is caring for a 7-year-old boy and his family, who are immigrants. Which intervention will most significantly affect the success of the care provided? a. Communicating with sensitivity using understandable terms b. Inquiring about common health problems in their home country c. Asking about transportation to the appointment d. Referring them to state and local aid programs

a. Being understood is essential to the provision of all nursing care. An interpreter may be needed. Speaking slowly and using simple terms is also useful. Inquiring about common health problems in their home country, asking about transportation, and helping them access aid programs are all secondary to and dependent upon effective communication.

A clinical nurse specialist is conducting a review class for a group of nurses about cultural competence and cultural diversity. Which term reflects the view of the world and set of traditions of a specific social group passed down through generations? a. culture b. race c. values d. ethnicity

a. Culture consists of the world view and group of traditions shared by a social group and passed down through generations.

A nurse admits a young boy who lives in a homeless shelter with his pregnant teenage mother. What would the nurse realize is a priority when planning this boy's case management? a. The toddler's situation puts him at risk for higher incidence of disease and limited access to appropriate care. b. The toddler's visits to the emergency department assist in lowering his incidence of disease. c. There will be adequate coordination of other available services with visits to the emergency department. d. Medicaid coordination of health services from the emergency visits will cause a lower incidence of disease.

a. Factors that may increase morbidity include homelessness, poverty, chronic health disorders, and barriers to health care. Children who live in poverty have a higher incidence of disease, limited coordination of health services, and limited access to health care, except for visits to the emergency department.

A home care nurse is visiting a pregnant client from the Arab culture. During the health history the husband frequently answers questions for the client. How should the nurse respond? a. Continue with the health history. b. Stop the interview. c. Ask the husband to leave. d. Specifically ask the client to answer.

a. In being culturally aware the nurse will recognize that the client and her husband come from a culture that is a patriarchal structure. The nurse should continue with the health history.

The nurse is providing care to a woman who has just given birth to a healthy term neonate. The woman's partner arrives and asks about the neonate's status. Which action by the nurse would be appropriate? a. Check the medical record for written client approval with whom to share information. b. Ask the partner for identification first before sharing any information. c. Answer the partner's questions honestly and without hesitation. d. Tell the partner that no information can be shared with him or her at this time.

a. In maternal and newborn health care, information is shared only with the client, legal partner, parents, legal guardians, or individuals as established in writing by the client or the child's parents. This law promotes the security and privacy of health care and health information for all clients. Therefore, the nurse needs to check the medical record for written documentation that allows the partner to have this information. Any other action would be inappropriate.

The nurse is caring for a 2-year-old boy who needs a lumbar puncture. His mother is present. What would prevent informed consent from being obtained? a. learning the mother is not the custodial parent b. finding out the mother is younger than 18 years of age c. determining the mother cannot read the form d. establishing the mother was never married

a. It would not be legal for this mother to give consent. A mother younger than 18 years of age or never married may not be a problem in most states because she would be considered autonomous. The physician or nurse could read the consent form to a mother who cannot read plus carefully explain the medical information in terms she understood.

The nurse is running an education program for early grade-school children. Which topic would address the number one cause of death for this age group? a. The importance of crossing streets safely b. The importance of immunizations c. Prevention of infection and communicable disease d. Exercise and good nutrition

a. Motor-vehicle accidents are a leading cause of death in this age group.

A nurse working at a child health clinic is involved in primary prevention activities. Which activity will the nurse perform in this role? a. teaching about healthy food choices b. performing hearing screenings c. reviewing laboratory test results d. assisting with physical therapy exercises after knee surgery

a. Primary prevention involves health-promoting activities to prevent the development of illness or injury, such as teaching about healthy food choices. This level of prevention includes giving information regarding safety, diet, rest, exercise, and disease prevention through immunizations and emphasizes the nursing roles of the educator and client advocate. Secondary prevention focuses on health screening activities that aid in early diagnosis and encourage prompt treatment before long-term negative effects arise, such as hearing screenings and reviewing laboratory test results. Tertiary prevention involves health-promoting activities that focus on rehabilitation, such as physical therapy exercises after surgery, and providing information to prevent further injury or illness.

Before World War II women moved from home to hospital births. What was the primary reason for that shift in maternity care? a. They were convinced that setting would improve birth outcomes. b. They were convinced that setting would provide them with natural birth. c. They had no other options as many could not afford midwives. d. They were convinced that setting would lessen the pain of labor.

a. Prior to World War II, American women moved from home to the hospital for birth in part because they were convinced that setting would improve birth outcomes. Women who labored and gave birth at home were traditionally attended to by relatives and midwives. Many women were attracted to hospitals because this showed affluence and hospitals provided pain management, which was not available in home births.

When a woman is admitted to the labor-and-delivery unit, her husband says he is going to work and asks you to call when the baby is born. Your best response to this husband would be to a. ask him if he knows that he can stay with his wife during labor. b. tell him that all fathers now stay with their wives during labor. c. tell him he is missing out on the opportunity of a lifetime by leaving. d. insist he stay with his wife during labor because she will need his support.

a. Respect for cultural values is important for developing effective nurse-client relationships.

In order to advocate for children and families, the nurse must first acknowledge that the basic system in which health behavior and care are organized, secured, and performed is the: a. family. b. community. c. health care facility. d. government.

a. The family is the basic system in which health behavior and care are organized, secured, and performed. In most families, the parents or guardians, as advocates for their child, provide health promotion and health prevention care, as well as primary management of care when the child is sick. Parents and guardians have the prime responsibility for initiating and coordinating services rendered by health professionals.

A nurse is analyzing a journal article presenting statistics concerning newborn and infant health. Which condition has been determined to be the most likely cause of infant mortality in the United States? a. congenital abnormalities b. increased birth weight c. breech presentation d. shoulder dystocia

a. The main causes of early infant death in the United States include problems occurring at birth or shortly thereafter such as prematurity, low birth weight, congenital and chromosomal anomalies, sudden infant death syndrome (SIDS), respiratory distress syndrome, unintentional injuries, bacterial sepsis, and necrotizing enterocolitis. Increased birth weight, breech presentation, and shoulder dystocia are not recognized as potential causes.

The nurse is caring for a pregnant client in a prenatal clinic who states, "I want to view this labor and delivery as a natural process....not like an illness." How will the nurse respond? a. Refer the client to a midwife in preparation for a birth in a birthing center. b. Schedule the client for a tour of the women's unit at a large urban hospital. c. Offer education regarding how to best utilize pharmacologic pain medication during labor. d. Provide the names of several obstetricians in the local area for consideration.

a. The nurse recognizes the client is seeking a birth with little intervention. This client would benefit from a consultation with a midwife in preparation for a home birth or in a birthing center birth. An obstetrician would be beneficial for a woman seeking specialized care with more interventions, not less. A tour of the local hospital would be beneficial if the woman was planning a hospital birth with interventions traditionally available at a hospital. Pain control during labor is considered an intervention and would not benefit a woman wanting limited intervention.

The nurse is admitting a client to the emergency clinic and notes that the client's gender expression is different than the gender noted in the driver's license and insurance documentation. What is the nurse's best response? a. The nurse should communicate with the client based on the client's preferred pronoun. b. The nurse should communicate with the client by the gender indicated on the driver's license. c. The nurse needs to clarify the conflict between the driver's license and the client's appearance. d. The nurse should refer to the client using the pronoun based on the legal documented gender. e. The nurse should avoid using any gender specific pronouns or anything that relates to either gender.

a. The nurse should take care to refer to the client by the client's preferred pronoun. The gender indicated in a person's insurance coverage documentation and other legal documents may be discordant with the person's gender identity, and many electronic health records do not allow space for gender identity as opposed to natal sex, making correctly addressing the client more challenging. Building a trusting relationship with all clients is a critical step. It is unnecessary to clarify the conflict between the driver's license and appearance at this time, as this will only cause distrust and emotional harm to the client. Avoiding using gender specific pronouns is a form of avoidance and can also cause distrust and emotional harm.

Advocacy for the client is an important aspect of community-based nursing. What is one way a nurse can advocate for a pediatric client? a. Assist the family to apply for Medicaid or other forms of health care reimbursement. b. Report a new case of whooping cough to the county health authorities. c. Arrange for educational events for the local hospital staff. d. Assure a young mother that the nurse will not report the suspicious bruises on her toddler to social services.

a. The nurse working in a community setting may often develop a long-standing relationship with families because of the continuous nature of client contact in an outpatient, school, or other setting. This type of relationship may allow the nurse to advocate for the client on a broader scale in health and welfare issues. Examples of interventions include helping the family apply for Medicaid or other forms of health care reimbursement. Reporting new cases of whooping cough is done to state authorities, not county authorities, and is not considered advocating for a client. Arranging educational events for the local hospital staff is not a community-based function. Assuring a mother that the bruises on her toddler will not be reported to social services—when it is mandated that any suspected case of child violence be reported—is not advocating for the child.

A nurse is visiting the home of a mother and her newborn. The woman's family members are present when the nurse arrives. What is the nurse's best approach to the situation? a. Assess the family members' interactions with the newborn and one another. b. Do brief maternal and infant assessments with the family members present. c. Reschedule the assessment visit for another time without family present. d. Ask the family to leave to meet to perform the assessments privately.

a. The philosophy of family-centered care recognizes the family as the constant. The health and functioning of the family affect the health of the client and other members of the family. Family members support one another well beyond the health care provider's brief time with them, such as during the birth process or during a child's illness.

A nurse is speaking with a client who has just learned that she is pregnant with her first child. The nurse reads in the client's chart that she does not drink alcohol on a regular basis. However, the nurse decides to go ahead and warn the client about the dangers of drinking alcohol while pregnant. Which phase of health care would this action be classified as? a. health promotion b. health maintenance c. health restoration d. health rehabilitation

a. This action is an example of health promotion, which may be defined as educating parents and children to follow sound health practices through teaching and role modeling. Health maintenance is intervening to maintain health when risk for illness is present. Health restoration is using conscientious assessment to be certain symptoms of illness are identified and interventions are begun to return the client to wellness most rapidly. Health rehabilitation is helping prevent complications from illness, helping a client with residual effects achieve an optimal state of wellness and independence, and helping a client to accept inevitable death.

A parent brings a foster child to the clinic for immunizations. What is the first responsibility of the nurse? a. verify legal documentation b. confirm identity with foster agency c. have case worker present d. obtain authorization to treat

a. When a child is placed in foster care the foster parents are given legal documentation of this placement and of their ability to seek and consent to treatment. This documentation should be confirmed before any treatment is rendered. A child placed with a foster parent is not required to have a case manager present at medical visits. The foster parent can take the child for care whenever needed. The authorization to treat is part of the legal documentation provided to the foster parent, so it is not necessary to obtain a new consent at every clinic visit.

A client receiving home care is on bed rest. She says to the nurse, "I feel so useless like this. I'm not used to being so inactive." What response from the nurse might help the client to better cope with her situation? a. "Are there hobbies you enjoy that you can do while resting that you never get a chance to do otherwise?" b. "Think about how much faster you'll recover by following directions." c. "Let me talk to the primary care provider to see if the restrictions can be eased back at all." d. "I would love the opportunity to sit back and relax. Take advantage of this while you can."

a. Women and children on bed rest react in several ways, but many report feeling "tied down," "like a prisoner," and "as if I'm missing out." One solution to help them cope with the stress of the experience is for them to keep busy or use their time to learn a new skill. Most women can name activities they would like to do but have never had time to begin to do. For example, if a woman likes to read but does not always have the time, bed rest at home may provide an ideal time for her to catch up on her reading.

An expectant client asks the childbirth educator about the advantages of going to a birthing center for delivery. Which are the nurse's best responses? Select all that apply. a. Birthing centers are a good option for every pregnant client who wants this alternative. b. Birth centers are designed to be more home-like and comfortable with fewer interventions. c. Birth centers focus on pregnancy as a state of wellness and on women and families as a whole. d. The rate of cesarean delivery for clients who deliver in a birth center is less than for those who deliver in hospitals. e. Care is usually provided by midwives for pregnant clients with lower risk of complications. f. Birth centers have transfer agreements with local hospitals if complications occur.

b, c, d, e, f. Approximately 0.3% of births occur in independent birth centers, most of which are freestanding and not located in a hospital. Midwives attend most of these births. Birth centers are designed to be more home-like and comfortable and cater to women with low-risk pregnancies. Approximately 85% of women who are pregnant are eligible to deliver in a birth center rather than a hospital. Birth centers have standing transfer agreements with hospitals, and approximately 16% of women, 82% of whom are primigravidas, or infants are transferred to the hospital from the birthing center before, during, or after the birth. Birth centers focus on pregnancy as a state of wellness and on women and families as a whole. The rate of cesarean delivery for women choosing to deliver in a birth center is approximately one fifth that of women who deliver in hospitals, and neonate morbidity and mortality rates are equivalent to those of low-risk deliveries in hospitals.

A pregnant woman reports she does not see the value in attending prepared childbirth classes because she plans to have an epidural to manage the discomfort from the contractions. What information should be included in responses by the nurse? Select all that apply. a. "You are correct since the focus of these courses is on pain management." b. "There are many different topics included in prepared childbirth classes aside from pain management." c. "Preparation for labor and birth can enhance your experience and coping abilities." d. "You can learn tools to assist you in the labor process both before and after your epidural." e. "You are limiting your options by not considering attending the classes."

b, c, d. Prepared childbirth education courses provide information for prospective and expecting parents. The information in prepared childbirth education is not limited to pain management. Courses can include other topics such as potential procedures during the labor process, newborn care and postpartum care. Education and an accompanying understanding of the labor and birth experience can improve the coping skills of the mother and her partner. Advising the woman that she is limiting her options is argumentative and not appropriately stated.

A healthy client without a primary care provider is exploring the options available for a health care provider to assist with her pregnancy. Which health care provider can the nurse point out as a best option? a. Women's health nurse practitioner (NP) b. Certified nurse midwife c. Lay midwife d. Clinical nurse specialist (CNS)

b. A certified nurse midwife is a nurse with advanced practice training in the care of women, specific to pregnancy and birth. A women's health NP would not be able to deliver the infant. A CNS is an advanced practice role but not specific to the care of pregnancy women. The lay midwife has no formal education.

While caring for a hospitalized child, the nurse notes the parent does not take into consideration the wishes or opinions of the child. During a period of discussion, the parent states "My child needs clear rules and knows I expect my child to do as I say." This is consistent with which style of parenting? a. authoritative b. authoritarian c. permissive d. uninvolved

b. Authoritarian parenting style consists of the parents making the rules for the child to follow. There is little to no flexibility or decision making by the child. Authoritative parenting is also referred to as democratic parenting. In this style of parenting, there are rules and expectations of behavior but the parents embrace the individuality of the child and do allow some input by the child. Permissive parents have little control over the behavior of their children. Rules are often inconsistent, unclear or nonexistent. Uninvolved parents are indifferent. They do not provide rules or standards.

A nurse is reviewing a labor plan with a client who has been admitted to the labor and birth unit. The client states that she has been drinking a significant amount of herbal teas lately to help with uterine contractions. Which is the priority action by the nurse? a. Ask about other complementary and alternative therapies. b. Determine the type of herbal teas recently consumed. c. Ask the client when she last ate or drank anything. d. Prepare the client for a fetal nonstress test.

b. Certain herbal teas can be used during pregnancy, and most are made with flower or berries that are safe for both mother and fetus. To determine if the herbal tea is safe or has had any effect on the status of the birth, it is important for the nurse to find out what type of tea the client has been consuming and in what quantities.

A community-based nurse is assessing the needs of a family of four, which includes a physically challenged 9-year-old. Which activity would the nurse prioritize to be an advocate for this family? a. Ensure case history is complete for all family members. b. Establish eligibility for assistive devices for child. c. Ensure the client follows physical therapy recommendations. d. Train the school nurse on the needs of the child.

b. Client advocacy is acting on behalf of the client. Making calls to arrange for special equipment is one role of a nurse who is acting on behalf of the client. A case history would be taken at the initial visit to the treatment center by the attending nurse. The client's progress in physical therapy is to be noted by the therapist or PT assistant. The community-based nurse should not have to train the school nurse but would share the needs of the child so that the school nurse is prepared to provide appropriate care.

Which intervention best demonstrates the L & D nurse is respectful of a client who is deaf and in early labor? a. Write down information on a piece of paper that the client can keep. b. Seek assistance from another health care professional who can converse in sign language. c. Utilize the labor coach so he or she can interpret and relay information to the client. d. Utilize hand signals like in charades to try to communicate important pieces of information.

b. Cultural differences occur across not only different ethnic backgrounds but also different sociodemographic groups. A parent who has been deaf since birth, for example, expects her deaf culture to be respected by having health care professionals locate a sign language interpreter for her while she is in labor. If an interpreter cannot be located, writing down questions and answers is an alternative, assuming the parent has the ability to read and comprehend while dealing with labor contractions. Use of friends and family members is not considered to be the best option since many family/friends find it difficult to interpret medical terms. Hand signals can help in an emergency; however, it not the best way to interpret questions/comments from a laboring woman who is deaf.

Female infant, aged 23 days, brought in for 1-month checkup. Birth data as follows: Vaginal birth without complications. Birth weight 3348 g. Current weight 4050 g. Length: 21 in (53 cm) at birth. Formula fed since birth. Takes bottle "well" per mother. Mother reports "fussy" during diaper changes, otherwise sleeps "well." Maternal age at birth was 38. Prenatal care with Dr. Drake began at 10 weeks gestation and continued. Mother reports continued follow-up with Dr. Donahue, an HIV specialist for care management of her HIV status and disease. The nurse is reviewing the chart of a pediatric client (above). What data support a high risk for infant mortality? a. weight b. maternal factors c. nutrition d. prenatal care

b. Infant mortality rates are affected by many factors, including several from the maternal aspect. Maternal HIV infection would put the infant at risk for mortality. The birth weight for this infant is average, and the infant is gaining weight as planned/expected, so nutrition is adequate, and weight does not support risk for infant mortality. A maternal age of less than 18 years or more than 40 years would put the infant at risk, as would lack of early or nonexistent prenatal care. Obtaining prenatal care at 10 weeks' gestation is a positive outcome. This mother is 38 years of age.

The nurse is assessing a 9-year-old boy during a back-to-school checkup. Which finding is a factor for childhood injury? a. Records show child weighed 2,450 g at birth. b. Mother reports she has used alcohol and drugs. c. The parents adopted the boy from Guatemala. d. Mother reports the child is hostile to other children.

b. One of the factors associated with childhood injuries is parental drug or alcohol use. This is the leading cause for child mortality. Low-birth-weight babies are at higher risk for infant mortality. Foreign-born adoption is a factor for childhood morbidity. The child's hostility toward other children may be an environmental or psychosocial factor for childhood morbidity.

Cost containment in the health care field has become a sought-after necessity because of rising costs and the influence of managed care. Nurses are instrumental in assisting with cost containment of health care by prioritizing which strategy? a. Alternative birth systems b. Health promotion c. Client teaching d. Quality improvement

b. Specific cost-containment strategies that nurses have been instrumental in implementing include health promotion, case management, and critical care paths. Health promotion is accomplished by methods such as using alternative birth systems, providing client teaching, and quality improvement.

The home care nurse visited a newly assigned primigravida client with preeclampsia. When conducting the assessment and teaching, the client continued to cry and state she was in disbelief this was happening to her. When should the nurse schedule the second visit with the client? a. In seven days; the order was for once a week. b. The next day; the client is not coping well. c. In three days; this is a halfway point in the week. d. In five days; this will allow the client to adjust.

b. The client is in a state of disbelief regarding the diagnosis. The nurse will need to assess to see if the teaching that was conducted was retained and to evaluate whether this client is an appropriate client for home care.

A 15-year-old client has just given birth and states that she does not want her infant to receive any newborn vaccines. What is the appropriate action for the nurse to take? a. Administer the newborn vaccines. b. Withhold the vaccines. c. Call the primary care provider. d. Ask the grandparents for permission.

b. The client would frequently be considered emancipated and therefore legally able to make legal decisions regarding the health care of the infant. The nurse should withhold the vaccines but inquire as to the reason for no vaccination.

What makes the area of family nursing a difficult arena in which to provide care at times? a. the presence of family in the hospital rooms b. family members who might not agree on treatments and a plan of care c. family-centered care plans and use of community areas in acute care settings d. nurses having different values from families

b. The family may not agree of the plan of care or treatment plans with each other or for their children. This is a difficult situation as the children do not have their rights to make their own decisions and the family needs to agree.

A nurse is reviewing the statistical outcomes related to fetal deaths nationwide and notes that the numbers have changed over the last several decades. The best explanation would be: a. expectant mothers are seeking prenatal care later, resulting in decreases in the number. b. improved prenatal care has reduced the numbers of fetal deaths. c. increases are noted because of the growing number of pregnancies to teen mothers. d. increases are noted due the increasing number of older mothers becoming pregnant.

b. The number of fetal deaths in the United States has fallen. This has been attributed to the improvement in the delivery of prenatal care.

A woman in active labor begins to recite a lullaby to "call the baby outside." Which action by the nurse caring for this woman is most appropriate at this time? a. Administer IV pain medication. b. Allow the client to perform a ritual. c. Ask the woman the importance of this lullaby to her culture. d. Ask the family to join in with the singing of this lullaby.

b. The nurse can acknowledge and celebrate a client's culture without stereotyping by such actions as ensuring that the client has the opportunity to perform her cultural traditions during labor, such as reciting a lullaby to "call her child outside." There is no indication that pain medication is needed at this time. When one respects the culture, quizzing about the purpose or importance of reciting this lullaby is inappropriate.

The nurse visits a pediatric client at home for complications of sickle cell anemia. Upon entering the home, the nurse notices a significant number of candles being burned, statues surrounding the client, and herbs around the room. What is the most appropriate response by the nurse? a. Call the social worker. b. Proceed with the visit. c. Call 9-1-1. d. Remove the items from the room.

b. The nurse should procced with the visit as scheduled. Many clients will use complementary and alternative therapies, which include spiritual and faith practices in the treatment of illness. The nurse should understand this to be the case.

A woman is in the second stage of labor, 8-cm dilated and 90% effaced. She keeps very quiet without expressing any outward signs of pain. Which intervention by the nurse would be most appropriate in this situation? a. Administer pain medication without asking the woman. b. Allow the woman to labor according to her cultural beliefs. c. Monitor the woman to attempt to predict when it is time to push. d. Review the nursing goal to minimize the amount of pain with birth.

b. The way people respond to pain is an example of a trait that is heavily influenced by culture. Some women and children scream with pain; others remain stoic and quiet. Both are "proper" responses, just culturally different. The best nursing intervention is to allow the woman to labor the way that is best for her. Administering pain medication without permission is unacceptable nursing practice. The nurse will have to assess the woman frequently, especially noting if any blood is coming from the vagina and performing frequent vaginal exams to know when it is time to push. "A close watch" could mean just looking into the room frequently. In some cultures, being stoic is expected behavior, so minimizing the pain may not be the goal of the woman.

The nurse is caring for a child. The nurse is trying to ensure that the family's cultural practices are supported. Which statement by the nurse indicates a lack of understanding regarding cultural competence? a. "Are there any dietary practices related to your culture that we should know about?" b. "Since your child is only 8, I doubt that your child has any cultural practices we need to be aware of." c. "Most cultures have certain practices that are important to them. We want to honor any that we can." d. "Is there a particular religion that we should note in your chart that may impact your care?"

b. Typically, a child begins to understand his or her culture at approximately 5 years of age, so stating that the child does not have any cultural practices at the age of 8 is inaccurate. Diet, cultural practices, and religious practices related to culture are important for the nurse to know so that the nursing staff can support as many of these practices as possible.

The nurse is assessing an infant girl at her first well-baby visit. The nurse also observes the actions of the 6-year-old brother and parents who share that she was the best anniversary present they received this year. Which type of family will the nurse conclude these individuals represent? a. cohabitation b. adoptive c. immediate d. extended

c. An immediate family is defined as consisting of parents and children. An extended family consists of one or more nuclear families plus other relatives, often crossing generations to include grandparents, aunts, uncles, and cousins. In the cohabitation family, couples live together but are not married. The children in this family may be children of earlier unions, or they may be a result of this union. The adoptive family is created when parents take in children who are not biologically theirs but raise them as if they were.

A group of women are attending a community presentation regarding the leading health concerns of women. Which interventions should the nurse recommend to have the greatest impact on the leading cause of death? a. yearly gynecological exams b. prompt attention to respiratory tract infections c. weight control and being knowledgeable about family history of cardiovascular disease d. regular neurologic exams to note any cognitive or behavioral changes early

c. Cardiovascular disease is the leading cause of death of women in the United States. Interventions that address reduction of this risk would be a priority. Elevations in death rates are in part attributed to the difficulty recognizing cardiovascular concerns in women. The second leading cause of death in women is cancer, specifically lung and cervical. Lower respiratory tract infections have increased over recent years as a cause of death in women, but they are not the number one cause. Alzheimer disease, although impacting the mortality rates of women, is not the greatest cause of death.

The nurse is reviewing the nursing care plan on a pediatric client admitted to the hospital with a respiratory illness. Which goal is written correctly? a. The client will breathe better by the end of the shift today. b. The client will walk 200 feet (60 meters) with an oxygen saturation of 95% or greater without oxygen. c. The client will have an oxygen saturation of greater than 95% on room air within 8 hours. d. The client will have a better respiratory rate and better oxygen saturation by tomorrow at 0800.

c. Goals must be specific, measurable, and have a time frame. The goal identified as "The client will have an oxygen saturation of greater than 95% on room air within 8 hours" includes both measurable, objective information (the oxygen saturation on room air and a time frame). It is client-centered and appropriate. The goals of "breathing better" and "better respiratory rate and oxygen saturation" are vague and do not have a measurable outcome. What is considered "better" is subjective and not measurable. Not having a time frame makes the goal inaccurate as well. The goal of the client walking a specific distance on room air would be a good goal if a date/time frame were included in the goal.

A nurse is caring for a 31-year-old pregnant client who is subjected to intimate partner violence. The client has developed a feeling of hopelessness and does not feel confident in dealing with the situation at home, which makes her feel suicidal. Which nursing intervention should the nurse offer to help the client deal with her situation? a. Counsel the client's partner to refrain from subjecting his partner to violence. b. Help the client understand the legal impact of her situation to help protect her. c. Provide emotional support to empower the client to help herself. d. Introduce the client to a women's rights group.

c. In cases of intimate partner violence, nurses can serve their clients best by not trying to rescue them but by helping them build on their strengths, providing support, and empowering them to help themselves. Counseling the client's partner against violence, helping the client know the legal impact of her situation, and introducing the client to a women's rights group to garner support are not the best ways of serving the client.

The nursing instructor is discussing the infant mortality rate with a group of students. The instructor determines the discussion is successful when the students correctly choose which action as a priority to help decrease infant mortality in the United States? a. Improving maternal nutrition during pregnancy b. Spending more money on health care for infants and mothers c. Improving accessibility to postnatal health care for infants d. Mothering classes for all primiparous mothers

c. Infant deaths are due to many factors, including congenital malformations, deformations, and chromosomal abnormalities; disorders related to short gestation and low birth weight; and newborns affected by maternal complications of pregnancy. The mother's nutritional status would affect a limited number of infants. Making postnatal health care available and affordable would have the greatest impact on improving infant health and reducing the number of infant deaths. Spending more money on health care has proven to not improve infant survival rates postnatally. Although providing for mothering classes and improving maternal nutrition would be a good idea, these measures will not positively impact infant survival rates.

A nurse is developing a plan of care for a client of Muslim faith. Which action demonstrates an understanding of providing culturally competent care? a. The client is asked to encourage family members to bring in special foods. b. The client's traditional healing and health practices will be assessed for implementation. c. The client is asked if there are any meal preferences to be included in the care plan. d. The client will be referred to the hospital chaplain for spiritual support.

c. It is important for all nurses to incorporate the client's traditional healing and health practices with conventional medicine. Some clients may prefer certain foods or drinks when they are ill. Diets may be different and need to be considered during the process of determining the appropriate course of treatment. Family members may not be able to bring in foods necessary for the client. Nurses should be competent in their ability to provide care to diverse populations without referrals.

The primary care provider has prescribed a biophysical profile (BPP) test for a pregnant woman. The nurse quickly discovers that the client and spouse do not read or write the dominant language. Which is the best way for the nurse to obtain their consent for the test? a. Read the information on the consent form slowly, in the dominant language, and instruct the client to sign the consent. b. Allow the primary care provider to deal with this situation. Assume that the client and spouse will understand. c. Locate a medical interpreter to translate before the couple signs the consent. d. Use nonverbal language (pictures, gestures) to explain what will happen and ensure that the couple understands.

c. Many families that do not speak the dominant language use the health care system. Interpreters or cultural health brokers can assist with translating services to facilitate effective communication and informed consent. It is prudent to identify a staff member or hospital visitor who is fluent in the language. Consent must be obtained in the language of origin. The other options do not ensure that the client and spouse have understood everything about the BPP and can truly give consent to the procedure.

A nurse is conducting an in-service program for a group of nurses on the health of women and their families, incorporating information from Healthy People 2030. The nurse determines that additional discussion and clarification is needed when the group makes which statement? a. "Obesity, drug use, and smoking are major problems." b. "Maintaining a physically active lifestyle is strongly encouraged." c. "Immunizations may be the cause of some illnesses being seen currently." d. "Environmental quality is directly related to health care."

c. Obesity, drug use, and smoking are major concerns. Physical activity and environmental quality are also on the list of leading health indicators in Healthy People 2030. The students who have determined that immunizations are the causes of diseases need clarification since maintenance of immunizations is one of the health indicators listed in the report.

The nurse is working with the parents of an infant and a toddler in the clinical setting. What statement by the parent would alert the nurse to assess the children further? a. "We enjoy taking walks in the park and playing outside as much as possible." b. "We have baby gates installed at the top and bottom of our stairs in our home." c. "We live in an old house, and we are planning to renovate it when the kids are older." d. "We have to hire babysitters sometimes because our parents are not able to help babysit."

c. Older homes are at risk for having lead-based paint (usually those built before the mid to late 1970s) and lead pipes used in plumbing. The nurse would further assess for the children's exposure to lead pipes, paint, paint chips, lead dust, and soil containing lead dust and paint chips. Additionally, during a renovation, the children will be exposed to areas with lead paint and/or dust. The other options are appropriate and demonstrate information only and not a need to further assess.

A nurse is caring for a child. Which individual would the nurse identify as being primarily responsible for initiating and coordinating health care? a. social worker b. case manager c. parents d. community

c. Parents and guardians have the primary responsibility for initiating and coordinating services rendered by health professionals. A social worker or case manager may be involved but are not primarily responsible for the child's care. The community provides programs to promote and support children's health.

An 8-year-old with cystic fibrosis has had a noted decline on the growth chart. Which nursing intervention is best for maintaining adequate nutrition? a. Provide high caloric meals to the client's liking. b. Delay pancreatic enzymes until food enters the small intestine. c. Encourage high calorie, high protein snacks. d. Limit sodium to a 2 gram sodium restricted diet

c. The best nursing intervention is a high calorie, high protein snack. Calories can be obtained from non-nutritious foods. It is not only that the client needs calories for energy, but nutrition needs to be present. Pancreatic enzymes aid in digestion so they need to be available for foods; thus they are given prior to ingestion. Sodium is encouraged due to the high sodium loss.

A group of nurses are discussing the most recent statistics on death due to prematurity in the United States. Which action, if implemented, would have the greatest impact on improving outcomes? a. Track the incidence of violent crime against pregnant women. b. Examine health disparities between ethnic groups. c. Improve women's access to receiving prenatal care. d. Identify specific national health goals related to maternal and infant health.

c. The best way to have a positive impact on perinatal outcomes and reduce prematurity is to improve women's access to prenatal care. Tracking the incidence of violent crime does not give information on how to improve outcomes, nor does examining health disparities between ethnic groups or identifying specific national goals related to maternal and infant care without acting on the information. None of these address the true problem.

The case manager is looking over the inpatient census on the floor to determine which client would be the best candidate for discharge with home health care follow-up. Which client would the case manager recommend to the practitioner for discharge? a. 6-year-old asthmatic child who is wheezing and has an O2 saturation of 92%. b. 3-day-old infant with a bilirubin level of 20 who is currently under phototherapy. c. 10-year-old boy with an infected laceration on his leg who has been treated with IV antibiotics for 3 days. d. Adolescent who was admitted in sickle cell crisis yesterday and rates his pain as an 8/10.

c. The child with the infected laceration is on IV antibiotics and can safely be discharged to complete the medication at home with home health care. The 6-year-old is too unstable for discharge. The infant's bilirubin is too high for doing home health phototherapy and may need additional hydration. The adolescent is on sickle cell crisis and is not ready for discharge 1 day after admission.

A nurse is providing care to a client who has undergone a mastectomy. The nurse provides the woman with information about where to obtain a breast prosthesis. This is an example of which type of community-based nursing intervention? a. health education program b. health screening program c. health system referral d. telephone consultation

c. The nurse is passing along information about the location of and services offered for the client, an example of a health system referral. Health education programs assist clients in making health-related decisions about self-care and use of resources. Health screening programs focus on detecting unrecognized or preclinical illness among individuals such as mammography. Telephone consultation involves listening and providing support, information or instruction given over the phone, and documenting the interaction.

The nurse is concluding her initial visit with a client at 28 weeks' gestation on bedrest for premature rupture of membranes. What information should the nurse consider when determining the timing of the next home visit? a. the location of the home b. client's type of health insurance c. amount of supervision and health education needed by the client d. the home visit schedule and needs of the agency to plan the visit

c. The nurse should complete an assessment on the client to include the amount of education and ability of the client to understand the health care provider's orders. The nurse should allow the agency to work with the insurance company, and the location of the home should not influence the decision of the nurse.

A nursing instructor is teaching a group of nursing students about the various options available to provide nursing care in a community. The instructor determines the session is successful when the students correctly choose which action as the primary focus of home care nursing? a. Provide care based on insurance coverage. b. Teach and supervise caregivers. c. Provide direct client care. d. Act as a liaison between health care provider and family.

c. The primary focus of home care nursing is to provide direct care. Teaching and supervising caregivers and acting as a liaison between the health care provider and family are additional functions of the home care nurse that support the direct care. The nurse should be aware of potential insurance restrictions so that other options may be explored if insurance will not cover specific treatments or medications that the health care provider has determined essential to the client. In these instances, the nurse can then act as the advocate to help find the necessary resources the client may need.

While reviewing a woman's compliance with prenatal care, a nurse notes that which comment made by the woman may be a result of poverty rather than a cultural difference? a. "I just don't understand why I have to give up alcohol when I am pregnant." b. "In my culture, many women crave pica foods throughout the pregnancy." c. "I can't rest in the afternoon since I work all day, and then I have to pick my children up from day care." d. "I wish I could afford fresh fruits for vitamin C because I just don't remember to take those prenatal vitamins."

d. Many characteristic responses described as cultural limitations are actually the consequences of poverty (e.g., parents seeking medical care for their children late in the course of an illness or a woman not taking prenatal vitamins during pregnancy). Solving these problems may be a question of locating adequate financial resources and may not be related to cultural differences. Reducing alcohol intake may be an addiction problem. Many women work when they are pregnant and cannot take an afternoon rest. Working may or may not be due to poverty, so the nurse will have to ask additional questions before concluding the woman lives in poverty.

A woman has presented to the clinic with her sick school-aged child. The child's mother reports she rarely has enough money to meet the health care needs of her chronically ill child. What information should be provided to the woman? a. Medicaid is a federal program designed to meet the specific needs of children. b. Medicaid is a state assistance program that provides health care for all children under the age of 13. c. Medicare may be available to help with the health care needs of indigent children. d. Medicaid may be available to low-income parents and their children.

d. Medicaid is a joint federal and state program that provides health insurance to low-income parents and their children. It is state-administered, and each state has its own set of guidelines.

A family is seeking sources for payment of health care costs. When talking with the family, the nurse determines that the family makes too much money to qualify for the federal program. Which source would the nurse likely recommend? a. Medicaid b. Special Supplemental Nutrition Program for Women, Infants, and Children (WIC) c. Centers for Disease Control d. State Child Health Insurance Program

d. The State Child Health Insurance Program, first known by its acronym "SCHIP" now referred to as "CHIP," was enacted in 1997. CHIP provides health insurance to newborns and children in low-income families who do not otherwise qualify for Medicaid and are uninsured. Based on the information given, the family does not qualify for Medicaid, a federal program that supplies grants to states to provide health care for individuals who have low incomes and meet other eligibility criteria. The Special Supplemental Nutrition Program for Women, Infants, and Children (WIC) provides nutritional services to low-income, nutritionally at-risk pregnant, breast-feeding, and postpartum women and their children (as old as 5 years). The Centers for Disease Control does not provide funding for health care services.

A nurse is addressing a group of women on the issue of women's health during their reproductive years. Which reason does the nurse provide regarding the need for comprehensive, community-centered care to women during this time period? a. Women have more health problems during their reproductive years. b. Increased stress causes more health problems during their reproductive years. c. A woman's immune system weakens immediately after birth. d. Women's health care needs change with their reproductive goals.

d. The nurse should inform the women that comprehensive community-centered care should be given to women during their reproductive years. This is because as their reproductive goals change, their health care needs change as well. A woman's immune system does not weaken immediately after birth. Similarly, women do not have more health problems specifically during their reproductive years, nor are they more susceptible to stress during their reproductive years.

The nurse working in a free health clinic is providing care to a 1-year-old girl and her single teenage mother. Which action would be most important initially? a. discussing family health history b. assessing the child's cognitive level c. obtaining food stamps for the family d. observing the mother/child interaction

d. The nurse's initial interactions with the parent and child can provide valuable insights into maternal-child attachment. This information can be used to inform other aspects of the health assessment. This observation is likely to precede formal assessments, such as cognitive assessment and family health history. At this point, it is not known if food stamps are needed. Assessment is the priority.

A woman who just gave birth tells the obstetrical nurse that her mother has noticed that so much has changed in maternity care in recent years. One change she could likely be referring to is: a. infections are prevented as births happen in highly advanced, sterile rooms. b. the family is now allowed to visit at prescribed visiting hours. c. the newborn stays in a newborn nursery for care to allow the mother to rest. d. newborns stay at the mother's bedside as long the infant is well.

d. There has been increased access to care for all women (regardless of their ability to pay) and many hospital redesigns of labor, birth, and recovery rooms and postpartum spaces aimed at keeping families together during the birth experience and minimizing interruptions. Rooming-in and liberal visiting policies allow parents and other family members to participate in the child's care. Births happen in birthing suites/rooms as opposed to sterile birth rooms. Family is allowed to visit at most times of the day and night. Rooming in with the newborn is considered standard care now.


संबंधित स्टडी सेट्स

Face and Neck Trauma JBlearn Interactive Lecture

View Set

Chapter 6: Memory Psychology 1010

View Set

RN professionalism and Leadership Assessment

View Set

Forensic Science Pop Culture Trivia

View Set

Bontrager Workbook: Chapter 5-Humerus and Shoulder Girdle

View Set

Chapter 13 Human Resource Management

View Set